You are on page 1of 201

JURISPRUDENCE

Topic 1: Legal System of the World & Indian Legal System


Topic 2: Basic Theories of Law
Topic 3: Hierarchy of Courts & Jurisdiction
Topic 4: Legal service & Lok Adalat
Topic 5: Methods of legal Studies & Rules of Interpretation
Topic 6: Source of Law
Topic 7: Separation of Power
Topic 8: Rule of Law
Topic 9: Legal Profession
Topic 10: Utilization of Library in Legal Studies
Legal System of the World & Indian Legal System
(Q) Enumerate different kinds of legal systems in the world. What is the
difference between the Common Law and Civil Law Systems? What kind of
legal system exists in India? Explain the elements of the Indian Legal System.
[2019(1)]

(Q) What is legal system? What are the main components of the Indian legal
system? Discuss. [2018(1)(a)]

(Q) State the distinguishing feature of the common law and civil law/Romano-
Germanic Legal Systems. [2018(1)(b)]

(Q) Which are the major legal systems of the world? Discuss their salient
features. Have these legal systems left any impact on the development of India‘s
legal system? [2017(1)]

(Q)Identify major legal systems in the world and show how the civil law system
can be differentiated from the common law system Explain why India is called a
mixed legal system. [2016(1)]

(Q)Common Law System. [2015(1)(a)]

Legal System of the World:

Legal systems vary from country to country, and sometimes within a single
country. Although they develop in different ways, legal systems also have some
similarities based on historically accepted justice ideals. Legal systems do fall into
groups or patterns with some similar features within each group. Among the main
groups that you might encounter are: 1) common law; 2) civil law; 3) religious
law; and 4) customary law. Many countries employ more than one of these
systems at the same time to create a hybrid system. In some places, the current
security situation can also impact the way that legal systems work. It is helpful to
understand some of the similarities and differences as you move through your case.

English Common Law Legal Systems


Historical Development:

English common law emerged from the changing and centralizing power of the
kings during the middle Ages. After the Norman Conquest in 1066, medieval
kings began to consolidate power and establish new institutions of royal authority
and justice. New forms of legal action established by the crown functioned
through a system of writs, or royal orders, each of which provided a specific
remedy for a specific wrong. The system of writs became so highly formalized that
the laws the courts could apply based on this system often were too rigid to
adequately achieve justice. In these cases, a further appeal to justice would have to
be made directly to the king. This difficulty gave birth to a new kind of court, the
court of equity, also known as the court of Chancery because it was the court of
the king‘s chancellor. Courts of equity were authorized to apply principles of
equity based on many sources (such as Roman law and natural law) rather than to
apply only the common law, to achieve a just outcome.
Courts of law and courts of equity thus functioned separately until the writs system
was abolished in the mid-nineteenth century. Even today, however, some U.S.
states maintain separate courts of equity. Likewise, certain kinds of writs, such as
warrants and subpoenas, still exist in the modern practice of common law. An
example is the writ of habeas corpus, which protects the individual from unlawful
detention. Originally an order from the king obtained by a prisoner or on his
behalf, a writ of habeas corpus summoned the prisoner to court to determine
whether he was being detained under lawful authority. Habeas corpus developed
during the same period that produced the 1215 Magna Carta, or Great Charter,
which declared certain individual liberties, one of the most famous being that a
freeman could not be imprisoned or punished without the judgment of his peers
under the law of the land—thus establishing the right to a jury trial.
In the middle Ages, common law in England coexisted, as civil law did in other
countries, with other systems of law. Church courts applied canon law, urban
and rural courts applied local customary law, Chancery and maritime courts
applied Roman law. Only in the seventeenth century did common law triumph
over the other laws, when Parliament established a permanent check on the power
of the English king and claimed the right to define the common law and declare
other laws subsidiary to it. This evolution of a national legal culture in England
was contemporaneous with the development of national legal systems in civil law
countries during the early modern period. But where legal humanists and
Enlightenment scholars on the continent looked to shared civil law tradition as well
as national legislation and custom, English jurists of this era took great pride in the
uniqueness of English legal customs and institutions.
That pride, perhaps mixed with envy inspired by the contemporary European
movement toward codification, resulted in the first systematic, analytic treatise on
English common law: William Blackstone‘s (1723-1780) Commentaries on the
Laws of England. In American law, Blackstone‘s work now functions as the
definitive source for common law precedents prior to the existence of the United
States.

Present Scenario

Common law was originally developed by judges through case-by-case court


decisions, rather than through legislation enacted by a legislature. In this system,
much of the law is made by judges‘ decisions, called precedent. This means that if
a similar case has been resolved by a court in the past, a court is bound to follow
the reasoning used in the prior decision. While judges are very important in
common law systems, legislatures still have a part to play. Common law systems
also rely on statutes that are passed by the legislature or a parliament, and judges
have the role of interpreting how the legislature‘s laws are applied in individual
cases.

General principles:
• The laws governing a case are based on both legal precedent, created by
judges, and statutory laws, created by legislatures;
• Usually an adversarial system, where the judge acts as an impartial referee
between opposing parties to a case; as opposite inquisitorial system
• A jury may determine the facts , and a judge will decide the law to be
applied;
• There is an active role for prosecutors and defense attorneys;
• Victims have a role as witnesses and may have rights as a victim to receive
information and limited participation – however, victims are not a party in
criminal cases;
• The U.S. and the U.K. are examples of common law systems.
In common law legal systems, legal proceedings are mostly adversarial, rather
than inquisitorial. This means that for the most part, two opposing parties
(adversaries) appear before a judge who moderates. Defendants are entitled to
be present and to be represented by a lawyer. The attorneys on both sides
generally have an active role in representing their clients throughout the case and
in presenting evidence and arguments in court. A jury of people without legal
training can decide the facts of the case, and if there is a conviction, then a judge
determines the appropriate sentence based on the jury‘s verdict.

Common Law Countries: USA, UK, India, Canada

Roman Civil Law Legal Systems:

Historical Development:

The term civil law derives from the Latin ius civile, the law applicable to all
Roman cives or citizens. Its origins and model are to be found in the monumental
compilation of Roman law commissioned by the Emperor Justinian in the sixth
century CE. While this compilation was lost to the West within decades of its
creation, it was rediscovered and made the basis for legal instruction in eleventh-
century Italy and in the sixteenth century came to be known as Corpus iuris
civilis. Succeeding generations of legal scholars throughout Europe adapted the
principles of ancient Roman law in the Corpus iuris civilis to contemporary needs.
Medieval scholars of Catholic church law, or canon law, were also influenced by
Roman law scholarship as they compiled existing religious legal sources into their
own comprehensive system of law and governance for the Church, an institution
central to medieval culture, politics, and higher learning. By the late Middle Ages,
these two laws, civil and canon, were taught at most universities and formed the
basis of a shared body of legal thought common to most of Europe. The birth and
evolution of the medieval civil law tradition based on Roman law was thus
integral to European legal development. It offered a store of legal principles and
rules invested with the authority of ancient Rome and centuries of distinguished
jurists, and it held out the possibility of a comprehensive legal code providing
substantive and procedural law for all situations.
As civil law came into practice throughout Europe, the role of local custom as a
source of law became increasingly important—particularly as growing European
states sought to unify and organize their individual legal systems. Throughout the
early modern period, this desire generated scholarly attempts to systematize
scattered, disparate legal provisions and local customary laws and bring them
into harmony with rational principles of civil law and natural law. Emblematic of
these attempts is the Dutch jurist Hugo Grotius‟ 1631 work, Introduction to Dutch
Jurisprudence, which synthesized Roman law and Dutch customary law into a
cohesive whole. In the eighteenth century, the reforming aspirations of
Enlightenment rulers aligned with jurists‘ desire to rationalize the law to produce
comprehensive, systematic legal codes including Austria‘s 1786 Code of Joseph II
and Complete Civil Code of 1811, Prussia‘s Complete Territorial Code of 1794,
and France‘s Civil Code (known as the Napoleonic Code) of 1804. Such codes,
shaped by the Roman law tradition, are the models of today‘s civil law systems.

Present Scenario

Civil law systems place greater emphasis on legal codes crafted by the legislature.
Civil law statutes tend to be more detailed than statutes under common law
systems, and contain continuously updated legal codes that specify all matters
capable of being brought before a court, the procedure to be followed, and the
appropriate punishment.
Civil law systems rely less on judges and more on academic legal experts to make
legal interpretations. In a civil law system, the judge‘s role is to establish the facts
of the case and to analyze and apply the legislature‘s written laws. Because of
this, legislators and legal scholars who draft and interpret the codes are important
in civil law legal systems.
General principles:
• Most of the law is statutory law created by legislatures and not by judges
following precedent;
• Usually an inquisitorial system, where an investigating judge is actively
involved in investigating the facts of a case;
• Juries are rarely used; a judge or panel of judges will decide the facts and
the law to be applied;
• Prosecutors and defense attorneys may play a more limited role;
• Victims may be parties and have rights regarding their involvement, which
may include having their own attorneys and filing the initial charges;
• In many civil law systems, victims may bring civil claims, e.g., for monetary
damages, in the context of a criminal prosecution.
• Many European countries, including France and Germany, and a number of
North, Central and South American countries, like Mexico and Brazil, are
examples of civil law systems.

The role of judges is different in civil law systems compared to common law
systems. There are two types of judges in a civil law system: an investigating
judge (or magistrate) and trial or sitting judges. Civil law systems are based on
the belief that justice is best served when a judge is an active participant in
investigating the facts of the case, thus the investigating judge or magistrate will
typically lead the investigation. Unlike common law systems, which focus on the
trial to determine the facts, civil law legal systems mostly focus on pre-trial
investigation and hearings to establish the facts. The actual trials can be relatively
brief and informal because the trial judge will review the case file developed by
an investigating judge. During trial, witnesses are generally allowed to give
additional kinds of evidence and the defendant often gives a statement. Cross
examination is rare.

Civil Law Countries: China, Japan, Germany, France, Spain

Difference/Similarities between Common Law and Civil Law:

Civil Law Common Law

Legal System Legal system originating in Legal system characterized by


Europe whose most case law, which is law developed
prevalent feature is that its by judges through decisions of
core principles are codified courts and similar tribunals.
into a referable system
which serves as the
primary source of law.
Role of judges Chief investigator; makes Makes rulings; sets precedent;
rulings, usually non- referee between lawyers. Judges
binding to 3rd parties. In a decide matters of law and, where
civil law system, the a jury is absent, they also find
judge‘s role is to establish facts. Most judges rarely inquire
the facts of the case and to extensively into matters before
apply the provisions of the them, instead relying on
applicable code. Though arguments presented by the part
the judge often brings the
formal charge.
Countries Spain, China, Japan, United States, England,
Germany, most African Australia, Canada, India
nations, all South
American nations (except
Guyana), most of Europe
Constitution Always Not always

Precedent Only used to determine Used to rule on future or present


administrative of cases
constitutional court matters
Role of jury In cases of civil law, the Juries are comprised only of
opinion of the jury may not laypersons — never judges. In
have to be unanimous. the U.S., juries are employed in
Laws vary by state and both civil and criminal cases.
country. Juries are present Their function is to weigh
almost exclusively in evidence presented to them, and
criminal cases; virtually to find the facts and apply the
never involved in civil law.
actions. Judges ensure law
prevails over passion.
History The civil law tradition Common law systems have
developed in continental evolved primarily in England
Europe at the same time and its former colonies,
and was applied in the including all but one US
colonies of European jurisdiction and all but one
imperial powers such as Canadian jurisdiction. For the
Spain and Portugal. most part, the English-speaking
world operates under common
law.
Sources of Law 1. Constitution 2.
Legislation – statutes and 1. Constitution (not in the UK) 2.
subsidiary legislation 3. Legislation – Statutes and
Custom 4. International subsidiary legislation 3. Judicial
Law 5. [Nota bene: It may precedent – common law and
be argued that judicial equity 4. Custom 5. Convention
precedents and conventions 6. International Law
also function within
Continental systems, but
they are not generally
recognized.
Type of Inquisitorial. Judges, not Adversarial. Lawyers ask
argument and lawyers, ask questions and questions of witnesses, demand
role of lawyers demand evidence. Lawyers production of evidence, and
present arguments based on present cases based on the
the evidence the court evidence they have gathered
finds.
Evidence demands are Widely understood to be a
within the sovereign necessary part of the litigants‘
Evidence Taking inquisitorial function of the effective pursuit or defense of a
court — not within the claim. Litigants are given wide
lawyers‘ role. As such, latitude in US jurisdictions, but
―discovery‖ by foreign more limited outside the US.
attorneys is dimly viewed,
and can even lead to
criminal sanctions where
the court‘s role is usurp.
Both systems have similar Both systems have similar
Evolution sources of law- both have sources of law- both have
statutes and both have statutes and both have case law,
case law, they approach they approach regulation and
regulation and resolve resolve issues in different ways,
issues in different ways, from different perspectives
from different
perspectives

Religious Law Legal Systems – Canon Law, Islamic Law, and Talmudic Law
In traditional religious legal systems, criminal law is based mainly on religious
texts and interpretations of those texts. Religious legal systems include Canon
law (e.g., Roman Catholic, Anglican), Islamic law, and Talmudic (Jewish) law.
Even in countries that have common or civil law systems, religious courts may
exclusively hear some matters (examples: marriage, divorce and inheritance) for
the followers of individual faiths if the country has different religious groups.
Some countries incorporate some aspects of religious law into civil or common
law systems (see Hybrid Legal Systems, below). In some countries, elements of
Hindu, Buddhist, Confucian, or Sikh laws may be incorporated into the legal
systems.
Canon law is the body of laws and regulations made or adopted by ecclesiastical
authority for the government of Christian communities. Although at the root of
much of the Western legal tradition, Canon law is applied very seldom across the
world today.
Islamic law is the religious law that applies in many Muslim communities to
varying extents. Islamic law is a basic set of rules that are based on two primary
sources: the Koran and the Sunnah (the model written behavior of the Prophet
Muhammad), but also on a variety of legal interpretations. In some countries,
Islamic law only governs family matters and all other legal issues are handled
through the secular court system.

Other countries take a variety of approaches:


some apply Islamic in combination with secular law; in others, Islamic is applied
in a modified form; and some other countries apply strict Islamic legal
interpretations in all courts. Legal interpretation may depend on the branch of
Islam practiced within a country. Islamic law generally recognizes a role for
victims in the justice system.
Talmudic law applies in some countries and regions with heavily concentrated
Jewish populations. The major sources of law in this legal system are the Torah
(the first five books of the Old Testament in the Christian western tradition), both
written and oral, as well as the Talmud. The Talmud is a written commentary of
valuable opinions about the content of the written and oral Torahs. Similar to the
importance placed on interpretations of academic scholars and legal experts in
the civil law system, the Talmudic legal system also relies on the written opinions
of those learned in the law.

Hindu Law:
Given the historical bases of the term 'Hindu', Hindu law has had varied
understandings. In the ethnic Indian context, some have understood Hindu law to
include the diverse laws prevalent in India from the ancient Vedic times until 1772
when the British adopted rules for administration of justice in Bengal. Some have
used it to distinguish from the Islamic legal system that existed in parts of India
annexed by the Muslim Mughal Empires between thirteenth and sixteenth
centuries, as well as the British legal system from 1772 onwards. Others have
categorized Hindu law as being applicable only to those communities that were
subjected to it while others followed their own diverse customary laws.

Hindu law can primarily be divided into three categories: the 'Classical Hindu
Law', the 'Anglo Hindu Law', and the 'Modern Hindu Law'. These three
divisions also have an historic context.

The Classical Hindu Law includes the diverse legal practices connected with the
Vedic traditions in some ways and existing from the Vedic times until 1772 when
the British adopted rules for administration of justice in Bengal. The Anglo-Hindu
Law was evolved from the classical Hindu law during the British rule in India
from 1772 to 1947. The British adopted the modern law or the English legal
system and replaced the existing Indian laws except for family or personal laws in
matters such as marriage, inheritance and succession of property. Family law or the
personal law applicable to Hindus is the Modern Hindu Law.

Other Legal Systems - Customary Law and Hybrid Legal Systems

Customary law - Countries that do not historically have strong formal justice
systems may rely upon customary law. Customary law is generally found at the
tribal or local level in districts, counties, and villages, and is a vast set of practices
that vary from community to community. These traditional rights and obligations
are generally unique to a particular society or culture. Customary law is based on
longstanding local customs which greatly shape the ideas of justice. Customary
law is often oral, not written. It generally uses a case-by-case approach to dispute
resolution. Customary law can sometimes involve informal mediation or
arbitration, and typically does not include a formal trial. Customary law frequently
becomes a function of tribal or village elders in the absence of a functioning formal
justice system, as in a conflict or post-conflict country.
Hybrid Legal Systems - Countries may have mixed legal systems that draw on
common law and/or civil law traditions, mixed with customary or religious laws.
For example, Islamic law operates alongside civil or common law in some
countries. India has a common law system combined with separate personal law
codes that apply to Muslims, Christians, and Hindus. Pakistan‘s legal system
combines common law and Islamic law. Nepal‘s legal system combines Hindu
legal concepts and common law. The Philippines has a mixed legal system of civil,
common, Islamic and customary law. Sri Lanka‘s legal system combines civil law,
common law and customary law. Most Pacific island countries recognize
customary law as well as common law. In some African countries, customary law
still has great influence, and local values play a role in informal justice systems
and accountability.

Indian legal System:

What is a legal system? A legal system encompasses a set of legal principles and
norms to protect and promote a secure living to its subjects in a cultured society. It
recognizes rights, prescribes duties of people and provides the ways and means of
enforcing the same. To achieve this particular objective, the legal system
considering the sociological, economic and political conditions in the society
designs its own goals and evolves a set of principles/rules/laws which help the
society to attain its identified goals.
A system connotes a coherent whole. It is animated by a philosophy or ideas which
connect its different parts leading to a harmonious working. The corpus of the
system is its variable elements. Changes in the laws may result from legislation
and judicial decision. But the concepts and methods of the system are its constant
elements.1 In the material content of laws there is much overlapping among the
laws of different countries. The major legal systems of the world may be classified
as (1) the common law, (2) the civil law, (3) the socialist legality and (4)
religious systems of law. While the Indian legal system is basically a common law
system, it contains elements of the other three systems as well. It is an open system
taking in what is most suitable to our needs.

The common law base


British rule in India introduced the common law into this country. This provided
the basis of our present legal system. The significance of the coming of the
common law to India has not received adequate recognition in spite of the attention
called to it by several legal scholars.

In Additional District Magistrate, Jabalpur v: Shivakant Shukla the question


which arose was whether the right to personal liberty under the common law and
the relevant statutes became unavailable only because the fundamental right to
personal liberty guaranteed by article 21 was suspended during the Emergency. It
is submitted with great respect that the following legal position might have been
found relevant in considering the question:

(1) The common law, custom and general principles of justice, equity and good
conscience which are drawn upon by the courts both in England and India provide
the basis and the environment in which statutes are enacted. It is a settled principle
of interpretation of statutes that the preexisting common law and principles of
justice, equity and good conscience are not altered by statute law except to the
extent of repugnancy between the two.
(2) In spite of the march of statute law in England (as in India) it has been
observed that "it is only where constitutional law is concerned, in that small but
vital sphere where liberty of person and of speech are guarded that it means the
rule of the common law. For here alone has Parliament seen fit to leave the law
substantially unaltered and to leave the protection of the freedom of individuals to
the operation of the common law." The same observation would hold good in India
inasmuch as freedom of the individuals is assumed to be the rule of law except
insofar as statutes restrict it. Even in England, the sovereignty of Parliament has
meant, "the supremacy of the existing law so long as Parliament was fit to leave it
unaltered." It is well known that Parliament identified itself with the cause of the
supremacy of the law and did not alter by statute the basic principle that the
individual enjoys all the liberties unless restrictions on them are placed by the
statutes.
(3) Judicial review of administrative action existed in India even before the
Constitution. The law courts could draw upon the common law principles of
justice, equity and good conscience and the relevant statutes to correct illegalities
in administrative action and give reliefs in suits filed for that purpose. Even after
the Constitution when High Courts are empowered to exercise the power of
judicial review of administrative action under article 226 of the Constitution, these
reliefs can still be obtained by way of suits. This is sufficient to show that the
Constitution was not to wipe out the pre-existing law which was not inconsistent
with the Constitution. This is confirmed by article 372 (1).
The reason why the Constitution had to be made the supreme law was that in
addition to judicial review of administrative action, the superior courts had to be
given the power of judicial review of legislation. These two powers of judicial
review are distinct from each other and are exercised at two different levels The
rule of law is, therefore, to be distinguished from the rule of the Constitution or
that of the higher law in India. Even if the latter or a part of it is suspended when
the enforcement of certain fundamental rights is suspended during emergency, the
former would continue to operate. The suspension of the rule of the Constitution or
of the higher law is intended to make legislation temporarily invulnerable. It can
have, however, no effect on the rule of the ordinary law which is directed mainly
against illegalities of administrative action. No difficulty is, therefore, experienced
in the co-existence of the Constitution with the pre-existing law. It is to be noted
that the fundamental rights are guaranteed expressly by article 13 only against
inconsistent legislation. Similarly, the Constitution is supreme law against any
inconsistent ordinary law. A fortiori, therefore, the fundamental rights and the rest
of the Constitution would prevail against administrative action. This should not,
however, cloud our understanding of the basic purpose of the Constitution which
was to provide the supreme law as against legislation. For, before the
commencement of the Constitution, ordinary legislation, however unjust, had to
prevail against the liberties of the people based upon the common law or the
principles of justice, equity and good conscience. The effect of the Constitution
was to place the fundamental rights and other provisions of the Constitution above
the ordinary law and to protect them against any inconsistent legislation.
(4) While legislation can be challenged as being inconsistent with the Constitution,
administrative action may be reviewed on totally different grounds such as being
based on mala fides or irrelevant considerations or being without jurisdiction or
showing apparent error of law on the face of the record, etc. The existence of the
Constitution and the remedies given by it are not a sine qua non for obtaining
judicial review against administrative action by way of suits even though a more
expeditious remedy may be available by way of writ petitions when disputed
questions of fact are not involved. It would appear, therefore, that the suspension
of the enforcement of a fundamental right during Emergency should not affect the
ordinary remedy of judicial review of administrative action without seeking the
enforcement of a fundamental right as embodied in the Constitution and not
challenging the validity of any law but restricted only to the validity of
administrative action on the abovementioned grounds all of which have existed
from before the Constitution.
Even after the making of the Constitution and enactment of relevant statutes, the
ecology of the Constitution and the statutes, is formed by "that part of common law
which has been received in India as rules of 'justice, equity and good conscience' as
suited to the genius of this country". This much of common law is in force in India
as recognised by article 372 (1) of the Constitution. A nine-judge Bench of the
Supreme Court had this to say about the common law in India:
It is well-known that the common law of England was applied as such in the
original sides of the High Courts of Calcutta, Bombay and Madras, and that in the
mofussil courts the principles embodied in the common law were invoked in
appropriate cases on the ground of justice, equity and good conscience.

The rule of law


The first direct product of the common law system is the public law. The most
important principle of it is the rule of law. The rule of law implies that government
authority may only be exercised in accordance with written laws which are adopted
through an established procedure. It means "government of laws and not of men."
The principle is intended to be a safeguard against arbitrary rulings in individual
cases. The doctrine does not speak anything about the 'justness' of the laws
themselves, but simply how the legal system upholds the law. The concept of rule
of law is generally associated with several other concepts like: (a) presumption of
innocence - all individuals are innocent until proven otherwise; (b) double
jeopardy - individuals may only be punished once for every specific crime
committed. Retrials may or may not be permitted on the grounds of new evidence;
(c) legal equality – all individuals are given the same rights without distinction to
their social stature, religion, political opinions, etc.; and (d) habeas corpus - term
meaning 'you must have the body'. A person who is arrested has the right to be told
what crimes he or she is accused of, and to request his or her custody be reviewed
by judicial authority. Persons unlawfully imprisoned have to be freed.
The classic description of the doctrine of rule of law by A. V. Dicey includes (1)
the absence of arbitrary powers on the part of the Government which always acts
according to law, (2) legal equality, that is, no man is above the law and that
everyone is subject to the ordinary law of the land and is amenable to the
jurisdiction of the ordinary courts and tribunals, and (3) the customary and the
common law rights of the people resulting in judicial decisions to form the
general rules of constitutional law. Basically, individual liberty was the outcome of
the rule of law. It could not be affected except in accordance with law. The burden
was, therefore, on the state or the public officials to show that their action affecting
an individual is justified by law.

Rule of the Constitution or the higher law in India


unfortunately, much British Indian legislation denied the enjoyment of civil and
political rights to the Indian citizens. The letters of the law, therefore, went against
the spirit of the law. Taking cue, therefore, from the earlier American example, the
Constitution of India was made the supreme law of the land in 1950. The
Constitution is based on the ideals of justice, social, economic and political, liberty
of thought, expression, belief, faith and worship, equality of status and of
opportunity and fraternity assuring the dignity of the individual and the unity and
integrity of the nation. The rule of law was, therefore, placed on a footing higher
than ordinary legislation.

Constituent power is, thus, superior to ordinary legislative power. Though under
the Constitution, the Parliament and the state legislatures have the power to make
laws within their respective jurisdiction; this power is not absolute in nature. The
Constitution vests in the judiciary the power to adjudicate upon the constitutional
validity of all laws. If a law made by the Parliament or the state legislatures
violates any provision of the Constitution, the Supreme Court has the power to
declare such a law invalid or ultra vires. Thus, unlike the British Parliament which
is a sovereign body, the powers and functions of the Indian Parliament and the
state legislatures are subject to limitations laid down in the Constitution.

Protection against constitutional amendments


The Constitution of India was apparently intended to entrench the more permanent
values cherished by the society, particularly in its part III. The founding fathers
wanted the Constitution to be an adaptable document rather than a rigid
framework for governance. Hence Parliament was invested with the power under
article 368 to amend the Constitution.
However, with the intention of preserving the original ideals envisioned by the
Constitution makers, the Supreme Court in Kesavananda Bharati v. State of
Keral held that Parliament could not distort, damage or alter the essential features
or the 'basic structure' of the Constitution under the pretext of amending it. The
enunciation of this doctrine can be said to be an attempt to preserve the spirit of the
rule of law from the otherwise unlimited power of Parliament to amend the
Constitution. Thus, Parliament's power to amend the Constitution is not absolute
and the Supreme Court is the final arbiter over and interpreter of all constitutional
amendments. Since what constitutes 'basic structure' is not spelt out in the
Constitution, the features that constitute the same cannot be laid down until another
authoritative pronouncement is rendered by the apex court. However, it can be said
that the sovereign, democratic and secular character of the polity, rule of law,
independence of the judiciary, fundamental rights of citizens, etc. are some of the
essential features of the Constitution.
Fundamental rights
The provisions of part III of the Constitution which enumerates fundamental
rights are more elaborate than those of any other existing written Constitutions of
the world and cover a wide range of topics. The purpose of the fundamental rights
is to act as limitations not only upon the powers of the executive but also upon the
powers of the legislature. These rights include individual rights common to most
liberal democracies such as equality before the law or the right to equality, tight to
freedom of religion, right against exploitation, cultural and educational rights,
freedom of speech and expression, freedom of association and peaceful assembly,
and the right to constitutional remedies for the protection of civil rights such as
habeas corpus.
Independence of judiciary
India is credited with having the most powerful and independent judiciary in the
world. Indian judiciary owes its origin to the judicial system which existed in the
British India. After independence, the constituent assembly, which drafted the
Constitution provided for the establishment of a three-tier judiciary which is
completely independent of the other two organs of the state - the executive and the
legislature. The reason for making judiciary independent was that during the
British rule executive and judicial functions were combined in the collector-
magistrate in a district, making him a local dictator. The framers of the
Constitution did not want this to be repeated and hence established a judicial
system under which from the highest court of the land to the lowest, they function
in a spirit of judicial independence, i.e., independent of the executive and the
legislature. Independence of the judiciary is thus a basic structure or a basic feature
of our Constitution.There are two dimensions of judicial independence: (a)
individual independence of a judge; and (b) institutional or collective
independence of the court or tribunal of which that judge is a member. The
Supreme Court stands at the apex of the judicial hierarchy along with high courts
and subordinate judiciary, all with specified powers and functions.

Change in the concept of law


The British tradition introduced in India was that the function of the judges was to
interpret and apply the law and not to make the law. The American Constitution
gave a wider role to the judges by making legislation itself subject to judicial
review. Opinions have been expressed in England by Lord Hailsham, T. B. Smith
and Sir Leslie Scarman, that a Bill of Rights should,, be enacted even in England
and be placed beyond the reach of ordinary parliamentary legislation. But the
majority of the English judges prefer their modest role. The British view is based
on the positivist concept of law, which is the lawyers' view of the law. According
to this view, anything is law if it meets the conditions of validity laid down in the
system's rules of recognition or any other rules of the system. But the American
Realist School and other jurists such as Lon Fuller emphasize moral purpose of law
and the necessity to allow the courts to have review powers over legislation and
administrative action to ensure conformity to the rule of law. From the inception of
the Constitution of India also a tug of-war between two views of law may be seen
to be going on. One view is that the final say as to what the law is must be with
Parliament particularly when Parliament resorts to constitutional amendments to
make the intention of the Constitution clear. On this view the judges should not
make innovations in law by construing the Constitution to reserve the last say to
the courts. The competing view is that the Constitution itself has intended the
lasting values enshrined in it to be beyond the day to day controversies and that the
function of the judges is to preserve these values by an appropriate interpretation of
the Constitution (e.g., the majority decision in Kesavananda Bharati's case). This
is not to thwart the will of the people, but, on the contrary, to give effect to the
same will as enshrined in the Constitution with a view to ensuring to them a
comparative permanence. It is to be seen whether Parliament reconciles itself to
this judicial view of law and the judicial function either by refraining from
amending the Constitution to abrogate the doctrine of basic structure or features of
the Constitution as being unamendable, or by incorporating into the Constitution
the doctrine by further constitutional amendment.

Justice, equity and good conscience


whenever statute law is absent, the judges, according to many state statutes, are to
be guided in deciding cases by principles of "justice, equity and good
conscience". Initially this expression was construed by the judges to mean the
rules of the common law except in so far as any particular rule was unsuitable for
being applied to Indian conditions. The judges have felt free in scrutinizing the
common law rules in their application in India. Certain archaic or unjust doctrines
such as actio personalis moritur cum persona (a personal action dies with the
person) or the doctrine of common employment protecting the employer from
liability in tort to an employee for the fault of another employee, have been
rejected by Indian judges as being inapplicable to Indian conditions. It would
appear that "justice, equity and good conscience" should provide a strong basis to
the law courts in India to decide cases not covered by statutes in constructive spirit
to find out just solutions of new problems of law which are ever arising in the law
courts. The statutes expressly empower the judges to act on these principles.

Roman Civil law in Indian Legal System:

Speaking on the Charter Bill of 1833 Macaulay said:


I believe that no country ever stood so much in need of a code of laws as India,
and I believe also that never was a country in which the want might so easily be
supplied. Section 53 of the Charter Act, 1853 declared that it was expedient:
that such laws as may be applicable in common to all classes of the inhabitants...
due regard being had to the rights, feelings and peculiar usages of the people,
should be enacted: and that all laws and customs having the force of law should
be ascertained and consolidated and, as occasion may require, amended.

The first Law Commission which drafted the Indian penal Code acknowledged
its
indebtedness to the French Penal code. In a letter of 2 May 1837 addressed to the
Governor General the Commission stated that it derived much valuable assistance
from the French code and from the decisions of the French courts of justice on
questions touching the construction of that Code.

The second Law Commission which sat in London from 1853 to 1856 expressed
its view that:
what India wants is a body of substantive civil law, in preparing which the law of
England should be used as a basis. It, however, emphasised that such a body of
law ought to be prepared with a constant regard to the conditions and institutions
of India, and the character, religious and usages of the population. It also stated
that in the social condition existing in India it was necessary to allow certain
general classes of persons to have special laws, recognised and enforced by our
courts of justice, with respect to certain kinds of transaction among themselves.

We shall refer to few instances where the influence of the civil law is clearly
discernible.
Section 11 of the Indian Evidence Act adopted in 1872 could not have been
enacted in a fit of absent-mindedness.
The section which lays down guidelines to determine relevance in the admissibility
of evidence is a clear, and presumably a deliberate, departure from the English
rule and brings the Indian law in this respect very relevant and fair. Another
provision which is of interest in this regard is section 165 of the Act. Commenting
on it, Stephen has said:
Section 165 is intended to arm the judge with the most extensive power possible
for the purpose of getting at the truth. The effect of this section is that in order to
get to the bottom of the matter before it the court will be able to look at and
enquire into every fact whatever.

In the law of contract, consideration plays a significant role in India as in England.


But the words of section 25 of the Indian Contract Act which accords validity to
a registered agreement, even though without consideration, appear to reflect the
concept of cause in French law.

In the early nineteen sixties a number of territories where the civil law prevailed
became parts of the Indian Union. In the Union territory of Goa, Daman and Diu,
Portuguese civil law was in force, even after the extension of several Indian
enactments to the territory, it is generally the provisions of the Portuguese Civil
Code which apply to the people of this territory in matters of personal law. In the
former French settlements of Pondicherry. Karaikal, Mahe and Yanam
which, when ceded, were formed into the Union territory now known as
Pondichery, there are Indian citizens who are governed in matters of personal law
by the provisions of the French civil code as they existed at the time of the
cession. There are also other renoncants who are French citizens living in
Pondicherry to whom provisions of the French Civil Code relative to personal law
will apply with all subsequent amendments. In these circumstances, the element of
the civil law in the fabric of Indian law cannot be brushed aside as negligible. And
this element affects domestic relations which are on negligible part of a citizen's
life. The customary laws of various tribal communities and other ethnic groups
also form part of the law administered in India. To cite one instance: matriliny
among the Mappila Muslims of Kerala, though not favoured by the tenets of Islam,
is permitted to play a decisive role in the rules of succession applicable to them.

When India adopts a civil code, under the directive in the Constitution it is likely
to be eclectic in character, it may have in it a harmonious admixture of various
laws based on religion and customary laws, as well as provisions derived from
western codes and the English common law. Owing to its eclectic character and
especially because it would attempt to harmonise provisions of personal laws
derived from religion prevalent in the region, the civil code may be found worthy
of emulation in south and southeast Asia. It may thus pave the way for unification
of laws, though perhaps limited geographically in extent. If in ancient days,
Indian culture was permitted, without any hitch or demur, to permeate social and
political institutions and life in general in this region, there is no reason why Indian
legal culture cannot play a similar role in the near future as well. What the
Napoleonic code has done for continental Europe, the Americas, and parts of Asia
and Africa, a well-framed Indian civil code may easily do for south and Southeast
Asia.

Personal laws
India is a land of diverse religions. Hindus, Buddhists, Jains Christians, Muslims,
Parsees, Sikhs and Jews form the nation. Each community has its own personal
laws governing marriage, divorce, infants, minors, adoption, wills, intestacy and
succession. The oldest part of our legal system is the personal laws governing these
different communities. Some of the laws which govern them include: Indian
Divorce Act, 1869; Indian Christian Marriage Act, 1872; Indian Succession Act,
1925; Parsi Marriage and Divorce Act, 1936; Dissolution of Muslim Marriage Act,
1939; Special Marriage Act, 1954; Hindu Marriage Act, 1955; Foreign Marriage
Act, 1969; and Muslim Women (Protection of Rights on Divorce) Act, 1986.

It is inevitable that the ideals animating these personal laws should change with
the change in the manners and customs of our society which are themselves
brought about by an increasing assimilation of the manners and customs of other
people in the world by our people. Recently necessary amendments have been
incorporated in the Special Marriage Act, Hindu Marriage Act, Parsi Marriage and
Divorce Act and the Indian Divorce Act. Under the Special Marriage Act, persons
can specifically register their marriage even though they are of different religious
faiths. The Act also provides that marriage celebrated under any other form can
also be registered under this Act, if it satisfies the requirements of the Act. By the
Special Marriage Laws (Amendment) Act, 2001 section 4(b)(iii) of the Hindu
Marriage Act, 1955 was amended to omit the words "or epilepsy", which was
earlier a statutory bar for marriage. Sections 36 and 38 have also been amended to
provide that an application for alimony pendente lite or the maintenance and
education of minor children be disposed of within 60 days from the date of service
of notice on the respondent.

Uniform civil code for the citizens


Article 44 of the Constitution envisages that the state shall endeavour to secure for
the citizens a uniform civil code throughout the territory of India. The framers of
the Constitution apparently felt that national integration would not be complete
unless everyone in India is governed by uniform laws which do not distinguish
between persons on the grounds of religion, etc. It is an anomaly that Hindus and
Muslims should be governed by different laws solely because they profess
different religions. The freedom of religion guaranteed by article 25 and the
protection given to the cultural and educational rights of the minorities by articles
29 and 30 do not require that people professing different religions and minorities
based on religion should be governed by personal laws. For, religion, cultural and
educational rights are equally guaranteed to everyone. The guarantees will be the
same irrespective of what the religion, culture and methods of education may be.
These fundamental rights have to be construed firstly in the light of entry 5 of List
III - Concurrent List of the Seventh Schedule of the Constitution. Both
Parliament and state legislatures are competent to legislate not only on marriage
and divorce, infants and minors; adoption; wills, intestacy and succession; joint
family and partition which are subject matter of personal laws but also on all
matters in respect of which parties in judicial proceedings were immediately before
the commencement of the Constitution subject to their personal laws. All this
would show that the fundamental rights guaranteed in the Constitution are not a bar
to the achievement of a uniform legal system throughout India. Secondly,
Parliament and state legislatures are expressly empowered to bring about such
uniformity.

Basic Theories of Law

John Austin

(Q) How did Austin demarcate the field of Jurisprudence? What is ―Law‖
according to him? How did Austin treat customary law and judge made law in his
command theory? What varieties of law are not covered by the command Theory
of Austin ? [2019(6)]

(Q) Explain Austin„s concept of ‗law‟ as discussed in his book ―The province of
Jurisprudence Determined”. How far is his concept applicable to the
Contemporary Indian Legal System? [2017(4)]

(Q) ―John Austin first demarcates the province of jurisprudence and then goes on
to explain what all is to be included in this province.‖ Elucidate the above
statement and make a special mention as to how Austin treats the Status of
customary and judge made law within such a province. [2016(5)]

(Q) Austin conceptualizes a sovereign as being illimitable and indivisible.


Critically evaluate his theory of sovereignty in the light of modern states having a
written constitution and characterized both by limits on legislative power and the
doctrine of separation of powers. [2018(4)]

Province of Jurisprudence Determined


Positivist or analysist school of thought is one of the various schools of thought
in jurisprudence. Legal positivists tend to view and understand the law as it is
made and deem it imperative to accept it as such without going into the
question of justness and unjustness, goodness and badness of the law.
Positivists concentrate on definiteness of the law and "Positus" means ―as is‖.
The law is treated as an imperative or command emanating from the State. For
this reason this school is also known as imperative school.
This school of thought originated from England and so called also as English
school. The founder of English Positivum is John Austin who was the first
occupant of the chair of Jurisprudence in the University of London.
Austin gave his theory of jurisprudence in six lectures called as-―Province of
Jurisprudence Determined‖. Before Austin gave his theory there was utter
confusion in the United Kingdom as to
What is law and
What is/are the source of the law?

Various customs and usages then prevalent, judicial precedents, dictates of the
Church etc. were treated either as the law or as the sources of the law. Lack of
codified laws and absence of a definite law maker added to the confusion.
This scenario formed the background when Austin visited Germany in early
nineteenth century. The German legal system already had codified and definite
civil and penal laws.

Thus, in order to impart certainty and definiteness to the English legal system
like Germany, he propounded his theory of sovereignty.

He started with the explanation of what is loosly termed as law. Loosly


speaking, law is a rule of guidance laid down for one intelligent being by
another intelligent being who has power over the former being. Thereafter, he
sought to filter out what is law simply and strictly so called (positive law) from
all that which is loosly called by common law as law
This process is like the process of metallurgy whereby the metal is extracted
from the ore.
Austin gave the status of positive law only to the laws given by political superior
to political inferior.

He categorized the law given by the superior to the inferior who are not political
and the law by analogy under the term ―Positive Morality”. He called it positive
because it was given by a man to another man and it was definite and certain. He
called it ―morality‖ because it was not backed by the legal sanction and thus
could be easily differentiated from the law.

According to the Austin, only positive law‘s determination falls within the
province of jurisprudence.

What is law: He said that every law or rule, properly so called, is a species of
command. Thus the word ―command‖ is a broader term and comprise of the term
―law‖

What is Command: is an expression of wish or desire of a person? Directing


another person to do or forbear from some act the violation Of which will be
followed by evil consequences on the person so directed.
Command envisages the presence Of two parties, the commander and the
commanded or the political superior and the political inferior.
Command is an authoritative desire which leaves no option but to obey it and
has an inbuilt notion of sanction i.e. should the desire be disobeyed sanction
follows automatically
A command is distinguished from other signification of desire not by the style
in which the desire is signified but by the power and purpose of the party
commanding to inflict an evil or pain in the case the desire be disregarded.
Case the desire be disregarded. The form may be that of request or appeal but
content may of compulsion and vice versa.
Command does not envisage reward: A command cannot be understood in
terms of reward because reward confers an eventual right in favour of the
person who complies with the expression of desire while its violation goes
unaddressed. In this way the command loses its imperativeness.

Command can have the element only of sanctions which imposes obligation on
the person commanded. And its imperative character is protected thereby.
A command can be given by words, spoken or written or by other signs e.g.
conduct.
Command, duty and sanction are inseparably connected terms: each embraces
the same idea as the others though each denotes those ideas in a peculiar order
or series. It can be explained thus:
(i) When I am talking directly of the expression or intimation of the wish. I
employ the term "command the expression or intimation of the wish being
presented prominently to my hearer while the evil to be incurred and the chance
of incurring it are kept in the background of my expression.
(ii) When I am talking directly of the chance of incurring the evil, or liability of the
obnoxiousness to the evil, I employ the term duty or the obligation; the liability or
obnoxiousness being signified implicitly.

(iii) when I am talking immediately of the evil itself I employ the term ―sanction‖;
the evil to be incurred being signified directly whilst the obnoxiousness to the evil,
with the expression of the wish are indicated indirectly or obliquely.

Thus each of three terms signifies the same notion, but each denotes a different
part of that notion and connotes the residue.

The relationship of command, duty and sanction can be better explained through
example:

In the expression-

―Do not jump the red light, otherwise you will be challenged‖

―Do not jump the red light‖ is a command as it is an authoritative expression of


the desire

―Otherwise you will be challenged‖ is a duty as it signifies the


obnoxiousness/liability to the evil.

―Challan‖ is a sanction as it is the evil itself.

What command is law: it is not every command that occupies the status of law the
positive law. It is the only the general command and not the particular command
that has the status of positive law.

Command (genus): Law and Rules (general command), occasional or Particular


command.

Where a command obliges generally to act or forbearance of a class, a command is


a law or rule. But where it obliges to a specific act or forbearance or to acts or
forbearance which it determines specifically or individually, a command is
occasional or particular for e.g if you command your servant to rise at a given hour
on a given morning, or to rise at that hour during the next week or month the
command is occasional or particular. It is because the act or acts enjoined or
forbidden are specifically determined or assigned.
But if you command him simply to rise at that hour till further orders, it may be
said with propriety, that you lay down a rule for the guidance of your servant's
conduct. For , no specific act is assigned by the command but the command
obliges him generally to acts of a determined class.
Thus the following commands may be said to be the law:
a. Commands by which acts or forbearance of a class are enjoined generally.
b. Commands that lay down a rule for the guidance of future conduct.
c. Commands that regulate the behaviour in continuum
According to Austin it is very difficult to draw a distinct boundary between the
laws and the occasional commands.
Sometimes the command is issued by the parliament or the sovereign legislature to
deal with a particular situation and enjoins an act or forbearance specifically and is
operational only for the transient period. However, since it is issued by the
sovereign legislature and with the forms of legislation it would be called the law.
In other words, where proper procedure of making the law is followed even a
specific or particular command may assume the character of law. e.g promulgation
of ordinance by the President of India under art 123 of the constitution. Though it
is issued to meet an emergency situation and is transient by nature, it still has the
status of law.
Judges do not make laws: Austin says that judicial commands are commonly
occasional or particular, although the commands which they are calculated to
enforce are commonly laws or rules.
Judges role is not to make law but merely to apply the already existing law made
by the legislature to the specific case before it.
In his analysis, Austin found that there are some objects improperly termed as laws
since they are not command. They may yet qualify to be included within the
province of jurisprudence as following exceptional instance. They are
A. Declaratory law: they explain the positive laws and declare what the duties of
the subjects are, e.g. judicial interpretation of the established laws.
B. Laws to repeal laws and to release from existing duties. They release from the
duties imposed by existing laws and are named permissive laws or permissions.
But such laws are often imperative. For, the parties released from duties are
restored to liberties or rights and duties answering those rights are thus created.
C. Imperfect laws or laws of imperfect obligation- An imperfect law lacks
sanction and therefore is not binding. Such is a kind of law which speaks of desire
of political superior, but which its maker ( by oversight or design ) has not
provided with sanctions.
A law declaring that certain acts are crime but annexing no punishment to the
commission of acts of the class is the simplest and most obvious example.

D. Laws which merely creates rights and not duties. It is doubtful whether a
law/command can create a right in someone without creating a corresponding duty
in some other.

E. Rules based on customs and traditions: Austin considered the customs as


rules of positive morality since customary law arise from the consent of the
governed and not from the position or establishment of political superior.

The Command Theory of Law Austin argues that laws are a species of command:
Laws are the (general) commands of the sovereign to his subjects.

–Commands are expressed wishes that something be done, with a known ―evil‖ or
sanction to be imposed if the wish is not complied with.

- the sovereign is a person, or a determinate body of persons, who the bulk of the
population is in the habit of obeying, but who is not in the habit of obeying anyone
else,

–An independent political society is the largest group of persons (subjects)


governed by a sovereign.

The proper subject of jurisprudence, according to Austin, is positive (or man--


made) law; but he finds his theoretical framework useful for understanding other
phenomena sometimes described as laws: Divine Law (commands of God, which
are backed by threats of punishment), ―positive morality‖ – the moral norms
accepted by a particular society, laws of honor or etiquette, specific orders.

- Divine law is not positive law because it does not have a human source.

- Positive morality is not law because it is not issued by a determinate superior.

- Laws also differ from specific orders in their generality: they aim to preclude a
class of actions, rather than an individual action, and generally govern a class of
people, rather than a single person.
Austin defined positive law as comprising of commands of a political sovereign
backed by
sanctions on the ones who disobey the commands.
There are primarily three key constituents of this concept of law:
1. Political sovereign
2. Command
3. Sanction
Austin noted that a society which does not have a political sovereign does not have
law in the strict sense of positive law. Political sovereign was regarded as a
necessary feature of a political society which considers or claims itself to be
independent. Austin was of the notion that where there is no sovereign, there is no
independent political society where as the vice versa is also true.
As pointed out by Austin, positive law is the result of a sovereign‘s command. A
command is an imperative that creates a duty by the presence of a sanction which
would follow if there is an incidence of non-compliance. Command is something
which is of such importance that it can never be distinguished from duty and
sanction and they can all be considered to be varied aspects of a single event.
Where ever there exists a duty, there will also be the presence of a command;
while where there is a command, there also is a duty. The duty arises from the
existence of punishment or sanction of sorts which follow in case of
noncompliance or when there is a breach of duty.

LAW EMANATES FROM SOVEREIGN


Austin‘s most important contribution to legal theory according to Friedmann was
his substitution of the command of sovereign for any ideal justice in the definition
of law. The first jurist to make jurisprudence as a ‗science‘ was John Austin who is
often described as Father of jurisprudence. Sovereign defined and analyzed:- While
defining a sovereign Austin said, ―if a determinate human superior, not in a habit
of obedience to alike superior, receives habitual obedience from the bulk of a given
society, that determinate superior is sovereign in that society and the society
(including the superior) is a society political and independent. According to Austin,
the superior may be either an individual or a body or aggregate of individuals.
Thus, English sovereign for him is merely the ‗person‘ who has the last word in a
particular connection.
His conception of sovereignty asserts that in every human society where there is
law, there is to be found latent beneath the variety of political forms, in a
democracy as well as in an absolute monarchy, a relationship between subjects
rendering habitual obedience and a sovereign who renders habitual obedience to
none. Involved in this are two main points of special importance viz., the idea of
obedience and due position occupied by the sovereign above the law. The
expression obedience often suggests deference to authority and not merely
compliance with orders backed by threats. The idea of obedience in-fact fails in
two different though related ways, to account for the continuity to be observed in
every normal legal system when one legislator succeeds another. An illustration of
this kind is the change in law of incest made in Rome by the then emperor
Claudius for his own private purposes. Desiring to marry Agrippina, the daughter
of his brother, he procured a change in the law which permitted a marriage
between a niece and her parental uncle, leaving the law unaltered as to other
marriages between uncles and nieces or aunts and nephews, so that these remained
incestuous. secondly, habitual obedience to the old law giver cannot by itself
render probable, or find any presumption, that the new legislator‘s orders will be
obeyed. If there is to be this right and this presumption at the movement of
succession during the reign of the earlier legislators, there must have been the
acceptance of the rule under which the new legislator is entitled to succeed.
The dictum, therefore, that English sovereign is merely the person who has the last
word is not in the least true to any federation, where the legislatures are bound by
the constitution and in most cases a court has the power to decide whether a
particular statute is constitutional or not.

LAW AS A COMMAND
Austin defines command as ―if you express or intimate a wish that I shall do or
forbear from some act and if you will visit me with an evil, in case I comply with
your wish-it is a command‖. A command is different from other significations of
desire, not by the style in which the desire is signified but by the power and the
purpose of the partly commanding to inflict an evil or pain in case the desire be
disregarded. Thus, a command is significance of desire. But a command is
distinguished from other significations of desire by this peculiarity that the party to
whom it is directed is liable to evil from other, in case he complies not with the
desire.
According to Austin law, therefore, signifies a command which obliges a person or
persons to a course of conduct. The person who receives the command must realise
that there is a possibility of incurring some evil in the event of disobedience. For
Austin, every command does not create a law. A law determines acts of a class, a
particular command determines merely a specific act. Austin, therefore suggests
that a statute issued by Parliament that corn then shipped and import should not be
a law, since it relates merely to a specific case but he hares that in popular speech it
would be called a law. Generality is a normal mark of law because of the
impossibility and undesirability of issuing particular commands for each specific
act. Now, if we regard a law as a command, then every act done must either be
permitted or forbidden. Further, if we consider law as a command or an order, it
must also be seen in the first sight to be orders given to the judges to do or abstain
from doing anything and there should, of course, be no choice why the law should
not by special rules prohibit a judge under penalty from exceeding his jurisdiction
or trying a case in which he has some financial interest. These rules imposing such
legal duties would be conducive to those conferring judicial powers on him and
defining his jurisdiction.
SANCTION
Austin said, ―Sanction operate upon the desires and that men are obliged to do or
forbear through the desires. For, he is necessarily averse from every evil
whatsoever. That every sanction operates upon the will of the obliged is not true. If
the duty be positive, and if he fulfills the duty out of regard to the sanction, it may
be said with propriety that the sanction operates upon his will. For his desire of
avoiding the evil which impends from the law, makes him do and therefore, will
the act which is the object of the command and duty. But if the duty be negative
and if he fulfills the duty out of regard to the sanction, it can scarcely be said with
propriety that the sanction operates upon his will. His desire of avoiding the evil
which impends from the law makes him forbear from the act which the law
prohibits. But though he intends the forbearance, he does not will the act forborne,
or he remains in a state of inaction which equally excludes it. In the former case he
does not will the forbearance. In the latter case he wills nothing.‖
Only General Commands are law:
However, all the commands are not law, it is only the general commands, which
obliges to a course of conduct, is law.
Exceptions:
The general commands are the proper subject of study of jurisprudence. But
according to Austin, there are three kinds of laws which though not commands, are
still within the province of jurisprudence. They are.-

Declaratory or Explanatory laws: Austin does not regard them as commands,


because they are passed only to explain laws already in force

Laws to repeal laws: These too are not commands but are rather the revocation of
a command.

Laws of imperfect obligations: These laws have no sanction attached to them.

Relevance in modern Indian politics & legal society/Criticism

In this part of the paper an attempt has been made to analyze the existence of

Austin's sovereign in modern India.

1. .Austin's notion that all laws come from the sovereign may be true theoretically,

and laws in our country (i.e. statue made laws at least) are a result of the act of the

politically superior that is the legislators but the same is not true practically as they

are not a reflection of the will of the superior in the real sense. Though many laws

come directly from the parliament , but they merely reflect the desire of these

politicians to maintain support of the major organized groups in the country and to

meet their interests satisfactorily. Under the conditions of the day the huge

combination of labor capital , capital , with their expert lobby sit and wealthy

treasuries any group is able to compel recognition and secure desired legislation.

Only the fact that these groups are competing amongst themselves prevents the
government from becoming a helpless tools in their hands. Even then the grinding

impact of competing pressures upon the government requires political astuteness of

a high order to keep them satisfied and prevent the withdrawal of support in the

next elections. Thus we can say that Austins emphasis that sovereign is the main

stream of law is not just.. Law emanating only from the sovereign may be

fit for a totalitarian regime (which is soon becoming a rare phenomena in present

world order) where the government can use its monopoly of law making and

executive powers for the re shaping of laws in disregard of the democratic

processes, but in a democratic country like India the same is not possible. The

interplay between the public opinion and state action has become very complex

these days whether we are concerned with the abolition of dowry ,the creation of

legal remedies against administrative action or the introduction of a new ground of

divorce there is always some interrelation between the state machinery that

produces these changes and social opinion of the community in which they are

intended to operate. public opinion on vital issue is expressed through the elected

representatives in the house, and also through public discussion in press , radio,

public lectures .it can thus be concluded that legislative practices in our country

provide for opportunities to the public to participate in the legislative activities of

those to whom these powers are delegated.


2. Austin postulates a political superior in a political society who is habitually

obeyed by the majority of the population. This means that sovereign is the highest

authority , the strongest authority in a political system. According to Austin

sovereign is the person who has the last word in a particular connection. But the

issue is that how can one determine the 'highest authority' in

a democratic country like India , to identify the strongest power would involve an

investigation of a lot of legal as well as well extra legal forces which determine

how a state shall operate. Who is the highest authority, is it the masses who chose

the government, is the legislators who finally make laws, is it the judiciary that has

the power to strike down laws made by the parliament, is it the executive as laws

that are enforced are selected by administrators today, what they consider worthy

of implementing is duly enforced other laws are followed more in breach than in

obedience. is it the constitution according to which all others are expected to act or

is it again the masse by whom the constitution has been formed ? Who do we call

supreme. Besides these forces there are other socio - economic forces that have the

power to exert a lot of pressure to finally determine what laws are formulated and

most often have the final say.

3. Austin 's theory that law emanate only from the sovereign authority in India as

much as it would fail in other common law countries. There are various other very
important sources of law which cannot be ignored at all. His theory would fit only

one portion of law that is the law made by the legislative body. But the word law is

of wilder amplitude and includes not only laws but bye-laws, notifications,

customs which are not made by the state. Another important category that Austin

does not include in his definition of law is Judge made laws , in this era of judicial

activism where judiciary does not only interpret law but also makes law this

category cannot be ignored. principles of justice, equity, good conscience are

important principles that are always kept in mind while implementing any law

none of them.

4. Similarly Austin's concept of unlimited and indivisible sovereignty is quite

inappropriate in the Indian set up or any democracy. The sovereign does not have

the power to command anything that it desires. It is as much bound by rules and

regulation embodied in the constitution and other laws as any common man.

Legislature is bound by the constitution and in almost all cases court has the power

to decide whether an act done by the government is constitutional and hence valid

otherwise it can be struck down.

Thus we can say that the notion of sovereignty in India at present certainly not

what Austin would define as sovereignty, the concept of sovereignty is under

restraint which is very justified as the concept of an unlimited illimitable and


indivisible sovereignty is a superfluity that debases the very cannon of Indian

Jurisprudence.

Hans Kelsen

(Q) (a) What is Norm according to Hans Kelsen ? What is meant by ―Validity‖ of
a norm and what are the spheres of validity of a norm? [2019(7)(a)]

(b) What are the characteristics of a Legal Order according to Hans Kelsen ?
[2019(7)(b)]

(Q) Describe Kelson‟s concept of Grundnorm. How will you locate Grundnorm
in the Indian Legal System? Does it apply to a revolutionary state of affairs?
[2018(6)]

(Q) Explain how according to Hans Kelsen a legal order is a social, coercive and a
normative order. Enumerate the different kinds of norms that can be identified in a
legal order. Where will you place the Constitution of India according to Kelson
in the Indian Legal System? [2016(8)]

(Q) “The pure theory of law undertakes to delimit the cognition of law against the
disciplines of social science, not because it ignores or denies the connection, but
because it wishes to avoid the uncritical mixture of methodologically different
disciplines which obscures the essence of the science of law and obliterates the
limits imposed upon it by the nature of its subject-matter.” Discuss pure theory of
law in the context of above statement. [2017(6)]

(Q) Grund Norm (Hans Kelsen)[2015(6)(c)]

Jeremy Bentham heralded a new era in the history of legal thought. Although,
Austin is considered to be the father of positivist thought yet, Bentham whose
many works have lately become known appears to be the founder of this approach.
Positivist approach insists on a strict separation of positive law from ethics and
social policy and identifies justice with legality. Positivism indeed concentrates on
law as it is and not on law as it ought to be. This separation eliminates all
considerations of ideologies and value judgments.

While Bentham and Austin are being considered to be the forerunners of positivist
school in England, the school later received encouragement in the United States as
well as in the European continent from a number of jurists. Hans Kelsen, who has
developed the ―Pure Theory of Law‖ with great analytical refinement, belongs to
this school. He was one among the jurists who has the credit of reviving the
original analytical legal thought in 20th century through his pure theory of law
which is considered to be Kelsen‘s unique contribution to legal theory.

Kelsen rejected Austin‘s definition of law as a command because it introduces


subjective considerations whereas he wanted legal theory to be objective. He tried
to modify Austin‘s theory of law by propounding his own theory. For Austin,
sovereign being the law maker was considered superior to law. But Kelsen denies
the existence of sovereign as a personal entity. When all derive their power and
validity ultimately from the Grundnorm there can be no supreme or superior
person as sovereign. He denies the existence of State as an entity distinct from law,
but they are in fact one and the same. Kelsen, however, observed that Grundnorm
need not be same in every legal order but there will always be a Grundnorm of
some kind.
Thus, it can be asserted that Kelsen‘s pure theory of law made a substantial
refinement of the theories propounded by his predecessors. Indeed, Kelsen
attempted to device a logically consistent theory which could be uniformly
acceptable to any legal system. Kelsen believed that a theory of law should be
uniform, that is, it should be applicable at all times, and in all places. In this
endeavour, the present study made an attempt to testify the relevance of Kelsen‘s
theory in context of Indian legal system.

Hans Kelsen and His Contribution to Legal Theory

Hans Kelsen was an Austrian jurist and legal philosopher, with major area of
interests ―Philosophy of Law‖. The German Reine Rechtslehre (Pure Theory of
Law) is a book by Hans Kelsen, first published in 1934. Late in his career while at
the University of California (although officially retired in 1952), Kelsen rewrote
his short book of 1934 which resulted in a greatly expanded second edition
(effectively a new book) published in 1960. The second edition of the book
appeared in English translation in 1967 as Pure Theory of Law and the first edition
in English translation in 1992 as Introduction to the Problems of Legal Theory.
The theory proposed in this book has probably been the most influential theory of
law produced during the 20th century. Kelsen throughout his active career was also
significant contributor to the theory of judicial review, the hierarchical and
dynamic theory of positive law, the science of law etc.

Kelsen‟s Pure Theory of Law- Defined

The distinction between propositions of science and propositions of law is the


starting point of Kelsen‘s reasoning. Kelsen described law as a ―normative
science‖ as distinguished from natural science. Law does not attempt to describe
what actually occurs but only prescribes certain rules. It articulates, ―If one breaks
the law, s(he) ought to be punished‖ and thereby, it is the ought proposition which
provides normative character to law. Thus according to Kelsen, law is a ―primary
norm which stipulates sanction‖. Norms are regulations setting forth how men
ought to behave, and positive law is thus a normative order regulating human
conduct in a specific way (Kelsen, 1941). These legal ought norms differ from
morality norms in the sense that the former are backed by physical compulsion
which the latter lack.
Kelsen‘s theory is known as the pure theory of law because according to him a
theory of law should be free from all extra-legal disciplines. Kelsen, thus, does not
admit the Austin‘s idea of command as it introduces a psychological element into a
theory of law and therefore, be rejected. Hence, a theory of law, according to
Kelsen, should be pure.
The Grundnorm- The Starting Point- A Fiction
Grundnorm is a German word meaning ―fundamental norm‖. Kelsen‘s pure theory
of law is based on pyramidical structure of hierarchy of norms with Grundnorm at
the apex. Kelsen defined Grundnorm as ―the postulated ultimate rule according to
which the norms of this order are established and annulled, receive or lose their
validity‖. The Grundnorm is thus at the top of the hierarchy of norms which inspire
the prescriptive elements of the positive laws of a legal system (Hopton, 1978).
The Grundnorm is the starting point in a legal system and from this base; a legal
system broadens down in gradation becoming more and more detailed and specific
as it progresses. This is a dynamic process (Patterson, 1952). He named Austin‘s
theory static because it considers law as a system of rules complete and ready for
application without paying any attention to the process of their creation. But the
study of dynamics of law is also necessary because law regulates its own creation
and Kelsen‘s theory includes it. Thus, according to Kelsen in every legal order
there will always be a Grundnorm of some kind.

Kelsen, however, added that any discussion about the nature and origin of the
Grundnorm is not within the province of pure theory of law. The task of legal
theory is to clarify the relations between the fundamental and all lower norms, but
not to say whether this fundamental norm itself is good or bad. He considers
Grundnorm as a fiction rather than a hypothesis.
Pyramid of Norms- Legal Order of a State- A Dynamic Process
In Kelsen‘s view State is a ―synonym for the legal order which is nothing but a
pyramid of norms‖. Kelsen treats State as a unity of legal order. The legal order as
conceived by him receives its unity from the fact that all manifold norms of which
the legal order is composed can be traced back to a final source. The process of one
norm deriving its power from the norm immediately superior to it, until it reaches
the Grundnorm has been termed by him as ―concretisation‖ of the legal system
(Paranjape, 2013). Thus the system of norms proceeds from downward to upward
and finally it closes at the Grundnorm at the top.

Every legal act relates to a norm which gives legal validity to it. The validity of a
norm in turn, however, is not to be derived from any fact outside the law, but from
some other norm standing behind it and imparting validity to it. The validity of a
norm is ascertained with reference to its authorising norm, which confers a power
to create it, and may also specify conditions for its exercise. A particular norm,
therefore, is authorised if it can be includedunder a more general norm. In any
legal order, a hierarchy of norms is traceable back to some initial fundamental
norm on which the validity of all others ultimately rests. Thus, the entire hierarchy
of norm-making organs and the process of concretisation of norms which in
Kelsen‘s view termed as the legal order of a State take the shape of a pyramid.

Assumptions
Kelsen‘s theory, thus, is founded on certain basic assumptions namely,

nce, not volition for it is knowledge of what the law is, not of
what the law ought to be;

concerned with the effectiveness of


legal norms;

specific way, etc.

Application of the Theory In India

Kelsen used the word Grundnorm to denote the basic norm, order, or rule that
forms an underlying basis for a legal system, and it is regarded as the source of the
validity of positive law of that very legal system. Thus, the question is whether
Constitution can be regarded as the Grundnorm? The answer is simply no. The
Grundnorm is the reason for the validity of the Constitution and merely marks the
fact that a Constitution is accepted by the legal system. It is not the Constitution
itself. It is for this reason why Kelsen said that Grundnorm is not the Constitution,
it is simply the pre-supposition made in the theory for the interests of legal science
that this ―Constitution ought to be obeyed‖ (Hopton, 1978). The theory is based on
a need to find a point of origin for all laws (similar to the concept of first
principle). Hence, Grundnorm only imparts validity to the Constitution, and all
other norms derived from it; but it does not dictate its content. The Grundnorm can
only be changed by political revolution.

Analytical positivism which dominated the English legal system for more than a
century was mainly founded on three basic assumptions namely, First, sovereign as
the law creating authority; Secondly, emphasis on law as it is and Thirdly,
insistence on sanction, that is, the coercive force behind enforcement of laws. The
analytical positivism of the English legal system when examined in the light of the
ancient Indian jurisprudence would bring to forefront certain contradiction. In the
Austinian positivism, sovereign being the law maker was considered superior to
law. On the contrary, the ancient India legal order was based on sovereignty of
Dharma and not that of a monarch, which was given the highest place by which
the subjects as well as the ruler were uniformly bound.

India has an unbeatable tradition of the principle of Dharma governing all


activities of all persons in community life individually and collectively. The
concept of Dharma has been well understood and accepted as a code of conduct to
be observed by all. The present study thus hereby records the initial finding that the
application of Kelsenite idea of Grundnorm in Indian context can be traced back to
the legal philosophy of ancient time in so far as the Indian jurists also subordinated
the authority of the King to Dharma. The scriptures enjoined upon the King, a duty
to rule and administer justice in accordance with Dharma. Moreover, the same was
accepted by all whole heartedly for the very survival of the society.

However, the advent of British rule in India brought about radical changes in the
then existing legal system which sought to embed British imperialism in the land.
Macaulay, the Law Member of the Governor-General-in-Council rejected ancient
Indian legal and political institutions. He then gradually introduced the notions of
British juristic concepts through equity, justice and good conscience and brought
about codification of laws. Theses codified British laws were similar to Austinian
concept of positive law having the element of certainty, definiteness, effective
enforcement and sanction, and as such, positivism found its place in the Indian
legal system during the British colonial rule.

The struggle for independence was over by August 1947. But the attainment of
independence was not an end in itself. It was only the beginning of struggle, that is,
the struggle to live as an independent nation and also to establish democracy based
on the ideas of Justice, Liberty,
Equality and Fraternity. The need of the Constitution forming the basic law of the
land for the realisation of these ideas was paramount. Therefore, one of the first
tasks undertaken by independent India was framing the Constitution which came
into force on 26th January, 1950.

The Preamble to the Constitution begins with the words ―WE, THE PEOPLE OF
INDIA, having solemnly resolved ….‖, thus, clearly indicates the source of all
authority of the Constitution. Further, the Preamble ends with the words ―…. IN
OUR CONSTITUENT ASSEMBLY this twenty-sixth day of November 1949, do
HEREBY ADOPT, ENACT, AND GIVE TO OURSELVES THIS
CONSTITUTION‖, which further indicates that it is the people of India who
adopted, enacted and given to themselves this Constitution. Thus, the Preamble
declares that the Constitution has been ―given by the people to themselves‖.
Moreover, the people of India themselves after ―having solemnly resolved‖,
declared to be bound by this Constitution without any exception.

Hence, Grundnorm test is satisfied for the pre-supposition demanded by the theory
found its place in Indian legal system even during post-independence era.

Further, as early as 1951, the argument that there is a distinction between


Constitutional law and ordinary law was accepted in India. Patanjali Sastri, J., in
Sankari Prasad‟s Case (1951) did not elaborate the point. But he emphasised the
distinction when he observed:
“There is a clear demarcation between ordinary law which is made in exercise of
legislative power, and constitutional law which is made in exercise of constituent
power”.

Constitution in India is regarded as basic law of the land due to its social
acceptance or recognition and other laws assume validity because of their
conformity with the Constitution. The institutions established under the
Constitution namely, the Legislature, the Executive and the Judiciary (being norm-
creating agencies in the Kelsenite sense) are subordinate to and have to act in
conformity with provisions of the Constitution. For example, the legislative power
of Parliament and the State Legislature has been subjected to certain limitations.
The power derived from Articles 245 and 246 to make laws has to be exercised
keeping in view the limitations outlined under Article 13 of the Constitution.

Hence, unity of the legal system is established for all laws enacted trace validity
from a single source, that is, the source of validity of all laws here is the
Constitution.

Although, the fact that the Constitution can be amended shows that it is possible to
derogate from the authority of the Constitution itself. If a Constitutional provision
is amended substantially, it can no longer confer validity upon the laws under it.
Similar would be the effect, if, a provision of a Constitution is repealed. Yet, the
reason for incorporating the provisions for amendment in the Constitution was that
if no provisions for amendment were provided, there would be constant danger of
revolution. Again, if the methods of amendment were too easy, there would be the
danger of too hastily action all the time. Thus, a proper balance was kept between
the danger of having non amendable Constitution and a Constitution which is too
easily amendable.
In this backdrop, the Hon‘ble Supreme Court in its landmark judgment given in
Kesavananda Bharati‟s Case (1973) made it clear that under Article 368
Parliament cannot amend the ―Basic Structure” of the Constitution. The question
involved was as to what was the extent of the amending power conferred by
Article 368 of the Constitution? A Special Bench of 13 Judges was constituted to
hear the case. A proposition enunciated, by a majority consisting of Sikri, C.J., and
Shelat, Hegde, Grover, Jaganmohan Reddy, Khanna, and Mukherjee, JJ., is that the
power to amend does not include the power to alter the basic structure or
framework of the Constitution to the extent of changing its identity. It is this
proposition that will be applied in testing the validity of a constitutional
amendment in the future. This ratio is common in the opinions of seven judges
may be substantiated by the pure mechanical process of presenting excerpts from
the opinions. Sikri C.J., however, in his conclusion observed:

“The expression „amendment of this Constitution‟ does not enable the Parliament
… to completely change the fundamental features of the Constitution so as to
destroy its identity”. Though, the majority decision held that the basic structure of
the Constitution cannot be destroyed by means of amendment, however, what
constitutes the basic structure is not clearly made out. However, Sikri C.J., sets
down the following as forming what he calls the
―Basic Structure‖ of the Constitution.
“(1) Supremacy of the Constitution;
(2) Republican and Democratic form of government;
(3) Secular character of the Constitution;
(4) Separation of Powers between the Legislature, the Executive and the Judiciary;
(5) Federal character of the Constitution”.

While the Judges enumerated certain essentials of the basic structure of the
Constitution, they also made it clear that they were only illustrative and not
exhaustive. They will be determined on the basis of the facts in each case.
In Indira Nehru Gandhi v. Raj Narain (1975), the Hon‘ble Supreme Court
applied the doctrine of ―Basis Structure”, and struck down clause (4) of Article
329A inserted by the Constitution (Thirty-ninth Amendment) Act, 1975 on the
ground that it was beyond the amending power of the Parliament as it destroyed
the basic structure of the Constitution. The amendment was made to validate with
retrospective effect the election of the then Prime Minister which was set aside by
the Allahabad High Court. Khanna J., struck down the clause on the ground that it
violated free and fair election which was an essential postulate of democracy
which in turn was a part of the basic structure of the Constitution. Chandrachud J.,
struck down clauses (4) and (5) as unconstitutional on the ground that they were
outright negation of the right to equality conferred by Article 14, a right which is a
basic postulate of our Constitution. He held that these provisions were calculated to
damage or destroy the Rule
of Law. The Hon‘ble Court has thus added the following to the list of basic
structure laid down earlier.
“(1) Rule of Law;
(2) Judicial Review;
(3) Democracy;
(4) Jurisdiction of the Court under Article 32”.

Further, in Minerva Mills‟s Case (1980), the Hon‘ble Court has held that the
following are the ―Basic Features‖ of the Constitution.
“(1) Limited power of Parliament to amend the Constitution;
(2) Harmony and balance between Fundamental Rights and Directive Principles;

3) Fundamental Rights in certain cases”.

Moreover, In M. Nagraj‟s case (2007), the Hon‘ble Supreme Court has explained
the ―Basic Structure Theory‖ again in detail. The Court has held that basic
structure theory develops systematic principles underlying and connecting
provisions of the Constitution. These principles provide coherence and durability
to the Constitution. The basic structure theory has its roots in the German
Constitution. The theory is based on the concept of Constitution identity, and main
object behind the theory is continuity.
Thus, the constitutional amendments made under Article 368 can still be
challenged on the ground that they are destructive of the ―Basic Structure‖ of the
Constitution. Hence, supremacy of the Constitution prevails at all times, and the
ideals upon which the Constitution is based cannot ever be destroyed.

H.L.A Hart

(Q) What is the difference between primary and secondary rules according to
H.L.A Hart? [2018(5) (a)]

(Q) Discuss H.L.A Hart primary and secondary rules. [2017(5)(a)]

(Q) Legal System is a union of Primary rules and Secondary Rules. Comment
on the statement. [2019(5)(b)]

(Q) What are the three defects in a simple social structure of primary rules of
obligation? How does H.L.A Hart try to remove them in a legal system?
[2016(6)(a)]

In his essay, Laws as a Union of Primary and Secondary Rules, Hart criticizes
Austin‘s theory of laws as commands and argues for a new framework which
describes laws as rules. Hart, like Austin, is a positivist and wants to separate the
descriptive question of what law is from the prescriptive question of what law
should be. But, he does believe that there is a normative aspect to the law, which is
reflected in the obligation we feel to follow it. In his analysis Hart makes a
distinction between two types of rules (primary and secondary). The separation
of rules into these two different categories allows him to establish a method to
determine the validity of a law, which is what determines whether it creates an
obligation among citizens in a society or not. For the most part, Hart is able to
create a very neat and consistent model to describe the legal system, but one
inconsistency that I saw, and which I will address, is with the way that Hart
incorporates judicial decisions into his system of rules.

First of all, in Hart‟s analysis, laws are described as rules in order to be


distinguished from Austin‘s theory of laws as commands. A brief observation of
existing laws will present us a wide range of laws that do not neatly present
themselves in command form. For example, power-conferring laws, which
describe or direct agreements between people such as contracts or marriages,
appear to be granting people rights or describing the way public officials should
react to certain circumstances rather than commanding people to behave in a
certain way. Furthermore, the command theory leaves out an explanation for how,
in modern representative systems, the rule-makers who issue the commands find
themselves bound by them as well (Hart, 68-69). For these reasons, Hart believes
that a more appropriate metaphor for thinking about laws is that of rules in a
sporting competition. Rules can not only direct the players to perform or refrain
from performing certain actions, but they also give directions to the umpire or
score keeper. Furthermore, players feel themselves bound by the rules. The rules
themselves provide a reason to act, not just the fear of punishment as in the
command theory. Hart calls this point of view, where the existence of the rule
provides an obligation for action, the internal perspective to the law. (Hart, 73)

Hart divides rules into two categories, primary rules and secondary rules.
According to Hart‘s definitions, primary rules either forbid or require certain
actions and can generate duties or obligations. For a citizen with an internal
perspective to the law, the existence of a primary rule will create an obligation for
him or her to behave a certain way (Hart, 74). When we think of something being
against the law, or required by the law, we are generally in the realm of primary
rules. A primary rule can be the law against walking out of the Apple Store with
an IPod without paying or the law requiring you to stop at a red light. In the ―rules
of the game‖ metaphor, an example of a primary rule would be that in football, it is
illegal to restrain a player who is not in possession of the ball. Secondary rules on
the other hand, set up the procedures through which primary rules can be
introduced, modified, or enforced.

Primary and Secondary Rules


According to Hart, a legal system may be best considered a "union" of primary
rules of obligation or duty and secondary rules of recognition, change, and
adjudication. Hart characterizes the distinction between primary and secondary
rules in the following way:
Rules of the first type impose duties; rules of the second type confer powers,
public or private. Rules of the first type concern actions involving physical
movement or changes; rules of the second type provide for operations which lead
not merely to physical movement or change, but to the creation or variation of
duties or
obligations.

Secondary rules can be thought of as rules about the rules (Hart, 76).
Continuing with our football metaphor, an example of a secondary rule would be
that a coach is permitted to challenge a call by the referee, but must accept the
final decision of the ref following the viewing of the instant replay. When
analyzing the necessity for secondary rules, Hart imagines a simple society, with
only primary rules, but concludes that such a society would face a number of
challenges:

(i) because there would be no systematic method of rule creation, there would be
uncertainty about what the rules actually are;

(ii) the system would be very static, since any changes in the rules would have to
occur organically;

(iii)finally, without a defined adjudication method, inefficiencies would arise


from disputes over whether a rule was actually broken (Hart, 75).

These three problems can be remedied with the introduction of three types of
secondary rules, in order:

(i)rules of recognition,

(ii) rules of change, and

(iii)rules of adjudication (Hart, 76-77).

Of these three secondary rules, Hart believes that rule of recognition is the most
important. The rule of recognition tells us how to identify a law. In modern
systems with multiple sources of law such as a written constitution, legislative
enactments, and judicial precedents, rules of recognition can be quite complex and
require a hierarchy where some types rules overrule others (Hart, 76). But, by far
the most important function of the rule of recognition is that it allows us to
determine the validity of a rule. Validity is what allows us to determine which rules
should be considered laws, and therefore, which rules should create obligations for
citizens with an internal perspective to the law. According to Hart, validity is not
determined by whether a rule is obeyed, its morality, or its efficiency, but by
whether it fits the criteria set forth by the rule of recognition (Hart, 80). In more
complex legal systems we may have to trace the origin of a rule back a few steps to
the ―supreme rule‖ of that system (Hart, 81-82).

In the context of Hart‘s definition of validity (whether the law is derived from a
source and in a manner approved by other rules) it simply does not make sense to
ask about the validity of the rule of recognition in its supreme form. Once we have
reached the rule of recognition, there is no higher level of rules to provide us with
the criteria with which to judge its validity. Other writers have made the claim that
the rule of recognition can simply be ―assumed to be valid‖. Hart, however,
believes that this description is inaccurate and prefers the explanation that the
rule of recognition is ―presumed to exist‖. The word ―validity‖ can only be used
to answer questions about the status of a rule within a certain system of rules.
Since the rule of recognition is the standard which we use in order to judge the
validity of other rules, it cannot itself have a validity test. Hart states that asking
about the validity of the rule of recognition can be equated to asking whether the
standard meter bar in Paris is in fact a meter. In the same way that the Parisian
standard bar identifies the accuracy of instruments used to measure a meter, the
rule of recognition identifies the validity of a rule. You can use the Meter Bar to
check the accuracy of other instruments, but the bar itself cannot be accurate or
inaccurate because accuracy is only defined by how well an instrument
approximates the standard (Hart, 83).

In this context, the rule of recognition cannot be described in terms of validity,


but only in terms of existence. The rule or recognition is presumed to exist if it is
actually accepted and employed in general practice. In this respect, the existence of
the rule of recognition is an external statement of fact. While laws can be valid
even if de facto no one abides by them, the rule of recognition can only exist if
courts, legislators, officials, and citizens act in a consistent way that corresponds
with the presumed existence and acceptance of such a rule (Hart, 83-84).

One area where I think that Hart‟s theory falls short is the way he incorporates
laws that created on the basis of judicial decisions into his framework of rules.
According to Hart, once a rule is established according to the rule of recognition,
whether by the legislature or by judicial precedent, it becomes part of the legal
pedigree and there is little further uncertainty about its meaning or validity. This
analysis, and even the choice of the word ―rules‖ implies that the laws are very
explicit and do not contain much room for interpretation. However, in many
real life examples we find that the laws are rather vague or flexible, or that they
turn on abstract concepts, like in contract law, where judges often make decisions
based on whether the parties acted in good faith.

Let‘s take the discussion further with the contract law example. One of the parties
has intentionally hidden some documents from its contract partner. A judge will
consider this evidence of a lack of good faith and it may cost the first party the
case, even if the law never explicitly listed ―hiding documents‖ among the
activities that are considered illegal. Hart would defend his theory by saying that in
this case the judge went outside the boundaries of the existing law to create a
new law, which never existed before that moment, and which states that it is illegal
to hide documents from your contract partner. But this seems inconsistent with the
way that judges view their own position in the legal system. No judge sees his or
her role on the bench as a maker of new laws, but as an interpreter of the
existing laws. In the metaphor that Hart gives of the rules of a game, the judge
would be like a referee. The referee can identify when a rule was broken, but it
shouldn‘t be within her power to create new rules. A judge writing a decision in
the contract example wouldn‘t say that there was no rule against concealing
documents, but now that the question has come up, she thinks it would be good to
have one. She would say that the rule against concealing documents was there all
along; it wasn‘t explicitly written, but it was in the implied subtext of the general
understanding about the rules of contract law, and that the first party should have
known it was doing something wrong even if it wasn‘t printed in the text.

If Hart would believe that any judicial decision in which there is uncertainty or a
lack of specificity within the law in question constitutes the creation of new laws
by a judge, then that amounts to an incredibly large quantity of retroactive laws,
which is problematic to the consistency Hart wishes to maintain in his theory of
rules. The reason that Hart gave for the necessity of a rule of recognition was to
solve the problem of uncertainty about the rules by making it easy for people to
clearly identify what the laws are. But if new laws can be created every time a
judge makes a difficult decision, it makes identifying the rules just as difficult as if
there were no rule of recognition at all.
This means that judges are cannot be free and unconstrained to make new law in
any way they wish every time a case comes up where existing laws were undefined
or when the factors that influence a decision are not explicitly part of a distinct set
of secondary rules. Intuitively, a judge who decides that hiding documents from a
contract partner is wrong, even if this is not explicitly stated in the law, has made a
fair or just choice (it would almost seem unfair if the decision had gone the other
way), but the types of considerations that would guide the judge in making this
decision do not seem consistent with what would fall under the rule of
recognition as described by Hart. Dworkin makes a similar objection when he
discusses the way in which judges make an appeal to principles when deciding
cases. Principles are not hard and fast rules since, standing alone, they do not
constrain behavior in the legal sense. They do however they provide guidelines for
how the law is to be interpreted and applied. (Dworkin, 74- 75)

In conclusion, Hart‟s analysis of primary and secondary rules provides a very


useful framework for understanding the sources of law and how we can
distinguish valid laws from invalid ones without entering into subjective moral
territory. Hart‘s system creates a way to reconcile some of the inconsistences in
Austin‘s theory, while also incorporating some of the more normative nuances of
the law without making any moral claims. Hart observes that people feel an
obligation to follow primary laws, even in cases where the likelihood of being
caught and punished is slim to none. Since Austin defines laws as demands issued
by a sovereign under threat of sanctions, this observation cannot be explained by
Austin‘s theory. Hart argues that this obligation does not come from the moral
content of the law, but from its validity, which is why we need secondary laws to
determine the validity of the primary laws. Because people who take the internal
perspective to the law presume the existence of the rule of recognition, they
accept to be bound by laws that are valid according to the criteria set forth in the
rule of recognition and in the secondary laws derived from this rule. I do feel,
however, that Hart‟s theory on judicial decisions fails to address the reality of
how judges see their role in the legal system, as interpreters or arbitrators of
the law rather than creators of new laws.

Answer to [2016(6)(a)] :
Primary and Secondary Rules According to Hart, a legal system may be best
considered a "union" of primary rules of obligation or duty and secondary rules of
recognition, change, and adjudication. Hart characterizes the distinction between
primary and secondary rules in the following way:

Rules of the first type impose duties; rules of the second type confer powers,
public or private. Rules of the first type concern actions involving physical
movement or changes; rules of the second type provide for operations which lead
not merely to physical movement or change, but to the creation or variation of
duties or obligations.

In Hart's view, a society might live only by primary rules of obligation, but
will suffer from three defects:

(i)there will be uncertainty as to what the rules are and their scope;

(ii)the rules will be static insofar as there is no means of deliberately eliminating


or introducing rules; and,

(iii) the social pressure by which rules are maintained will be inefficient because
there is no agency for finally settling disputes over violations of the primary rules."

These defects may be remedied by secondary rules that "specify the ways in
which the primary rules may be conclusively ascertained, introduced, eliminated,
varied, and the fact of their violation conclusively determined."

Thus, a secondary rule of recognition will remove the uncertainty of the regime
of primary rules by providing a rule for conclusive identification of the primary
rules of obligation."

Secondary rules of change will remedy the static character of primary rules by
empowering an individual or body to introduce new primary rules of conduct for
the group, or for some class within it, and to eliminate old rules."

The remedy for the inefficiency of the social pressure is secondary rules of
adjudication empowering individuals to determine authoritatively whether, on a
particular occasion, a primary rule has been broken.
Hart concludes that "The introduction of the remedy for each defect might, in
itself, be considered a step from the pre-legal into the legal world," and that
"certainly all three remedies together are enough to convert the regime of primary
rules into what is indisputably a legal system."

The "union" of primary and secondary rules may be regarded as the "essence" of
law," for a modern municipal legal system will combine both kinds of rules, and
the secondary rule of recognition will unite all the rules of the system in that all
genuine laws will satisfy the criteria of validity specified by the rule of recognition.

Karl Von Savingy

(Q) Critically analyse the idea of Volksgeist given by Karl Von Savingy or
critically analyse Roscoe Pound‘s Theory of Interests. [2019(8)]

(Q) Explain Savigny‟s theory of law. Do you consider that only a law which
reflects the spirit of the people can be considered ‗law‟ in the legal system?
[2018(5)(b)]

(Q) Discuss Savigny‟s theory of law. Why is the historical school considered
close to the people of a nation? [2017(7)]

(Q) Critically analyses Karl von Savigny‟s theory of Volksgeist. [2016(6)(b)]

Savigny, stands as the undisputed head of the influential historial school of


jurisprudence, which he pioneered in the first half of the 19th century, and as the
founder of the study of relations between social and legal developments). A trail-
blazing legal scientist, Savigny made many lasting contributions to jurisprudence
that greatly influenced all the social disciplines as well. Among these contributions
are included the revealing of continuity between present legal institutions and those
of the past, the laying of foundations for legal sociology, and the articulating of
methods for historical research.
CONCEPT OF VOLKSGEIST AS A SOURCE OF LAW
Now after getting an idea of the Historical school of Jurisprudence, a clear concept
has to be drawn about the main theory of that school as laid down by Savigny, i.e
the Volksgeist theory. The term Volksgeist also Volksseele, Nationalgeist or Geist
der Nation, Volkscharakter, and in English ―national character‖, it is a term
connoting the productive principles of a spiritual or psychic character operating in
different national entities and manifesting itself in various creations like language,
folklore, mores, and legal orders.In simple term Volksgeist means the general or
common consciousness or the popular spirit of the people. Savigny believed that
the law of the people and a manifestation of their spirit. The basis of the origin of
law is found in volksgeist, which means people‘s consciousness or will and
consists of traditions, habits, practise and belief of the people. The concept of the
Volksgeist is however, an interesting and important aspect of German legal
science. The concept of Volksgeist in German legal science states that law can
only be understood as a manifestation of the spirit and consciousness of the
German people. German legal science assumes that the law can be studied
scientifically as naturally occurring phenomena from which inherent legal
principles and relationships can be discovered.

According to Savigny, a law made without taking into consideration the past
historical culture and tradition of community is likely to create more confusion
rather than solving the problems because ‗law‘ is not an ‗artificial lifeless
mechanical device‘ the origin of law lies in the popular spirit of the people which
Savigny termed as Volksgeist.Savigny‘s contribution to the development of
historical school may briefly be stated under the following heads:--

1. Law develops like language— Savigny pointed out that law has a national
character and it develops like language and binds the people into one whole
because of their common faith, beliefs and convictions. According to him
law grows with the growth of the society and gains its strength from the
society itself and finally it withers away as the nation loses its nationality.
Law, language, customs and Government have no separate existence from
the people who follow them. Common conviction of the people makes all
these as a single whole.19The central theme of savigny‘s historical
jurisprudence may be summarised thus:--
―The organic evolution of the law with the life and character of the people
develops with the ages, and in this it resembles language. As in the latter
there can be no instant of rest, there is always movement, and development
of law is governed by the same power of internal necessity as simple
phenomena. Law grows with nation, increases with it, and dies at its
dissolution and is a characteristic of it.‖
2. Early development of law is spontaneous; thereafter jurists develop it---
- Savigny stated that in the earliest stages law develops spontaneously
according to the internal needs of the community but after the community
reaches a certain level of civilisation, the different kinds of national
activities, hitherto developing as a whole, bifurcate in different branches to
be taken up for further study by specialists such as jurists, linguists,
anthropologists, scientists etc. Law has to play a dual role namely as a
regulator of general national life and as a distinct discipline for study. The
former may be called the political element of law while the latter as a juristic
element but both have a significant role in the development of law. The
history of Roman law furnishes the best illustration of these processes. At its
earliest stage it was founded on general consciousness of the people but as it
grew and developed, it assumed the complex and technical form of law of
edicts.
3. Savigny was opposed to codification of German Law---- Savigny was
not totally against codification of laws. He, however opposed the
codification of the German law of the French (Napoleonic Code) pattern at
that time because Germany was then divided into several smaller states and
its law was primitive, immature and lacked uniformity. He opined that
German law could be codified at a later stage when the unification of
Germany takes place and there is one law and one language throughout the
country. Since Volksgeist i.e. common consciousness had not adequately
developed at that time, therefore codification would have hindered the
evolution and growth of law. He emphasised that codification of German
law without having jurist of sufficient genius and adequate expertise in
Roman law would not serve the desired purpose as Roman law formed an
integral part of the German legal system t that time. He considered lawyers
and jurists as true representatives of the popular consciousness rather than
the legislators whose role is limited to law- making only.
4. Law is a continuous and unbreakable process: Tracing the evolution of
law from Volksgeist, namely, people‘s spirit of consciousness. Savigny
considered its growth as a continuous and unbreakable process bound by
common cultural traditions and beliefs. It has its roots in the historical
process hich should constitute the subject of study for the jurists. According
to him, codification of law may hamper its continuous growth and therefore,
it should be resorted to when the legal system has fully developed and
established.
5. Admiration for Roman law: While emphasising Volksgeist i.e. people‘s
spirit or as the essence of law, Savigny justified adoption of Roman law in
the texture of German law which was more or less discussed in it. He,
located Volksgeist in the Romanised German customary law and considered
Roman law as an inevitable tool for the development of unified system of
law in Germany.
Savigny‘s admiration for Roman law was, however, criticised by Professor
Eichhorn who was his customary Professor in the University of Berlin. He
wondered how a foreign law could be true Volksgeist (popular will) of the
German people. Professor Eichhorn was totally against Roman law and
wanted German law to be relieved from its influence. On the other hand
Savigny and his followers were opposed to the expulsion of Roman law
from Germany. Thus there was a conflict between the so called Romanist
and the Germanists, the former supporting the retention of Roman law while
the latter advocating its total expulsion from the German law. The rift
between the two so called resolved by the final German law draft of 1900
which was a combination of both German law and Roman law.

INDIAN VOLKSGEIST- NEED OF CAUTION


Nationalism, national unity and integration of India as a strong, inalienable
and progressive nation is the need of hour. This glorious ideal of India as a
nation we cherish from our culture and traditions in an unbroken fashion and
is both a geographical fact and a cultural and historical entity. Indeed the
nationalism is sustained by the love of the country in which we live, the
traditions we inherit and the hope of a common shared future. Of course, the
quintessence of Indian unity and heritage has been a continuing fusion
among different religion, beliefs, linguistic and regional variations by
harmonising them within the great mosaic ideal of India as a nation. The
seers of Upanishads, Buddha, Mahavira, Guru Nanak, Vivekananda,
Mahatma Gandhi and others have peached unity in diversity and not
excluvism. This unholy alliance has fragmented the society on the basis of
religion, caste etc. resulting in social and political tensions and conflicts
even on issue of national solidarity, unity and integrity. The after math of the
Shah Bano (1986) and the Khatoonisa (1994), and the controversy over
Uniform Civil Code are such instances whereby Muslims have been
advocating and perpetuating their separatist Islamic identity before after
1947 when India was partitioned in opposition to Indian Volksgeist. The
Supreme Court reminded,22 the orthodox and conservative Muslim politico –
religious leaders that those who [referred to remain in India after the
partition, fully knew that the Indian leaders did not believe in two nation
theory and that in the Indian republic there was to be only one Nation ----
Indian nation and no community could claim to remain a separate entity on
the basis of religion.‘ Of course, the need of the hour is to consolidate India
nation by fostering and promoting linkages and help in strengthening the
bonds of unity and avoid the politics of religion or caste which is responsible
for contemporary bigotary, fundamentalism, linguistic and regional
parochialism.
Peoples Union of Human Rights V. Union of India, President Citizen for
Democracy V. Union of India, G. Sumahati V. Director, Medical
Education, Madras. The balkanization of Yugoslavia and former Soviet
Russia should be an eye opener to all nationalist and patriotic Indians that
we must preserve and protect the constitution which Granville Austin had
described ‗Cornerstone of a Nation‘ as a guarantee of Indian unity. The
observation26of Sardar Patel in the constituent assembly are a grim lesson
and a warning to all those who divide people on the issue of exclusivism,
fundamentalism and human rights to balkanize the Indian union. Likewise
Nehru, exhorted people to understand and realise from the past history as he
put it Nehru Speech in the Constituent Assembly.
‗..................if India goes down, all will go down. If India thrives, all will
thrive and if India lives, all will live...............‖

CRITICISM AGAINST SAVIGNY‟S THEORY OF LAW


From the above it is clear that Savigny‘s Volksgeist theory carried huge
importance in German Legal science but besides its benefits this theory was
criticised by many jurists. Thus it is stated that Savigny‘s theory has been
opposed by his critics on several grounds, the main among those are as
follows:--
1. There are certain inconsistencies which are apparent in Savigny‘s theory.
He emphasized the national character of law but at the same time suggested
a model by which Roman law could be adopted and accepted as the law of
Germany. Again, he located origin of law in the Volksgeist, that is popular
conscience but at the same time asserted that certain customary principles of
Roman law had universal application. savigny‘s undue importance to Roman
law has been bitterly criticised by Eichhorn, Beselor, and Gierke and it
was because of their intervention that German code was drafted in
subsequent years.
2. It is often alleged that Savigny‘s theory of law is negative, obscure and
suffers from narrow sectarian outlook. He was against codification of law
which is one of the most accepted forms of modern progressive legislation.
This anti-codification attitude of Savigny thwarted the growth of German
law for several decades.
3. Savigny‘s assertion that popular consciousness is the sole source if law is
not wholly true. The theory of Volksgeist overlooks the impact of other
sources of law such as legislation, precedent etc. in the evolution of law.
There are many areas which would have been left without legal rules
because there never existed any popular consciousness about them.
4. Again, Savigny‘s view that customs are always based on the popular
consciousness is not accepted. Many customs such as slavery, bonded labour
etc. originated to accomplish the selfish interest of those who were in power.
They are adopted because they are being blindly followed and continued for
a long time and not because they‘re righteous and have the support of
popular consciousness.
5. Roscoe Pound has criticised for his juristic pessimism. Savigny‘s theory
hindered legal reforms and modernisation of law in the name of Volksgeist.
Criticising Savigny‘s legal theory, Pound observed that no legal system
would like to stick to the prevalent abuses and baneful customs only because
people are accustomed to them. Savigny therefore overlooked the creative
role of law by introducing legal reforms. As Prof. Porkunove rightly pointed
out Savigny‘s theory ―does not determine the connection between what is
national and what is universal‖.
6. Though savigny was not against legislative reform by way of codification
of laws, but his approach towards codification was rather cold and
pessimistic because in his view codification could never solve all the
problems that are likely to arise in future and imperfect code would create
more problems by ‗perpetuating follies‘underlying it, he firmly believed that
codification should be preceded by a progressive scientific study of law after
taking into consideration the historical evaluation of the particular law.
7. Last but not the least, Savigny‘s volksgeist helped many nations to
prevent it for promoting their own ideologies. Thus Nazi twisted it by giving
a racial colour, the Marxists used it giving economic interpretation of history
and Italy used to justify Fascism.
ROLE OF JUDICIARY Though much role of judiciary cannot be traced in
this aspect, but one reference case that can be cited is the--- Nepalese Case
Reference: Meera Kumari Dhungana v His Majesty‟s Government
Ministry of Law, Justice and Parliamentary Affairs and others:

"Making sudden changes in traditional social practices in matters of


social norms perused by the society since a long time ago, may create
problems in connection to adjustment in the society. And, it may cause
such a situation beyond perception. Therefore, before reaching a decision
all of a sudden, a just provision should be made by holding wide and
extensive discussions and deliberations taking into account the constitutional
provision vis-à- vis equality. As the family law relating to property is to be
wholly considered, it is hereby issued this directive order that HMG
introduce an appropriate Bill to Parliament within 1 year of receipt of this
order, by making necessary consultations with the recognized women's
organizations, sociologists, the concerned social organizations and the
lawyers as well and by studying and considering the legal provisions in this
regard on other countries."

Roscoe Pound
(Q) Describe Roscoe Pound‟s theory of ‗balancing of interests‟ and ‗social
engineering‘. How far do you consider his theory relevant in India? [2018(7)]

(Q)What are the different broad categorization of ‗interest‘ according to Rosco


pound‟s „Interest Theory‟. [2017(5)(b)]

(Q) Describe Roscoe Pound”s theory of balancing of interests. Critically


analyses the theory with examples from the Indian Legal System.. [2016(7)]

(Q) ―The making, interpretation and application of laws must take into account
several social facts including the recognition and identification of different interest
of society‖. Critically examine this statement by explaining the feature of
sociological school of law led by Roscoe Pound. [2015(8)]

This school is perhaps one which covers some of the biggest variety of
different approaches but ultimately, the name most associated with this type of
jurisprudence is that of Dean Roscoe Pound. At the time of the theory‘s
development, Pound identified that sociological jurisprudence tended to either
identify itself with the positivist type of jurisprudence insisting on a singularly
mechanical interpretive approach. Others presented the ideas of sociological
jurisprudence from an anthropological-ethnological stage which is also
interpretive but instead of concentrating on the mechanisms of social forces like
the first group, they rather are more concerned with ethnological interpretation –
making comparative studies of primitive institutions or generalising jural
materials gathered by ―a purely descriptive social science.‖

According to Pound, this leads to the ―unhappy tendency‖ that is commonly


identifiable in a lot of the older jurisprudential theory of insisting upon one
approach or one method of investigation or interpretation. Pound‘s primary
concern therefore was to advance a more unified approach blending the
methodological and normative precepts which is a feature of his work as well as
focusing more on the way law develops in society rather than analysis and
interpretation of legal texts. In Pound‘s words: “…in the past century, we
studied the law from within. The jurists of today are studying it from
without‖.
Pound defined the goals of sociological jurisprudence in an essay on the
philosophy of law:

―What we-are seeking to do and must do in a civilized society is to adjust relations


and order conduct in a world in which the goods of existence, the scope for free
activity, and the objects on which to exert free activity are limited, and the
demands upon those goods and those objects are infinite. To order the activities of
men in their endeavor to satisfy their demands so as to enable satisfaction of as
much of the whole scheme of demands with the least friction and waste has not
merely been what lawmakers and tribunals and jurists have been striving for, it has
also been put in one way or another by philosophers as what we ought to be
doing.‖

This statement contains a summary of Pound's program. In the perfect society all
claims put forward by individuals would be immediately satisfied. However, such
a utopia does not exist in the world and people cannot satisfy each of their
demands. Further, men come into conflict when they desire the same scarce
goods. The basic tenets of sociological jurisprudence place Pound in the tradition
of Hobbes:

―And therefore if any two men desire the same thing, which nevertheless they
cannot both enjoy, they become enemies: and in the way to their end, which is
principally their own conservation, and sometimes their delectation only, endeavor
to destroy or subdue one another.‖

Pound articulated a conflict theory founded on individual interest. To escape from


a state of nature men devise systems of law which enable them to fulfill "as much
of the whole scheme of demands with the least friction and waste." The legal
system is made effective through a monopoly of coercive power:

―I think of law as in one sense a highly specialized form of social control in a


developed politically organized society -a social control through the systematic and
orderly application of the force of such a society.‖

Thus, sociological jurisprudence introduces two components into its definition of


law. On the one hand, law is a means of alleviating conflict through the
imposition of organized force. On the other, law functions to secure the
realization of as many individual interests as possible. An appropriate question
arises here whether the second assertion is meant to state a fact or prescribe an
ideal. We will examine this question, but for now we may accept these components
as the starting point for Pound.

Although the theory of interests was prefigured in nineteenth century


jurisprudence, Pound brought the approach to maturity. He devised categories
under which to subsume the demands enunciated in social systems:

If we take, as it were, an inventory of the concrete claims which press upon the law
for satisfaction and seek to classify those which the law recognizes and endeavors
to secure, they fall conveniently into three groups: individual interests, public
interests, and social interests.
THE CONTRIBUTION OF ROSCOE POUND

The contribution of Roscoe pound to Sociological Jurisprudence may be studied


under the following heads-

1. EMPHASIS ON FUNCTIONAL ASPECT OF LAW-


2. POUND‟S THEORY OF SOCIAL ENGINEERING

i. Private interests

ii. Public interests

iii. Social interests


1. EMPHASIS ON FUNCTIONAL ASPECT OF LAW:

Roscoe pound added new dimensions to Sociological school of Jurisprudence.


His approach to Sociological Jurisprudence was different in the sense that he
attempted to cover social life as a whole unlike his predecessors who
considered law as the main subject of study and society is merely subsidiary to
it. Pound laid greater stress on functional aspect of law. He defined law is the
containing ― the rules , principles, conceptions and standards of conduct and
decision as also the precepts and doctrines of professional rules of art.‖ He thus
considers law as ameans of a developed technique and treats jurisprudence as a
„social engineering‟. The end of law according to him is to satisfy a maximum
of wants with a minimum of friction or confrontation. Elaborating the
functional aspect of law, Roscoe pound stated that the function of law is to
reconcile the conflicting interest of individuals in the community and harmonise
their inter-relations. He termed this as ―social Engineering‖.
2. POUNDS THEORY OF SOCIAL ENGINEERING:

Roscoe pound conceived law as a ‗social Engineering‘ its main task being to
accelerates the process of social ordering by making all possible efforts to avoid
conflicts of interest of individuals in the society . Thus courts, legislators,
administrators and jurists must work with a plan and make an effort to maintain
a balance between the competing interests in society. He enumerates various
interests which the law should seek to protect and classified them into three
broad categories, namely

I. Private Interests / Individual Interest Individual interests, according to


pound are claims, or demands or desires, involved in and looked at from
the stand point of the individual life immediately as such asserted in title
of the individual life‘. In individual interest Dean Pound includes-
1) Personality- interest of personality consist of interests in - a. the
physical person, b. freedom of will, c. honour and reputation, d.
Privacy ,and sensibilities and e. Belief and opinion.
2) Domestic relations - it is important to distinguish between the
interest of individuals in domestic relationships and that of society in
such institutions as family and marriage. Individual interests include
those of a. Parents and Children, b. Husbands and Wives. c. And
marital interests.
3) Interest of substance- this includes a. Interests of property, b.
Succession and testamentary disposition, c. freedom of industry and
contract, d. promised advantages e. advantageous relations with
others, f. freedom of association, and g. Continuity of employment.
2) Domestic relations - it is important to distinguish between the interest
of individuals in domestic relationships and that of society in such
institutions as family and marriage. Individual interests include those
of a. Parents and Children, b. Husbands and Wives. c. And marital
interests.
3) Interest of substance- this includes a. Interests of property, b.
Succession and testamentary disposition, c. freedom of industry and
contract, d. promised advantages e. advantageous relations with
others, f. freedom of association, and g. Continuity of employment.
II. Public Interest :

Public interests according to him are the claims or demands or desires asserted
by individuals involved in or looked at from the stand point of political life- life
in politically organised society. They are asserted in title of that organisation. It
is convenient to treat them as claims of politically organised society thought of
as a legal entity.The main public interest according to Roscoe pound are-

1. Interests of state as a juristic person which includes

a. Interests of state as a juristic person i.e. protection

b. Claims of of the politically organised society as a corporation to property


acquired and held for corporate purposes.

2. Interests of State as a guardian of social interest, namely superintendence


and administration of trusts,charitable endowments, protection of natural
environment, territorial waters, sea-shores, regulation of public employment
and so on to make use of thing which are open to public use , etc. this interest
seem to overlap with social interests.

III. Social Interests To pounds social interest are claims or demands or desires,
even some of the foregoing in other aspects, thought of in terms of social life
and generalised as claims of the social group. They are the claims functioning
of society; the wider demands or desires ascertained in the title of social life in
civilised society. Social interest are said to include -
a. Social interest in the general security, -

Social interest in the general security embraces those branches of the law which
relate to general safety, general health, peace and order, security of acquisitions
and security of transactions.
b. Social interest in the security of social institutions,
Social interest in the security social institutions comprises domestic institution,
religious institutions, political institutions and economic institutions. Divorce
legislation may be adduced as an example of the conflict between the social
interests in the security of the institution of marriage and the individual interests
of the unhappy spouses. There is tension between the individual interest in
religious freedom and the social interest in preserving the dominance of an
established church.

c. Social interest in general morals,-

Social interests in general morals covers a variety of laws, e.g. laws dealing
with prostitution, drunkenness and gambling;
d. Social interest in the conservation of social resources,-

Social interests in the conservation of social resources covers conservation of


social resources and protection and training of dependents and defectives , i.e. ,
conservation of human resources, protective and education of dependents and
defectives , reformation of delinquents, protection of economically dependents.
e. Social interest in general progress and –

Social interest in general progress has three aspects. Economic progress,


political progress and cultural progress. Economic progress covers freedom
of use and sale of properly, free, trade , free industry and encouragement of
inventions by the grant of patents. Political progress covers free speech and free
association, free opinion, free criticisms. Cultural progress covers free science,
free letters, encouragements of arts and letters, encouragements of higher
education and learning and aesthetics.
f. Social interest in individual life.

Meaning thereby each individual be able to live a human life according to the
individual‘s (a) political life,(b) physical life,(c) cultural ,( d) social and ( e)
economic life.

3.JURAL POSTULATES OF ROSCOE POUND

In order to evaluate the conflicting interests in due order of priority , pound


suggested that every society has certain basic assumption upon which its
ordering rests, through for most of the time they may be implicit rather than
expressly formulated. This assumption may be called as jural postulates of the
legal system of that society. Pound has mentioned five jural postulates as
follows

A. Jural postulate I- in civilised society men must be able to assume that


others will commit no intentional aggression upon them.

B. Jural postulate II- in civilised society men must be able to assume that they
may control for beneficial purposes what they have discovered and appropriated
to their own use, what they have created by their own labour and what they
have created by their own labour and what they have acquired under the
existing social and economic order.

C. Jural postulate –III – In a civilised society men must be able to assume that
those with whom they deal as a member of the society will act in good faith and
hencei. Will make good reasonable expectations which their promises or other
conduct reasonably create;
ii. Will carry out their undertaking according to the expectations which the
moral sentiment of the community attaches thereto.
iii. Will restore specifically or by equivalent what comes to them by mistake,
orfailure of the pre-suppositions of a transaction, or other unanticipated
situation whereby they receive at other‘s expense what they could not
reasonably have expected to receive under the actual circumstances.

D. Jural postulate iv- in civilised society men must be able to assume that
those who engage in some course of conduct will act with due care not to cast
an unreasonable risk of injury upon others.

E. Jural postulate V-in a civilised society men must be able to assume that
others who maintain things or employ agencies, harmless in the sphere of their
use but harmful in their normal action elsewhere, and having a natural tendency
to cross the boundaries of their proper use will restrain them and keep them
within their proper bounds.

Pounds confessed that these jural postulates are not absolute but they have a
relative value. These are a sort of ideal standards which law should pursue in
society they are of a changing nature and new postulates may emerge if the
changes in society so warrant. Thus the jural postulates by Roscoe pound
provide guidelines for righteous and civilised life and they also seek to strike a
synthesis between reality and idealism as also power and social accountability
of men in the community.
CRITICISM AGAINST POUND‟S THEORY:

Pound point out the responsibilities of the judge and the lawyer. Pounds
contribution to jurisprudence is great. He takes a middle way avoiding all
exaggeration. He speaks of value but says that they are relative. He emphasises
engineering but does not forget the task of maintaining of balance, his approach
is experimental. Pounds theory stands on a practical and firm ground and it has
inspired great practical field –work. His emphasis on studying the actual
working of legal rules in the society, the importance of social research for good-
law making and pointing out the great constructive function which the law is to
perform are very valuable contributions to jurisprudence. He point out the
responsibility of the lawyer, the judge and the jurist and gives a comprehensive
picture of the scope and field of the subject. Pound‘s influence on modern legal
thought is great and the study of the subject is being undertaken under the light
of his theory.

Despite pound‘s great contribution to sociological jurisprudence and his


emphasis on studying the actual working o flaw in the society, his theory
suffers from certain drawbacks. Pounds theory of social engineering has been
criticised on various grounds.

It is contended that the classification of interests by pound is in the nature of a


catalogue to which additions and changes have constantly to be made which is
neutral as regards the relative value and priority of the interest enumerated.
Pound‘s theory of social engineering has been criticised for the use of the term
engineering, which equates society to a factory like mechanism. Law is a social
process rather than the result of an applied engineering. Equating society with a
factory is also not correct because the former is changing and dynamic in nature
whereas the latter is more or less static. Again , pound‘s emphasis on
engineering ignores the fact that law evolves and develops in the society
according to social needs and wants for which law may either have develops in
the society according to social needs and wants for which law may either have
approbation or disapprobation.

In Case-ANIMAL AND ENVIRONMENT LEGAL DEFENCE FUND V/S


UNION OF INDIA AND OTHERS
JUDGEMENT- The supreme court applied the principles of Economic
sustainability and environment protection. If the villagers are not permitted
fishing, their livelihood shall be destroyed. If they are permitted there will be a
threaten to the ecology. Hence the Supreme Court ordered the concerned forest
authorities and the Board constituted to take necessary steps to protect the
environment. They shall watch the villagers and give suitable instructions to
them. They shall be educated on the importance of environment. The villagers
should not enter in other areas accepts to the lakes on which they are given
fishing rights.
PRINCIPLE-The supreme court applied sociological approaches in this case
for the welfare of tribals, whose source of livelihood is fishing. Not only in this
case, but also in every environmental case, the sociological approach of their
lordship is crystal clear. Their lordships often say that ―law is a social
engineering‖. It may be stated that pound‘s classification of interests in his
theory of social engineering cannot be said to be fool- proof and one may find
some overlapping of interests here and there. Pound himself accepted that the
various interests of individuals in a society can only be broadly classified and
they cannot be placed in water tight compartments. Julius stone has rejected the
division of public interests and social interests on the ground that in fact they
are all social interests.

Pounds tackled the problem of interests in terms of balancing of individual and


social interests. It is through the instrumentality of law that these interests are
sought to be balanced. as justice Cardozo rightly remarked , ― pound attempted
to emphasises the need for judicial awareness of the social values and interests‖.

Sociological School and Indian Position


To India, sociological school of jurisprudence has much relevance in terms of
realisation of the social and economic goals of the community. Law in free
India is being adjusted to serve the common needs and ends of society along
with individual interests. However, before 1947 the judges, the lawyers and law
administrators did not look around while making new laws. The pace of social
change was very slow as the law was conceived either in analytical fashion
emanating from the British Parliament or the law had no relationship or
relevance with the life of people of India. The role of judges was not to discover
the ‗inarticulate major premises‘ or ‗felt needs‘ of the people but to interpret the
law in its logical manner irrespective of the considerations of social justice. The
law was mostly imposed from above. It had no roots in the Indian soil and its
language too was foreign.
After 1947 there was a change in the perspective of law itself. India became
free and it adopted the new Constitution with a view to establish justice –
social, economic and political. To achieve these set goals Indian planners
introduced the system of economic planning in India with a view to promote the
welfare of the people by securing and protecting as effectively as it may a social
order in which justice, social, economic and political shall inform all
institutions of national life. Accordingly State evolved new social and economic
policies to achieve the above ends. The old analytical approach towards law
was obviously abandoned in the interest of common goods as it was
unnecessary, unreal and inconvenient to the emergence of new social order.
Hence a new sociological approach for reconciling conflicting social interests
and values became necessary for bringing peaceful social change through law.

Hohfeld

(Q) Explain Hohfeldian analysis of the concept of ―Right” with at least one
example for each of the jural correlatives. [2019(4)(b)]

(Q) Hohfeld‟s analysis of Duties towards the state and society. [2018(8)(b)]

(Q) Explain Hohfeld‟s Analysis of Rights & Duties. [2017(8) (b)]

Hohfeld‟s Theory of Jural Relations :

One of the greatest hindrances to the clear understanding, the incisive statement,
and the true solution of legal problems, frequently arises from the express or tacit
assumption that all legal relations may be reduced to ―rights‖ and ―duties,” and
that these latter categories are therefore adequate for the purpose of analyzing even
the most complex legal interests, such as trusts, options, escrows, ―future‖
interests, corporate interests, etc.

Even if the difficulty related merely to inadequacy and ambiguity of terminology,


its seriousness would nevertheless be worthy of definite recognition and persistent
effort toward improvement; for in any closely reasoned problem, whether legal or
non-legal, chameleon-hued words are a peril both to clear thought and to lucid
expression. As a matter of fact, however, the above mentioned inadequacy and
ambiguity of terms unfortunately reflect, all too often, corresponding paucity and
confusion as regards actual legal conceptions.

The strictly fundamental legal relations are, after all, sui generis; and thus it is that
attempts at formal definition are always unsatisfactory, if not altogether useless.
Accordingly, the most promising line of procedure seems to consist in exhibiting
all of the various relations in a scheme of ―opposites‖ and ―correlatives,‖ and then
proceeding to exemplify their individual scope and application in concrete cases.
An effort will be made to pursue this method:

Jural Rights Privilege Power Immunity


Opposite No-Rights Duty Disability Liability

Jural Right Privilege Power Immunity


Correlatives Duty No-Right Liability Disability

Rights and Duties: As already intimated, the term ―rights‖ tends to be used
indiscriminately to cover what in a given case may be a privilege, a power, or
immunity, rather than a right in the strictest sense; and this looseness of usage is
occasionally recognized by the authorities. ...
Recognizing, as we must, the very broad and indiscriminate use of the term,
―right,‖ what clue do we find, in ordinary legal discourse, toward limiting the word
in question to a definite and appropriate meaning. That clue lies in the correlative
―duty,‖ for it is certain that even those who use the word and the conception
―right‖ in the broadest possible way are accustomed to thinking of ―duty‖ as the
invariable correlative. ...
In other words, if X has a right against Y that he shall stay off the former‘s land,
the correlative (and equivalent) is that Y is under a duty toward X to stay off the
place. If, as seems desirable, we should seek a synonym for the term ―right‖ in this
limited and proper meaning, perhaps the word ―claim‖ would prove the best. ...

Privileges and “No-Rights.” As indicated in the above scheme of jural relations, a


privilege is the opposite of a duty, and the correlative of a ―no-right.‖

In the example last put, whereas X has a right or claim that Y, the other man,
should stay off the land, he himself has the privilege of entering on the land; or,
in equivalent words, X does not have a duty to stay off. The privilege of entering is
the negation of a duty to stay off. As indicated by this case, some caution is
necessary at this point, for, always, when it is said that a given privilege is the mere
negation of a duty, what is meant, of course, is a duty having a content or tenor
precisely opposite to that of the privilege in question. Thus, if, for some special
reason, X has contracted with Y to go on the former‘s own land, it is obvious that
X has, as regards Y, both the privilege of entering and the duty of entering. The
privilege is perfectly consistent with this sort of duty,—for the latter is of the
same content or tenor as the privilege;—but it still holds good that, as regards Y,
X‘s privilege of entering is the precise negation of a duty to stay off. Similarly, if
A has not contracted with B to perform certain work for the latter, A‘s privilege of
not doing so is the very negation of a duty of doing so. Here again the duty
contrasted is of a content or tenor exactly opposite to that of the privilege.

Passing now to the question of “correlatives,‖ it will be remembered, of course,


that a duty is the invariable correlative of that legal relation which is most properly
called a right or claim. That being so, if further evidence be needed as to the
fundamental and important difference between a right (or claim) and a privilege,
surely it is found in the fact that the correlative of the latter relation is a ―no-right,‖
there being no single term available to express the latter conception. Thus, the
correlative of X‘s right that Y shall not enter on the land is Y‘s duty not to enter;
but the correlative of X‘s privilege of entering himself is manifestly Y‘s ―no-right‖
that X shall not enter.
In view of the considerations thus far emphasized, the importance of keeping the
conception of a right (or claim) and the conception of a privilege quite distinct
from each other seems evident; and more than that, it is equally clear that there
should be a separate term to represent the latter relation. No doubt, as already
indicated, it is very common to use the term ―right‖ indiscriminately, even when
the relation designated is really that of privilege; and only too often this identity of
terms has involved for the particular speaker or writer a confusion or blurring of
ideas. ...
A ―liberty‖ considered as a legal relation (or ―right‖ in the loose and generic sense
of that term) must mean, if it have any definite content at all, precisely the same
thing as privilege ... . It is equally clear, as already indicated, that ... a privilege or
liberty to deal with others at will might very conceivably exist without any peculiar
concomitant rights against ―third parties‖ as regards certain kinds of interference.
Whether there should be such concomitant rights (or claims) is ultimately a
question of justice and policy; and it should be considered, as such, on its merits.
The only correlative logically implied by the privileges or liberties in question are
the ―no-rights‖ of ―third parties.‖ It would therefore be a non sequitur to conclude
from the mere existence of such liberties that ―third parties,‖ are under a duty not
to interfere, etc.

Answer to Question [2018(8)(b)] :

Hohfeld Duties :

The term "duty" is commonly used in ways that Hohfeld clearly did not have in
mind. By "duty" he did not mean what a father means when saying to his son, "It
is your duty to mow the lawn every week." Nor did he mean what a moralist means
when saying, "Every man has a duty to treat others as he would have them treat
him." These differences are easy to see; Hohfeld was talking about legal duties.
But still, there are different kinds of legal duties, corresponding to different views
of what "law" is. Most lawyers are legal positivists; they think of "law" as being a
body of rules, so they tend to see a legal duty as something derived from the rules
of positive law. However, many lawyers feel that "law" is inextricably tied up in
social policy and that a legal duty arises from the goals of society, the public good,
or whatever. When social policy gives rise to a right, they say, it is for the positive
law to spell out a remedy; and if it fails to provide for a remedy, that is a defect in
the positive law.Other lawyers (the legal realists) think of "law" as being what
officials, especially courts, do about disputes brought to them for settlement. These
lawyers would say the availability of an actual court judgment is the acid test for a
legal duty.5 Regardless of what is good for society or what the rules of positive
law say, the realists feel if a plaintiff's key witness dies and leaves him unable to
prove his case then in fact he has no right, and the wrongdoer has no duty.

All of the different meanings of the word "duty" seem to have one thing in
common, something bad is likely to happen somewhere when a duty is
breached.If the son should fail in his duty to mow the yard every week, his father
no doubt contemplates punishing him in some way, at least by withholding favors
he would otherwise allow.. Should a person fail in the moralist's duty to follow the
Golden Rule, then, at least in the moralist's view of things, the world would
somehow be worse off. For the son's lawnmowing duty, the "something bad" that
is likely to happen in the event of a breach is a kind of sanction to be imposed
against the errant son himself. But in the case of the moralist's duty, the
wrongdoer may himself go scot-free. The bad thing that results from his breach
of duty is borne by the whole world. The lawyer who identifies the "law" with
what is good for society might view a "legal" duty as something like the moralist's
duty. The "something bad" which results from a breach is not primarily a sanction
on the wrongdoer but rather an impairment of the values or goals of society. By
contrast, to the realist who sees law as what courts do, the bad thing that results
from a breach of a legal duty would be a sanction, a court judgment of liability,
that is realistically available against the wrongdoer.

The "duty," if it is of the Hohfeldian variety, arises from the rules of liability, and
if the rules do not make the defendant liable for what he did, then he had no duty
(in Hohfeld's sense), which he could breach. However, courts very often base their
decisions on "duties" that simply do not spring from the pre-existing rules of
positive law. Then, a statement like the above is likely to mean something like this:
The defendant is liable for plaintiff's injuries, not because the rules of law make
him liable, but because (in our opinion) it is socially desirable that he bear the loss
rather than the plaintiff.

It should be pointed out that there are many so-called "duties" which come from
the positive law but which evidently are not Hohfeldian duties because they are not
directly related to rules of liability. The so-called "duty" to mitigate damages is an
example. If an injured person who has a good cause of action fails to take
reasonable measures to minimize his injury, he may be precluded from recovering
the damages he could have avoided. The rules of law may be said to impose
"something bad" for his failure, but it is only a limitation on his right of recovery
rather than civil liability that is imposed.

Hohfeld used the term "duty" in an even more specific way than what we have
outlined thus far. Hohfeldian duties always involve two people, one who is said to
have or owe the duty, and another to whom the duty is said to be owed. Thus, A
might owe a Hohfeldian duty to B, but he cannot owe such a duty to himself, and
A might owe two separate duties to B and C, but he cannot owe the same duty to
both of them. The reason for this usage becomes clear when it is recalled that

(1) A Hohfeldian duty to do something arises when the positive law would make a
person civilly liable for not doing it, and

(2) Hohfeld was trying to reduce legal conceptualization to its lowest common
denominators. If under the rules of law A would be liable to B for not doing
something, then A owes a duty specifically to B to do it. When the rules of law
would make A liable to both B and C for not doing something, Hohfeld preferred
to say that A owes separate duties to B and C since his liability to them in event of
breach would be separable. Their causes of action would be independent of one
another, so A's duties (if they are to be "lowest common denominators") should
also be independent of one another. It is thus seen that Hohfeld used the term
"duty" to designate a certain kind of legal relation between pairs of specific people
with respect to an act. There is really no such thing as a "duty" beyond this. To say
that A owes B a duty to do a certain thing is merely to say something about how
the rules of law would treat A and B if it happened that A failed to do the thing.

(Q) ―The „is‟ and „ought‟ divide is a bone of contention between positivists and
naturalists. However, this divide is broken at a point of repeal of law by
legislative or judicial means.‖ Critically analyze the statement by comparing the
positivist and naturalist school of law. [2015(2)]

HOW ARE THE FACTS of the world in which man lives related to the values or
norms he lives by, the norms which are made imperative in law; are the realms of
fact (is) and value (ought) related in a particular way?
The Positivist Model

The positivists, in general, have always maintained a very clear notion of law,
being limited to what law ‗is‟. The primary aim for the positivist is only limited to
study the law, as it is. The law got the very sanction behind it since it has got its
validity from the authority itself. Positivist views this authority, as the only reason
to obey the law, in the famous words of Austin, it being the ―Command of the
Sovereign‖. Perhaps it might not be true to much extent in today‘s world, which
we will discuss later. Positivist, often does not entails themselves with, what ought
to be, separating the ideals from the law, which is why there seems a great
criticism of positivist separating law from moral.

Bentham and Austin

In the 18th and 19th century, legal thinker developed the legal positivism, mainly
by Jeremy Bentham and Austin. This came as an opposition to the natural law
theory. Contention made under this only relates to the fact about how law has
been created. A positivist will not go under the scrutiny of what law consist of
but will only prefer scrutinizing how the law comes into force. As per the
philosophy, it is not the work of legal positivist to judge the law by the question of
justice and humanity, but only by the way the law has been created. What all were
classic criticism of Positivism theory will be dealt under the naturalist model.

Jeremy Bentham was the one who initiated with that of legal positivism. But
Bentham, did move towards an angle of putting moral philosophy into it. He gave
his famous understanding of greatest happiness principle or the utilitarian
principle. In this, he explains that it is not just the usefulness but also determining
the extent to which it creates happiness. Bentham does give importance to the fact
of creation of moral obligation to produce the greatest amount of happiness. These
are much apparent in his work, ―Introduction to principles of Morals and
Legislation‖, where he gave importance to the rational principle, to provide for
guide in legal reform. Also, in contrast with Hobbes, Bentham wants the legislature
to change, as it gives more impetus to the legislature to work in consonance.
Nevertheless, Bentham did held that law is not rooted in the natural law but it is the
command expressing the will of the sovereign. This view was later developed by
Austin. Hence, even if the law questions the moral fabric, this is still law. In spite
of the fact, Bentham gives his utilitarian theory, he mostly remained against the
natural law fabric.

On similar lines, Austin also approached on a strict line, negating the concept of
natural law and providing with the command theory. According to Austin, the law
is nothing but a command of sovereign. The sovereign being the one to whom all
people grant respect. There is always a part of sanction that is associated with the
command. Certainly, Austin‘s theory doesn‘t mark well considering the current
situation of political system. We will not be able to find any political system where
Austin‘s command theory will lie absolutely fit. Austin was crystal clear in
maintaining the notion of law being not moral. In one of the lecture, Austin
clearly said, ―The most pernicious laws, and therefore those which are most posed
to the will of God, have been and are continually enforced as laws by Judicial
tribunals‖. Even if the law is opposed to nature it is still a law, and must be obeyed,
since it is given by the sovereign.

The Naturalist Model

In the previous instance, where an example was referred in which there was no
punishment for murder, will this result in people committing murder everywhere?
The general answer will definitely be a ‗no‘. Hence, this situation clearly implies
that there is certain moral obligation which binds humans, irrespective of the fact
whether there is law or not. But the intersection is quite difficult to get accustomed
with since it revolves around obligatory nature of moral obligation. Because, on
the same previous note, one cannot deny the fact that there will be an increase in
cases of murder, once there is no punishment. Hence, this moral obligation is
decided through the conscience of a person which is different for different person.
The Natural School advocates the importance of reason behind everything. The
naturalist model do consist of valid legal knowledge but unlike the positivist
model, where the legal knowledge is constrained to only what the authority
says, the naturalist will base the legal knowledge with reason as well as with
rational awareness and acceptance of the existence of objective values,
standards and norms of proper conduct. This factor makes the natural school
more appealing, since, it gives us the presupposition which enables us to deal with
all these facts or judgment in an intelligent manner. To moreover, get into the
detail, natural school does not delve into declaring or discriminating the present
legal system or any legal theory. The main contention being the element of reason
must be complied by, irrespective of the legal theory it discusses.

Hans Kelsen and Lon Fuller :

Theories of law have sometimes issued according to the way this question has been
perceived by their formulators. The principle that one cannot derive a conclusion
which is not explicitly or enthymematically contained in its premises is basic to
deductive logic. Similarly, there can be no induction from the property of one
thing to something which is other than the thing itself, induction being a process of
generalization of the nature of an entire class from the characteristics ot a number
of members of that class.

Within the constraints of these principles of logic, attempts have been made to
reduce ought to is by replacement of ought statements with is-supportable
statements and by derivation of an ought statement from is statements concerning
desires and beliefs ' Attempts have also been made to deduce ought from is
through the notions of the performative aspect of ought, the constitutive rules of
institutional facts, the reason for acting,' the ideal observer,' and the underlying
commitment." In legal theory, two opposite perceptions of the is-ought relationship
have emerged: one, the fission of is from ought; the other, the fusion of is and
ought. The pure theory of law proposed by Hans Kelsen follows the former
route.The latter approach is taken by both the purposive theory of law, proposed
by Lon Fuller," and by the phenomenological theories of law, proposed by N.A.
Poulantzas," W. Maihofer, and others.
THE FISSION

The pure theory of law is grounded in Immanual Kant's epistemology,' which


draws a fundamental distinction between man as part of nature, subject to the laws
of causality (mundus sensibilis), and man as a partly intelligible object which
regulates its conduct by imperatives (mundus intelligibilis). This distinction
results in an essential difference between is and ought , the ought being the
expression of a necessity or relationship which is not evident in the realm of
nature. Kant maintains that absolute reality as such is unknowable.

Kant also draws a basic distinction between form and matter: while sense
impressions constitute the matter of experience, the form is composed of the
mental modes of the cognitive subject (time and space) and the pure categories of
understanding (quantity, quality, relation, and modality). Formal categories bring
order to the chaos of emotion and sensations. Thus, through the forms of space
and time, emotions become perceptions. Similarly, through the categories of
understanding - substance and causality, quality and quantity - perceptions become
experience, with the judgments of experience being connected by general
principles.

There is a fundamental scission between nature and mind, and, correspondingly,


between the realms of cognition and volition.' Proceeding from this
epistemological distinction between is and ought, the pure theory of law divides
the sciences into causal sciences and normative sciences.'" The former deal with
reality, i.e., the is of actual events. The latter deal with ideality, i.e., ethical,
legal, esthetic, or other oughts. Law is a normative science, normative here being
employed in the sense of knowing the norm rather than constructing it. Being
normative, law deals not with the actual world of events (is), but with norms
(ought). Having made this formal-logical separation of the realms of is and ought,
the pure theory of law maintains that the inquiry into the sanction of an ought can
lead only to another ought.

Although the content of the is may or may not coincide with the content of the
ought, a contentual coincidence does not affect the logical division of the two
spheres of knowledge. The pure theory of law claims to be a formal and
universal theory, concerned with essentials of law of any kind, at any time, and
under any conditions. Its purity derives from being pure knowledge in the
Kantian sense, without "empirical admixture," and from being free of the alien
elements of morality and ethics. The pure theory characterizes the legal relation
as containing the threat of a sanction from an authority in response to a certain
act. The legal norm constitutes a relation of condition and sequence: if A is done, B
ought to happen.' A legal system is composed of a heirarchy of normative
propositions, each being derived from its superior. Every legal norm ultimately
derives from a highest basic norm (Grundnorm) which, not being capable of
deduction, must be assumed as an initial hypothesis.

A norm is a valid legal norm only by virtue of the fact that it has been created
according to a definite rule, with the basic norm of a legal order being the
postulated ultimate rule from which the legal norms are created or annulled. Law,
consequently, is created or annulled by acts of human beings. Therefore, it is
positive and independent of morality.

THE FUSION

The fusion of is and ought has been attempted by both the purposive theory of law
and the phenomenological theories of law. Proponents of these theories would
argue that the very distinction between is and ought disappears in a certain
type of activity. They would contend that the two do not represent autonomous
spheres of phenomena but only essential particularities in the structure of the
phenomena.

Fusion in the Purposive Theory of Law It has been argued that in a purposive
interpretation of human behavior the distinction between is and ought disappears,
and fact and value merge.

A conception of the activity's purpose becomes necessary in order to interpret what


is being observed. There are two aspects to understanding events: the control of
events and the prediction of events. The argument is that understanding cannot be
achieved in either of these two aspects without kowledge of the purpose of the
activity being pursued. Value inheres, for example, in a purely factual prediction
that a certain course of action will or will not be followed, since this prediction is
dependent upon knowing whether that course of action is suitable for its purpose,
or, in other words, whether it possesses value when judged in light of the purpose.
This value element is claimed to be intrinsic to the facts of a purposive activity.
Thus, the purposive theory does not view a concurrence of the observation of
events, the perception' of the purpose, and the provisional acceptance of this
purpose as creating an illusion of the merger of fact and value. On the contrary, it
holds that the course of events cannot be understood except through participation
in a process of evaluation.

This is the epistemology of Lon Fuller's purposive theory of law which


maintains that the legal process is the collaborative articulation of shared
purposes. Therefore, law is a purposive activity in which the merger of fact and
value occurs because of the internal morality of law.

This inner morality of law consists of eight principles: 1) its generality; 2) its
availability to the party affected (promulgation); 3) its prospective legal operation
(the general prohibition of retroactive laws) ; 4) its intelligibility and clarity; 5) its
avoidance of internal contradictions; 6) its avoidance of impossible demands; 7) its
constancy through time (avoiding frequent changes); and 8) congruence between
official action and declared rule.

It is doubtful that the is-ought distinction disappears in a purposive activity for


several significant reasons.

Hierarchy of Courts & Jurisdiction

(Q )Give a comparative account of the types of jurisdiction of the Supreme


Court and the High Courts in India. [2015(4)]

(Q) Hierarchy of courts in India. [2019(3)(a)]

(Q)Draw a flow chart of court system in India explaining briefly their


Jurisdiction. [2017(2)(a)]

(Q)Hierarchy of courts and jurisdiction in India [2016(3)(a)]

(Q)Hierarchy of lower judiciary in India. [2015(1)(b)]


Among the noble aims and objectives of the Constitution, the founding fathers
accorded the highest place to Justice . The Preamble speaks of ―We, the people of
India‖ resolving to secure inter alia Justice – social, economic and political‖ to
―all its citizens‖. The juxtaposition of words and concepts in the Preamble is
important. Most significantly Justice is placed higher than the other principles of
Liberty Equality and Fraternity.

Provisions in regard to the judiciary in India are contained in Part V (The Union)
under Chapter IV titled The Union Judiciary and Part VI (The States) under
Chapter VI titled Subordinate Courts respectively. It is, however important to
emphasize that unlike other federal systems, for example, that of the United States,
we do not have separate hierarchies of federal and State Courts. In India, though
the polity is dual, the judiciary is integrated. For the entire republic of India, there
is one unified judicial system – one hierarchy of courts – with the Supreme Court
as the highest court and also as the arbiter in matters of relations between the
Union and the States and the States inter se.

The Judiciary, in India, today is an extension of the British Legal System. The
Supreme Court is the apex body, followed by 24 High Courts, which in turn
supervise and govern numerous District Courts. Article 129 of the Constitution of
India makes the Supreme Court a 'court of record' and confers all the powers of
such a court including the power to punish for its contempt as well as of its
subordinate courts. Article 141 of the Constitution of India provides that the law
declared by Supreme Court is binding on all courts.

Hierarchy of Courts in India:

Supreme Court

High Court

Metropolitan Courts District and Session Court

Civil Criminal Civil Criminal


Munsif Court
Jurisdiction and Powers of Supreme Court(District Munsiff or
The Constitution has conferred a very extensive jurisdiction and vast powers on the
Junior Civil Judge)
Supreme Court. It is not only a Federal Court like the American Supreme Court but
also a final court of appeal like the British House of Lords (the Upper House of the
British Parliament). It is also the final interpreter and guardian of the Constitution
and guarantor of the fundamental rights of the citizens. Further, it has advisory and
supervisory powers. Therefore, Alladi Krishnaswamy Ayyar, a member of the
Drafting Committee of the Constitution, rightly remarked: ―The Supreme Court of
India has more powers than any other Supreme Court in any part of the world.‖
The jurisdiction and powers of the Supreme Court can be classified into the
following:

1. Original Jurisdiction.
2. Writ Jurisdiction.
3. Appellate Jurisdiction.
4. Advisory Jurisdiction.
5. A Court of Record.
6. Power of Judicial Review.
7. Other Powers.
Original Jurisdiction
As a federal court, the Supreme Court decides the disputes between different
units of the Indian
Federation. More elaborately, any dispute between:
(a) the Centre and one or more states; or
(b) the Centre and any state or states on one side and one or more states on
the other; or
(c) between two or more states.
In the above federal disputes, the Supreme Court has exclusive original
jurisdiction. Exclusive means, no other court can decide such disputes and
original means, the power to hear such disputes in the first instance, not by
way of appeal.
With regard to the exclusive original jurisdiction of the Supreme Court, two
points should be noted. One, the dispute must involve a question (whether of
law or fact) on which the existence or extent of a legal right depends. Thus,
the questions of political nature are excluded from it. Two, any suit brought
before the Supreme Court by a private citizen against the Centre or a state
cannot be entertained under this.
2. Writ Jurisdiction
The Constitution has constituted the Supreme Court as the guarantor and
defender of the fundamental rights of the citizens. The Supreme Court is
empowered to issue writs including habeas corpus, mandamus,
prohibition, quo-warrento and certiorari for the enforcement of the
fundamental rights of an aggrieved citizen. In this regard, the Supreme Court
has original jurisdiction in the sense that an
aggrieved citizen can directly go to the Supreme Court, not necessarily by
way of appeal. However, the writ jurisdiction of the Supreme Court is not
exclusive. The high courts are also empowered to issue writs for the
enforcement of the Fundamental Rights. It means, when the Fundamental
Rights of a citizen are violated, the aggrieved party has the option of moving
either the high court or the Supreme Court directly.
Therefore, the original jurisdiction of the Supreme Court with regard to
federal disputes is different from its original jurisdiction with regard to
disputes relating to fundamental rights. In the first case, it is exclusive and in
the second case, it is concurrent with high courts jurisdiction. Moreover, the
parties involved in the first case are units of the federation (Centre and
states) while the dispute in the second case is between a citizen and the
Government (Central or state).
There is also a difference between the writ jurisdiction of the Supreme Court
and that of the high court. The Supreme Court can issue writs only for the
enforcement of the Fundamental Rights and not for other purposes. The high
court, on the other hand, can issue writs not only for the enforcement of the
fundamental rights but also for other purposes. It means that the writ
jurisdiction of the high court is wider than that of the Supreme Court. But,
the Parliament can confer on the Supreme Court, the power to issue writs for
other purposes also.

Habeas Corpus
The Latin meaning of the word ‗Habeas Corpus‘ is ‗To have the body of.‘ This
writ is used to enforce the fundamental right of individual liberty against unlawful
detention. Through Habeas Corpus Supreme Court/High Court orders one person
who has arrested another person to bring the body of the latter before the court.
Facts about Habeas Corpus in India:

 The Supreme Court or High Court can issue this writ against both private
and public authorities.
 Habeas Corpus cannot be issued in the following cases:
 When detention is lawful
 When the proceeding is for contempt of a legislature or a court
 Detention is by a competent court
 Detention is outside the jurisdiction of the court
Mandamus
The literal meaning of this writ is ‗We command.‘ This writ is used by the court to
order the public official who has failed to perform his duty or refused to do his
duty, to resume his work. Besides public officials, Mandamus can be issued against
any public body, a corporation, an inferior court, a tribunal, or government for the
same purpose.
Facts about Mandamus in India:

 Unlike Habeas Corpus, Mandamus cannot be issued against a private


individual
 Mandamus cannot be issued in the following cases:
 To enforce departmental instruction that does not possess statutory
force
 To order someone to work when the kind of work is discretionary and
not mandatory
 To enforce a contractual obligation
 Mandamus can‘t be issued against the Indian President or State
Governors
 Against the Chief Justice of a High Court acting in a judicial capacity

Prohibition
The literal meaning of ‗Prohibition‘ is ‗To forbid.‘ A court that is higher in
position issues a Prohibition writ against a court that is lower in position to prevent
the latter from exceeding its jurisdiction or usurping a jurisdiction that it does not
possess. It directs inactivity.
Facts about Prohibition in India:

 Writ of Prohibition can only be issued against judicial and quasi-judicial


authorities.
 It can‘t be issued against administrative authorities, legislative bodies and
private individuals or bodies.
Certiorari
The literal meaning of the writ of ‗Certiorari‘ is ‗To be certified‘ or ‗To be
informed.‘ This writ is issued by a court higher in authority to a lower court or
tribunal ordering them either to transfer a case pending with them to itself or quash
their order in a case. It is issued on the grounds of an excess of jurisdiction or lack
of jurisdiction or error of law. It not only prevents but also cures for the mistakes in
the judiciary.
Facts about Certiorari in India:

Pre-1991: The writ of Certiorari used to be issued only against judicial and
quasi-judicial authorities and not against administrative authorities
 Post-1991: The Supreme Court ruled that the certiorari can be issued even
against administrative authorities affecting the rights of individuals
 It cannot be issued against legislative bodies and private individuals or
bodies.
Quo-Warranto
The literal meaning of the writ of ‗Quo-Warranto‘ is ‗By what authority or
warrant.‘ Supreme Court or High Court issue this writ to prevent illegal usurpation
of a public office by a person. Through this writ, the court enquires into the legality
of a claim of a person to a public office
Facts about Quo-Warranto in India:

 Quo-Warranto can be issued only when the substantive public office of a


permanent character created by a statute or by the Constitution is involved
 It can‘t be issued against private or ministerial office
Note: This writ gives the right to seek redressal to any individual other than the
aggrieved person.
General Facts about Writs in India:

 Article 32 also empowers Parliament to authorize any other court to issue


these writs
 Before 1950, only the High Courts of Calcutta, Bombay and Madras had the
power to issue the writs
 Article 226 empowers all the High Courts of India to issue the writs
 Writs of India are borrowed from English law where they are known as
‗Prerogative writs‘

3. Appellate Jurisdiction

As mentioned earlier, the Supreme Court has not only succeeded the Federal Court
of India but also replaced the British Privy Council as the highest court of appeal.
The Supreme Court is primarily a court of appeal and hears appeals against the
judgements of the lower courts. It enjoys a wide appellate jurisdiction which can
be classified under four heads:

(a) Appeals in constitutional matters.

(b) Appeals in civil matters.

(c) Appeals in criminal matters.

(d) Appeals by special leave.

(a) Constitutional Matters: In the constitutional cases, an appeal can be made to the
Supreme Court against the judgement of a high court if the high court certifies that
the case involves a substantial question of law that requires the interpretation of the
Constitution. Based on the certificate, the party in the case can appeal to the
Supreme Court on the ground that the question has been wrongly decided.

(b) Civil Matters: In civil cases, an appeal lies to the Supreme Court from any
judgement of a high court if the high court certifies—

(i) that the case involves a substantial question of law of general importance; and

(ii) that the question needs to be decided by the Supreme Court.

Originally, only those civil cases that involved a sum of `20,000 could be appealed
before the Supreme Court. But this monetary limit was removed by the 30th
Constitutional Amendment Act of 1972.

(c) Criminal Matters The Supreme Court hears appeals against the judgement in a
criminal proceeding of a high court if the high court—

(i) has on appeal reversed an order of acquittal of an accused person and sentenced
him to death; or

(ii) has taken before itself any case from any subordinate court and convicted the
accused person and sentenced him to death; or

(iii) certifies that the case is a fit one for appeal to the Supreme Court.

In the first two cases, an appeal lies to the Supreme Court as a matter of right (ie,
without any certificate of the high court). But if the high court has reversed the
order of conviction and has ordered the acquittal of the accused, there is no right to
appeal to the Supreme Court.

In 1970, the Parliament had enlarged the Criminal Appellate Jurisdiction of the
Supreme Court.Accordingly, an appeal lies to the Supreme Court from the
judgement of a high court if the high court:

(i) has on appeal, reversed an order of acquittal of an accused person and sentenced
him to imprisonment for life or for ten years; or

(ii) has taken before itself any case from any subordinate court and convicted the
accused person and sentenced him to imprisonment for life or for ten years.

Further, the appellate jurisdiction of the Supreme Court extends to all civil and
criminal cases in which the Federal Court of India had jurisdiction to hear appeals
from the high court but which are not covered under the civil and criminal
appellate jurisdiction of the Supreme Court mentioned above.

(d) Appeal by Special Leave The Supreme Court is authorised to grant in its
discretion special leave to appeal from any judgement in any matter passed by any
court or tribunal in the country (except military tribunal and court martial). This
provision contains the four aspects as under:

(i) It is a discretionary power and hence, cannot be claimed as a matter of right.

(ii) It can be granted in any judgement whether final or interlocutory.

(iii) It may be related to any matter—constitutional, civil, criminal, income-tax,


labour, revenue, advocates, etc.

(iv) It can be granted against any court or tribunal and not necessarily against a
high court (of course, except a military court).

Thus, the scope of this provision is very wide and it vests the Supreme Court with
a plenary jurisdiction to hear appeals. On the exercise of this power, the Supreme
Court itself held that ‗being an exceptional and overriding power, it has to be
exercised sparingly and with caution and only in special extraordinary situations.
Beyond that it is not possible to fetter the exercise of this power by any set formula
or rule‘.

4. Advisory Jurisdiction
The Constitution (Article 143) authorizes the president to seek the opinion of the
Supreme Court in the two categories of matters:

(a) On any question of law or fact of public importance which has arisen or which
is likely to arise.

(b) On any dispute arising out of any pre-constitution treaty, agreement, covenant,
engagement, sanad or other similar instruments.

In the first case, the Supreme Court may tender or may refuse to tender its opinion
to the president. But, in the second case, the Supreme Court ‗must‘ tender its
opinion to the president. In both the cases, the opinion expressed by the Supreme
Court is only advisory and not a judicial pronouncement. Hence, it is not binding
on the president; he may follow or may not follow the opinion. However, it
facilitates the government to have an authoritative legal opinion on a matter to be
decided by it.

5. A Court of Record

As a Court of Record, the Supreme Court has two powers:

(a) The judgements, proceedings and acts of the Supreme Court are recorded for
perpetual memory and testimony. These records are admitted to be of evidentiary
value and cannot be questioned when produced before any court. They are
recognised as legal precedents and legal references.

(b) It has power to punish for contempt of court, either with simple imprisonment
for a term up to six months or with fine up to `2,000 or with both. In 1991, the
Supreme Court has ruled that it has power to punish for contempt not only of itself
but also of high courts, subordinate courts and tribunals functioning in the entire
country.

6. Power of Judicial Review

Judicial review is the power of the Supreme Court to examine the constitutionality
of legislative enactments and executive orders of both the Central and state
governments. On examination, if they are found to be violative of the Constitution
(ultra-vires), they can be declared as illegal, unconstitutional and invalid (null and
void) by the Supreme Court. Consequently, they cannot be enforced by the
Government.
Judicial review is needed for the following reasons:

(a) To uphold the principle of the supremacy of the Constitution.

(b) To maintain federal equilibrium (balance between Centre and states).

(c) To protect the fundamental rights of the citizens.

High Court

In the Indian single integrated judicial system, high court operates below the
Supreme Court but above the Sub-ordinate courts. The judiciary in the state
consists of High Court and hierarchy of sub-ordinate courts. At present, there are
24 high courts in the country. Out of them, three are common high courts. Delhi is
the only union territory that has a high court of its own (since 1966). The other
union territories fall under the jurisdiction of different state high courts. The
Parilament can extend the jurisdiction of a high court to any union territory or
exclude the jurisdiction of a high court from any union territory.

Articles 214 to 231 in Part VI of the Constitution deal with the organisation,
independence, jurisdiction, powers, procedures and so on of the high courts.

Jurisdiction and Powers of High Court

Like the Supreme Court, the high court has been vested with quite extensive and
effective powers. It is the highest court of appeal in the state. It is the protector of
the Fundamental Rights of the citizens. It is vested with the power to interpret the
Constitution. Besides, it has supervisory and consultative roles.

At present, a high court enjoys the following jurisdiction and powers:

1. Original jurisdiction.

2. Writ jurisdiction.

3. Appellate jurisdiction.

4. Supervisory jurisdiction.

5. Control over subordinate courts.

6. A court of record.
7. Power of judicial review.

1. Original Jurisdiction

It means the power of a high court to hear disputes in the first instance, not by way
of appeal. It extends to the following:

(a) Matters of admirality, will, marriage, divorce, company laws and contempt of
court.

(b) Disputes relating to the election of members of Parliament and state


legislatures.

(c) Regarding revenue matter or an act ordered or done in revenue collection.

(d) Enforcement of fundamental rights of citizens.

(e) Cases ordered to be transferred from a subordinate court involving the


interpretation of the Constitution to its own file.

(f) The four high courts (i.e., Calcutta, Bombay, Madras and Delhi High Courts)
have original civil jurisdiction in cases of higher value.

2. Writ Jurisdiction

Article 226 of the Constitution empowers a high court to issue writs including
habeas corpus, mandamus, certiorari, prohibition and quo-warrento for the
enforcement of the fundamental rights of the citizens and for any other purpose.
The phrase ‗for any other purpose‟ refers to the enforcement of an ordinary legal
right. The high court can issue writs to any person, authority and government not
only within its territorial jurisdiction but also outside its territorial jurisdiction if
the cause of action arises within its territorial jurisdiction.

The writ jurisdiction of the high court (under Article 226) is not exclusive but
concurrent with the writ jurisdiction of the Supreme Court (under Article 32). It
means, when the fundamental rights of a citizen are violated, the aggrieved party
has the option of moving either the high court or the Supreme Court directly.
However, the writ jurisdiction of the high court is wider than that of the
Supreme Court. This is because, the Supreme Court can issue writs only for the
enforcement of fundamental rights and not for any other purpose, that is, it does
not extend to a case where the breach of an ordinary legal right is alleged.
3. Appellate Jurisdiction

A high court is primarily a court of appeal. It hears appeals against the judgements
of subordinate courts functioning in its territorial jurisdiction. It has appellate
jurisdiction in both civil and criminal matters. Hence, the appellate jurisdiction of a
high court is wider than its original jurisdiction.

(a) Civil Matters The civil appellate jurisdiction of a high court is as follows:

(i) First appeals from the orders and judgements of the district courts, additional
district courts and other subordinate courts lie directly to the high court, on both
questions of law and fact, if the amount exceeds the stipulated limit.

(ii) Second appeals from the orders and judgements of the district court or other
subordinate courts lie to the high court in the cases involving questions of law only
(and not questions of fact).

(iii) The Calcutta, Bombay and Madras High Courts have provision for intra-court
appeals. When a single judge of the high court has decided a case (either under the
original or appellate jurisdiction of the high court), an appeal from such a decision
lies to the division bench of the same high court.

(iv) Appeals from the decisions of the administrative and other tribunals lie to the
division bench of the state high court. In 1997, the Supreme Court ruled that the
tribunals are subject to the writ jurisdiction of the high courts. Consequently, it is
not possible for an aggrieved person to approach the Supreme Court directly
against the decisions of the tribunals, without first going to the high courts.

(b) Criminal Matters:The criminal appellate jurisdiction of a high court is as


follows:

(i) Appeals from the judgements of sessions court and additional sessions court lie
to the high court if the sentence is one of imprisonment for more than seven years.
It should also be noted here that a death sentence (popularly known as capital
punishment) awarded by a sessions court or an additional sessions court should be
confirmed by the high court before it can be executed, whether there is an appeal
by the convicted person or not.

(ii) In some cases specified in various provisions of the Criminal Procedure Code
(1973), the appeals from the judgements of the assistant sessions judge, metro-
politian magistrate or other magistrates (judicial) lie to the high court.
4. Supervisory Jurisdiction :A high court has the power of superintendence over all
courts and tribunals functioning in its territorial jurisdiction (except military courts
or tribunals). Thus, it may—

(a) call for returns from them; (b) make and issue, general rules and prescribe
forms for regulating the practice and proceedings of them; (c) prescribe forms in
which books, entries and accounts are to be kept by them; and (d) settle the fees
payable to the sheriff, clerks, officers and legal practitioners of them.

This power of superintendence of a high court is very broad because, (i) it extends
to all courts and tribunals whether they are subject to the appellate jurisdiction of
the high court or not; (ii) it covers not only administrative superintendence but also
judicial superintendence; (iii) it is a revisional jurisdiction; and (iv) it can be suo-
motu (on its own) and not necessarily on the application of a party.

However, this power does not vest the high court with any unlimited authority over
the subordinate courts and tribunals. It is an extraordinary power and hence has to
be used most sparingly and only in appropriate cases. Usually, it is limited to, (i)
excess of jurisdiction, (ii) gross violation of natural justice, (iii) error of law, (iv)
disregard to the law of superior courts, (v) perverse findings, and (vi) manifest
injustice.

5. Control over Subordinate Courts

In addition to its appellate jurisdiction and supervisory jurisdiction over the


subordinate courts as mentioned above, a high court has an administrative control
and other powers over them. These include the following:

(a) It is consulted by the governor in the matters of appointment, posting and


promotion of district judges and in the appointments of persons to the judicial
service of the state (other than district judges).

(b) It deals with the matters of posting, promotion, grant of leave, transfers and
discipline of the members of the judicial service of the state (other than district
judges).

(c) It can withdraw a case pending in a subordinate court if it involves a substantial


question of law that require the interpretation of the Constitution. It can then either
dispose of the case itself or determine the question of law and return the case to the
subordinate court with its judgement.
(d) Its law is binding on all subordinate courts functioning within its territorial
jurisdiction in the same sense as the law declared by the Supreme Court is binding
on all courts in India.

6. A Court of Record: As a court of record, a high court has two powers:

(a) The judgements, proceedings and acts of the high courts are recorded for
perpetual memory and testimony. These records are admitted to be of evidentiary
value and cannot be questioned when produced before any subordinate court. They
are recognised as legal precedents and legal references.

(b) It has power to punish for contempt of court, either with simple imprisonment
or with fine or with both.

7. Power of Judicial Review

Judicial review is the power of a high court to examine the constitutionality of


legislative enactments and executive orders of both the Central and state
governments. On examination, if they are found to be violative of the Constitution
(ultra-vires), they can be declared as illegal, unconstitutional and invalid (null and
viod) by the high court. Consequently, they cannot be enforced by the government.

Though the phrase ‗judicial review‘ has nowhere been used in the Constitution, the
provisions of Articles 13 and 226 explicitly confer the power of judicial review on
a high court. The constitutional validity of a legislative enactment or an executive
order can be challenged in a high court on the following three grounds:

(a) It infringes the fundamental rights (Part III),

(b) It is outside the competence of the authority which has framed it, and

(c) It is repugnant to the constitutional provisions.

Subordinate Courts in India

The subordinate courts, at the level of districts and lower levels, have almost
similar structure all over the country with slight variation. They deal with civil and
criminal cases in accordance with their respective jurisdictions. At the lowest
stage, the two branches of judicial system, civil and criminal, are bifurcated.
The Munsiff's Courts which are the lowest civil courts have jurisdiction over
claims from Rs. 1,000 upto Rs. 5,000 (in case of some specially empowered cases).
Above the Munsiffs are Subordinate Judges. The District Judge hears first
appeals from the decisions of Subordinate Judges and also from the Munsiffs
(unless they are transferred to a Subordinate Judge) and he possesses unlimited
original jurisdiction, both civil and criminal. Suits of a small value are tried by the
Provincial Small Causes Courts.

The District Judge is the highest judicial authority (civil and criminal) in the
district since the enactment of the Code of Criminal Procedure, 1973 (CrPC). The
criminal trials are conducted exclusively by Judicial Magistrates, except in
Jammu & Kashmir and Nagaland, to which the Cr PC does not apply. The Chief
Judicial Magistrate is the head of the criminal courts in a district. In metropolitan
areas, there are Metropolitan Magistrates.

The Civil Court Structure :

As a matter of fact, every suit should be instituted before the court of lowest
jurisdiction. In the civil side the Munsiff's Court is the court of lowest
jurisdiction. If the value of the subject matter of the suit is worth rupees one lakh or
below, the Munsiff's Court is the competent court to try the suit. If the value
exceeds above rupees one lakh the suit should be filed before the Subordinate
Judge's Court (Sub Court).

An appeal from the decision of the Munsiff court is filed before the District
Court. Appeals from the decisions of the Sub Court are filed before the District
Court if the subject matter of the suit is valued up to rupees two lakhs. If the value
is above rupees two lakhs, the appeal should be filed before the High Court and
next to the Supreme Court. An appeal shall lie to the Supreme Court from any
judgment, decree or final order in a civil proceeding of a High Court in the
territory of India if the High Court certifies-(a) that the case involves a substantial
question of law of general importance; and (b) that in the opinion of the High
Court the said question needs to be decided by the Supreme Court.

Hierarchy of The courts are :

Supreme Court

High Court

Metropolitan Level
Civil Side: City civil courts and courts of Small causes
Criminal Court: Session court, Metropolitan and Magistrate‘s court

District Level

Civil Side: District Court, Civil judge [Senior Division and junior division]

Criminal Side: Session Court and Judicial Magistrate.

Tribunals:

1. Central administrative Tribunal


2. Income Tax
3. Motor Accident Claims
4. Co-operative
5. Rent Control
6. National Green Tribunal

Industrial Court (Both city and district level) and Family Courts

Criminal Courts: Administration of criminal justice is carried out through


Magistrate Courts and Sessions Courts. The hierarchy of criminal courts is given
below.

Categories of Criminal Courts in India

Courts of Session:

As per Section 9 of CrPC, the court is established by the State Government for
every session‘s division. The court is presided over by a Judge, appointed by the
High Court of that particular state. The High Court may also appoint Additional
Sessions Judges and Assistant Sessions Judges in this court. It has the power to
impose any sentence including capital punishment.

Courts of Judicial Magistrates

Section 11 of CrPC states that in every district (not being a metropolitan area),
there shall be established as many Courts of Judicial Magistrates of the first class
and of the second class andat such places, as the State Government may after
consultation with the High Court, by notification specify. Courts of Judicial
Magistrate of First Class are at the second lowest level of the Criminal Court
structure in India. According to Section 15 of the CrPC, a Judicial Magistrate is
under the general control of the Sessions Judge and is subordinate to the Chief
Judicial Magistrate. In terms of Section 29 of the CrPC, a Judicial Magistrate of
First Class may pass a sentence of imprisonment for a term not exceeding three
years, or of fine not exceeding five thousand rupees or of both.

Chief Judicial Magistrate andAdditional Chief Judicial Magistrate, etc.

In every district (not being a metropolitan area), the High Court shall appoint a
Judicial Magistrate of the First Class to be the Chief Judicial Magistrate. A Chief
Judicial Magistrate may impose a sentence except (a) sentence of death, (b)
imprisonment of life, or (c) imprisonment for a term exceeding seven years. A
Chief Judicial Magistrate shall be subordinate to the Sessions Judge; and every
other Judicial Magistrate shall, subject to the general control of the Sessions Judge,
be subordinate to the Chief Judicial Magistrate.

Metropolitan Magistrates

The Courts of Metropolitan Magistrates were created by Section 16 of the


Criminal Procedure Code. The Court of Chief Metropolitan Magistrate and those
of The Additional Chief Metropolitan Magistrates were created by Section 17 of
the Code. Section 18 of the Code also provided for Special Metropolitan
Magistrates. The towns having population exceeding one million could be declared
as Metropolitan Areas. A Metropolitan magistrate is under the general control of
the Sessions Judge and is subordinate to the Chief Metropolitan Magistrate.

Executive Magistrates

In every district and in every metropolitan area, the State Government may appoint
as many persons as it thinks fit to be Executive Magistrates and shall appoint one
of them to be the District Magistrate. The State Government may appoint any
Executive Magistrate to be an Additional District Magistrate and such Magistrate
shall have such of the powers of a District Magistrate under this Code or under any
other law for the time being in force as may be directed by the State Government.

Special Executive Magistrates

Under Section 21 of the CrPC, the State Government may appoint, Executive
Magistrates, to be known as Special Executive Magistrates, for particular areas or
for the performance of particular functions and confer on such Special Executive
Magistrates such of the powers as are conferrable under this Code on Executive
Magistrates, as it may deem fit.

The Court at the lowest level is called Judicial Magistrate of the Second Class.
This Court is competent to try the case if the offence is punishable with
imprisonment for a term not exceeding one year, or with fine not exceeding five
thousand rupees, or with both.

The First Class Magistrate is competent to try offences punishable with


imprisonment for a term not exceeding three years or with fine up to ten
thousand rupees. In States such as Kerala, the Second and the First Class
Magistrate Courts have been unified.

The Chief Judicial Magistrate can impose any fine and impose punishment up to
seven years of imprisonment.

The Assistant Sessions Judge is competent to impose punishments up to ten


years imprisonment and impose any fine.

The Sessions Judge can impose any punishment authorized by law, but the
sentence of death passed by him should be subject to the confirmation by the
High Court. (See for details Sections 28 and 29 of CrPC).

Legal Sevice & Lok Adalat

(Q) Right to legal aid as a constitutional Mandate [2015(6)(b)]

(Q) Evolution of free legal aid in India. [2019(3)(c)]

(Q) ―Legal service and Legal Aid schemes were devised to effectively recognize
the constitutional mandate that economic incapacity shall not prevent people‘s
access to justice.‖ Explain the statement by throwing light to the relevance of Lok
Adalat as an effective tool of amicable dispute resolution. [2015(7)]
(Q) Discuss the advantage and disadvantages of Lok Adalats as means of
dispensation of justice. What measures would you suggest to make them more
efficacious? [2018(2)(a)]

(Q)Briefly state the constitution, functions & power of the Lok Adalats in India.
[2017(2)(b)]

(Q )Overview of the system of Lok Adalat and development of the doctrine of


Public Interest Litigation in India. [2016(3)(b)]

Answer to Question [2015(6)(b)], [2019(3)(c)], [2015(7)] :

Introduction to Legal Services :

Legal aid is the provision of assistance to people otherwise unable to afford legal
representation and access to the court system. Legal aid is regarded as central in
providing access to justice by ensuring equality before the law, the right to counsel
and the right to a fair trial. A number of delivery models for legal aid have
emerged, including duty lawyers, community legal clinics and the payment of
lawyers to deal with cases for individuals who are entitled to legal aid.

The Concept of Legal Aid

The concept of legal aid is in the form of Article 39A into our constitutional
framework. Hence, legal aid is not a charity or bounty, but is a constitutional
obligation of the state and right of the citizens. The problems of human law and
justice, guided by the constitutional goals to the solution of disparities, agonies,
despairs, and handicaps of the weaker, yet larger brackets of Bharat‟s humanity is
the prime object of the dogma of ―equal justice for all‖. Thus, legal aid strives to
ensure that the constitutional pledge is fulfilled in its letter and spirit and equal
justice is made available to the downtrodden and weaker sections of the society. It
is the duty of the State to see that the legal system promotes justice on the basis of
equal opportunity for all its citizens. It must therefore arrange to provide free legal
aid to those who cannot access justice due to economic and other disabilities.

Since 1952, the Govt. of India also started addressing to the question of legal aid
for the poor in various conferences of Law Ministers and Law Commissions. In
1960, some guidelines were drawn by the Govt. for legal aid schemes. In different
states legal aid schemes were floated through Legal Aid Boards, Societies and Law
Departments.

In 1980, a Committee at the national level was constituted to oversee and supervise
legal aid programmes throughout the country under the Chairmanship of Hon. Mr.
Justice P.N. Bhagwati then a Judge of the Supreme Court of India. This Committee
came to be known as CILAS (Committee for Implementing Legal Aid Schemes)
and started monitoring legal aid activities throughout the country. The introduction
of Lok Adalats added a new chapter to the justice dispensation system of this
country and succeeded in providing a supplementary forum to the litigants for
conciliatory settlement of their disputes. In 1987 Legal Services Authorities Act
was enacted to give a statutory base to legal aid programs throughout the country
on a uniform pattern. This Act was finally enforced on 9th of November 1995 after
certain amendments were introduced therein by the Amendment Act of 1994.

Contributions Made By Justice V.R.Krishna Iyer:

The contribution of justice Krishna Iyer towards the development and


incorporation of the concept of legal aid in the Indian legal system has been
tremendous. His report titled Processionals justice to poor‟ has gone a step further
in enabling the recognition of the poor for the purpose of giving legal aid. Justice
Krishna Iyer was appointed as the Chairman of Committee for Legal Aid. The
Committee was formulated as on the 22nd day of October 1972. The Committee
after conducting sample surveys of large part of the country submitted a 275 page
report to the Government on the 27th day of May, 1973. This report came to mark
the cornerstone of Legal Aid development in India. The report clearly laid down
that it is a democratic obligation of the State towards its subject to ensure that the
legal system becomes an effective tool in helping secure the ends of social justice.

History of Legal Aid :

Article 39A of the Constitution of India provides that State shall secure that the
operation of the legal system promotes justice on a basis of equal opportunity, and
shall in particular, provide free legal aid, by suitable legislation or schemes or in
any other way, to ensure that opportunities for securing justice are not denied to
any citizen by reason of economic or other disability.
Articles 14,21 and 22(1) also make it obligatory for the State to ensure equality
before law and a legal system which promotes justice on a basis of equal
opportunity to all. Legal aid strives to ensure that constitutional pledge is fulfilled
in its letter and spirit and equal justice is made available to the poor, downtrodden
and weaker sections of the society.

Sec. 304, Criminal Procedure Code: The Constitutional duty to provide legal aid
arises from the time the accused is produced before the Magistrate for the first time
and continues whenever he is produced for remand.

Since 1952, the Govt. of India also started addressing to the question of legal aid
for the poor in various conferences of Law Ministers and Law Commissions. In
1960, some guidelines were drawn by the Govt. for legal aid schemes. In different
states legal aid schemes were floated through Legal Aid Boards, Societies and Law
Departments.

In 1980, a Committee at the national level was constituted to oversee and supervise
legal aid programmes throughout the country under the Chairmanship of Hon. Mr.
Justice P.N. Bhagwati then a Judge of the Supreme Court of India. This Committee
came to be known as CILAS (Committee for Implementing Legal Aid Schemes)
and started monitoring legal aid activities throughout the country.

The introduction of Lok Adalats added a new chapter to the justice dispensation
system of this country and succeeded in providing a supplementary forum to the
litigants for conciliatory settlement of their disputes. In 1987 Legal Services
Authorities Act was enacted to give a statutory base to legal aid programmes
throughout the country on a uniform pattern. This Act was finally enforced on 9th
of November 1995 after certain amendments were introduced therein by the
Amendment Act of 1994.

Justice Krishna Iyer was appointed as the Chairman of Committee for Legal
Aid. The Committee was formulated as on the 22nd day of October 1972. The
Committee after conducting sample surveys of large part of the country submitted
a 275 page report to the Government on the 27th day of May, 1973. This report
came to mark the cornerstone of Legal Aid development in India. The report
clearly laid down that it is a democratic obligation of the State towards its subject
to ensure that the legal system becomes an effective tool in helping secure the ends
of social justice. He coined the word "Juridicare" to cover a scheme of legal aid
which brought justice to the doorstep of the lowly and which was comprehensive
in its coverage.

Justice Krishna Iyer rightly observed that, "Such a consummation, a proposition to


which we are Constitutionally dedicated is possible only through an activist
scheme of legal aid, conceived wisely and executed vigorously." He went on to
state that Law and Justice cannot be regarded as two separate wings any longer and
that it had become necessary that they in unison work towards resurrecting the
faith of the poor man in the legal system by providing him with adequate non-
Governmental as well as Governmental assistance.

The 14th Law Commission Report stated the fact that if laws do not provide for
an equality of opportunity to seek justice to all segments of society the have no
protective value and unless some arrangement is made for providing a poor man
the means to pay Court fee‟s, advocates fees and other incidental costs of
litigation, he is denied an opportunity to seek justice.

On 17th July 1973, Justice P.N. Bhagwati became the judge of the Supreme Court
of India. He was also Chairman of the Legal Aid Committee appointed by the
Government of Gujarat for suggesting ways and means of providing free legal aid
and advice to the poor and weaker section of the community; and also acted as
Chairman of the State Legal Aid Committee for running the Pilot Project of free
Legal Aid and Advice in Gujarat.He worked successfully to build up an elaborate
legal aid programme. He is widely regarded as the originator of India‟s legal aid
programme, including setting up of legal aid camps in rural areas, working with
NGOs, establishing legal aid clinics etc.

There was unanimous decision of the Committee that the State should regard it as
an obligation to provide legal assistance to the poor and indigent. It stated that this
obligation of the State was not merely, socio-economic or political but is also
constitutional by reason of Articles 14 and 22(1}.

Further the report stated that the legislation and rules so made by the government
should not be another piece of legislation made with the reference of any foreign
legislation, as there is a marked difference between socio-economic conditions
prevailing in advanced countries and those prevailing in developing countries like
India.

It also emphasized on having legal aid programmes and that the organization for
effectuating the legal service programme must be responsive to the poor in giving
legal service and must not be mechanical and wooden in its approach. Even after,
such a programme is introduced there must be a continues examination of its utility
and its responsiveness to the poor.

Supreme Court On Legal Aid

The linkage between Article 21 and the right to free legal aid was forged in the
decision in Hussainara Khatoon v. State of Bihar where the court was appalled
at the plight of thousands of undertrials languishing in the jails in Bihar for years
on end without ever being represented by a lawyer. The court declared that "there
can be no doubt that speedy trial, and by speedy trial, we mean reasonably
expeditious trial, is an integral and essential part of the fundamental right to life
and liberty enshrined in Article 21, " The court pointed out that Article 39-A
emphasised that free legal service was an inalienable element of ―reasonable, fair
and just‟ procedure and that the right to free legal services was implicit in the
guarantee of Article 21. In his inimitable style Justice Bhagwati declared:

Legal aid is really nothing else but equal justice in action. Legal aid is in fact the
delivery system of social justice. If free legal services are not provided to such an
accused, the trial itself may run the risk of being vitiated as contravening Article 21
and we have no doubt that every State Government would try to avoid such a
possible eventuality.

It‘s the constitutional right of every accused person who is unable to engage a
lawyer and secure legal services on account of reasons such as poverty, indigence
or incommunicado situation, to have free legal services provided to him by the
State and the State is under a constitutional mandate to provide a free lawyer to
such accused person if the needs of justice so require. If free legal services are not
provided to such an accused, the trial itself may run the risk of being vitiated as
contravening Article 21 and it is hoped that every State Government would try to
avoid such a possible eventuality.

In Khatri & Others v. St. of Bihar & others :

Bhagmati J. observed; Right to free legal aid, just, fail and reasonable procedures
is a fundamental right. It is elementary that the jeopardy to his personal liberty
arises as soon as the person is arrested and is produced before a magistrate for it is
at this stage that he gets the 1st opportunity to apply for bail and obtain his release
as also to resist remain to police or jail custody. This is the stage at which and
accused person needs competent legal advice and representation. No procedure can
be said to be just, fair and reasonable which denies legal advice representation to
the accused at this stage.

In Indira Gandhi v. Raj Narain ::

The Court said: "Rule Of Law is basic structure of constitution of India. Every
individual is guaranteed the give to him under the constitution. No one so condemn
unheard. Equality of justice. There ought to be a violation to the fundamental right
or prerogatives, or privileges, only then remedy go to Court of Law. But also at the
stage when he first is produced before the magistrate. In absence of legal aid, trial
is vitiated.

The National Legal Services Authority (NALSA) has been constituted under the
Legal Services Authorities Act, 1987, to provide free Legal Services to the
weaker sections of the society and to organize Lok Adalats for amicable settlement
of disputes.

In every State, State Legal Services Authority has been constituted to give effect to
the policies and directions of the NALSA and to give free legal services to the
people and conduct Lok Adalats in the State. Hon‟ble the Chief Justice of the
respective High Court who is the Patronin-Chief of the State Legal Services
Authority heads the State Legal Services Authority.

In every District, District Legal Services Authority has been constituted to


implement Legal Services Programmes in the District. The District Legal Services
Authority is situated in the District Courts Complex in every District and chaired
by the District Judge of the respective district.
Section 12 of the Legal Services Authorities Act, 1987 prescribes the criteria for
giving legal services to the eligible persons. Section 12 of the Act reads as under:-
"12.Every person who has to file or defend a case shall be entitled to legal services
under this Act if that person is –

(a) a member of a Scheduled Caste or Scheduled Tribe;

(b) a victim of trafficking in human beings or begar as referred to in Article 23 of


the Constitution;

(c) a woman or a child;

(d) a mentally ill or otherwise disabled person;

(e) a person under circumstances of undeserved want such as being a victim of a


mass disaster, ethnic violence, caste atrocity, flood, drought, earthquake or
industrial disaster; or

(f) an industrial workman; or

(g) in custody, including custody in a protective home within the meaning of


clause (g) of section 2 of the Immoral Traffic (Prevention) Act, 1956 (104 of
1956); or in a juvenile home within the meaning of clause (j) of section 2 of the
Juvenile Justice Act, 1986 (53 of 1986) or in a psychiatric hospital or psychiatric
nursing home within the meaning of clause (g) of section 2 of the Mental Health
Act, 1987 (14 of 1987); or

(h) in receipt of annual income less than rupees nine thousand or such other higher
amount as may be prescribed by the State Govt., if the case is before a court other
than the Supreme Court, and less than rupees twelve thousand or such other higher
amount as may be prescribed by the Central Govt., if the case is before the
Supreme Court."

Legal Services Authorities after examining the eligibility criteria of an applicant


and the existence of a prima facie case in his favour provide him counsel at State
expense, pay the required Court Fee in the matter and bear all incidental expenses
in connection with the case. The person to whom legal aid is provided is not called
upon to spend anything on the litigation once a Legal Services Authority supports
it.
A nationwide network has been envisaged under the Act for providing legal aid
and assistance. National Legal Services Authority is the apex body constituted to
lay down policies and principles for making legal services available under the
provisions of the Act and to frame most effective and economical schemes for
legal services. It also disburses funds and grants to State Legal Services Authorities
and NGOs for implementing legal aid schemes and programmes.

In every State a State Legal Services Authority is constituted to give effect to the
policies and directions of the Central Authority (NALSA) and to give legal
services to the people and conduct Lok Adalats in the State. The Chief Justice of
the State High Court who is its Patron-in-Chief heads state Legal Services
Authority. A serving or retired Judge of the High Court is nominated as its
Executive Chairman.

District Legal Services Authority is constituted in every District to implement


Legal Aid Programmes and Schemes in the District. The District Judge of the
District is its ex-officio Chairman.

Taluk Legal Services Committees are also constituted for each of the Taluk or
Mandal or for group of Taluk or Mandals to coordinate the activities of legal
services in the Taluk and to organise Lok Adalats. A senior Civil Judge operating
within the jurisdiction of the Committee who is its ex-officio Chairman heads
every Taluk Legal Services Committee.

Answer to Question [2016(3)(b)],[2017(2)(b)] :

Lok Adalat

‗Lok Adalat‘ is a system of conciliation or negotiation. It is also known as


‗people‘s court‘. It can be understood as a court involving the people who are
directly or indirectly affected by the dispute or grievance. ‗Lok Adalat‘,
established by the government settles dispute through conciliation and
compromise. The First ‗Lok Adalat‘ was held in Chennai in 1986. ‗Lok Adalat‘
accepts the cases which could be settled by conciliation and compromise and
pending in the regular courts within their jurisdiction.

Need of Lok Adalat :


As we know that justice delayed is justice denied. This statement becomes true
if we see the backlog of pending cases before courts of different hierarchy. It
resulted into delay justicing in India. Mounting arrears of cases has brought
the judiciary and the judicial process at the verge of collapse. In this given state
of affairs the mechanism of Lok Adalats is the only option left with the people
to resort to for availing cheap and speedy justice. Lok Adalats effectively deal
with the magnitude of arrears of cases. ‗Lok Adalat‘ has in view the social goals
of ending bitterness rather than pending disputes restoring peace in the family,
community and locality.
Statutory Foundation of Lok Adalat :

Under Article-39 A of the Constitution of India, the Parliament has enacted the
Legal Services Authorities Act, 1987 with the legislative intent to constitute
various legal services authorities to provide free and competent legal services
to the weaker sections of the society to ensure that opportunities for securing
justice are not denied to any citizen by reason of economic or other disabilities and
to organise ‗Lok Adalats‘ to ensure the operation of the legal system which
promotes justice on the basis of equal opportunity. The Act was passed to
advance the Constitutional mission of social justice by creating legal services
authorities and to organise ‗Lok Adalats‘ to provide cheap and quick justice to
the deprived and the destitute. The Act has conferred statutory status to ‗Lok
Adalats‘ for the first time through the parliamentary legislation, although the
institution had the glorious socio-cultural heritage in India.

Organisation of Lok Adalat :

Every State Authority or District Authority or Supreme Court Legal Services


Committee or every High Court Legal Services Committee or Taluk Legal
Services Committee may organize Lok Adalats. Lok Adalats are to consist of
three members – a sitting or retired judicial officer, a member of the legal
profession (advocate, law officer, and law teacher) and a social worker,
preferably women. The Act and regulations require the secretary of the legal
services authority or committee to associate students, social activists and
voluntary organisations in the community for facilitating the successful conduct
of ‗Lok Adalats‘.
Jurisdiction and Award of Lok Adalat :

Lok Adalats are intended to arrive at compromises and settlements. In doing


so, it has the power of a civil court in summoning and examining witnesses,
discovery of documents, recording of evidence on affidavits and requisitioning
of public records. Further, it is open to Lok Adalats to specify its own procedure
and it is considered judicial proceedings.
Whenever a settlement is reached, an ‗Award‟ is made which is deemed to be
a ‗Decree‟ of a civil court. It is to be written down in simple and clear terms.
No appeal is permissible against such awards which are deemed final. If no
compromise is reached, the same goes back to the court.
Many people saw the Lok Adalats as a measure to divert litigation from formal
courts and tribunals and a convenient strategy to reduce the mounting arrears
of cases in the formal court system. The insurance companies that found the
compensation amounts settled through Lok Adalats in motor accident cases
economically and administratively convenient started opting for the Lok Adalat
in preference to the Tribunals.

The hon‘ble supreme court in the case of P.T. Thomas v. Thomas Job,
:elaborately explained the meaning and benefits of Lok Adalat as follows:
• The "Lok Adalat" is an old form of adjudicating system prevailed in ancient India
• Its validity has not been taken away even in the modern days too.
• The word 'Lok Adalat' means 'People Court'.
• This system is based on Gandhian Principles.
• It is one of the components of Alternative Dispute Resolution System.
• As the Indian Courts are overburdened with the backlog of cases and involve a
lengthy, expensive and tedious procedure.
• The Court takes years together to settle even petty cases.
• Lok Adalat, therefore, provides alternative device for expedious and inexpensive
justice.
• In lok adalat proceedings there are no victors and loosers and thus no enmity.
• Experiment of 'Lok Adalat' as an alternate mode of dispute settlement has come
to be accepted in India, as a viable, economic, efficient and informal one.
• There is no court fee and if court fee is already paid the amount will be refunded
if the dispute is settled at Lok Adalat according to the rules.
In the case of State of West Bengal v. Union of India :, the hon‘ble Supreme
Court clarified the relationship between sections 12 and section 20 of the Act as
follows:
• The power of the trial Court under sub-section (1) of Section 20 in making
reference of any particular case to a Lok Adalat operates independently of Section
12 of the Act.
• The power of the concerned Authority or the Committee under sub-section (2) of
Section 20 organising the Lok Adalat is controlled by the criteria prescribed by
Section 12.
• If parties to dispute refer matter to Lok Adalat then they need not to fall in the
category of people entitled for legal aid as provided in the Act
• If legal services authority refer a case to Lok Adalat then parties associated with
the dispute should only be those who are entitled for free legal aid as per section 12
of the Act.

Permanent Lok Adalat :

There are many reasons for overburdening of Indian Courts. One reason is lack of
personal interest of party in settlement of public utility services disputes, which
reach to the Indian Courts in large number. Government officials involved in
public utility services disputes hardy give any attention to settlement of disputes
quickly. So the need was felt for the establishment of the permanent alternate
dispute resolution mechanism especially for the settlement of public utility services
disputes. This led to the establishment of Permanent Lok Adalat.
The main aim of Permanent Lok Adalat is to settle the public utility disputes
quickly and finally. So it has one additional power of deciding the dispute on merit
if parties fail to arrive at the settlement or compromise. This way it avoids the
chances of delay in settlement of disputes.
―Public Utility Service‖ means and includes:
.Transport service for the carriage of passengers or goods by air, road or water; or
postal, telegraph or telephone service; or
·Supply of power, light or water to the public by any establishment; or
·System of public conservancy or sanitation; or service in hospital or dispensary;
or Insurance service
.Includes any service which the Central Government or the State Government, as
the case may be, may, in the public interest, by notification, declare to be a public
utility service for the purposes of this chapter.

Composition:
· A person who is, or has been, a district judge or additional district judge or has
held judicial office higher in rank than that of a district judge, shall be the
Chairman of the Permanent Lok Adalat;
· Two other persons having adequate experience in public utility service.

.Any party to a dispute may, before the dispute is brought before any court, can
make an application to the Permanent Lok Adalat for the settlement of dispute.
· It does not has jurisdiction in respect of any matter relating to an offence not
compoundable under any law.
· It also has no jurisdiction in the matter where the value of the property in dispute
exceeds ten lakh rupees.
· The Central Government, may, by notification, increase the limit of ten lakh
rupees specified in the second proviso in consultation with the Central Authority.

Provisions like appointment of members from the legal and non-legal background
make the board of adjudicators more efficient for understanding the disputes of
parties and settling their disputes quickly and amicably. It has conciliatory as well
as adjudicatory power but there are enough provisions under the Act to put the
check of the arbitrary exercise of power.
To save the time in settlement of disputes it need not to follow the formal
procedure but it is bound to follow the principle of natural justice. If the arbitrary
act or misconduct is proved on the part of the board of settlement, then they can be
removed from the board and the award passed by such persons will not enjoy the
status of finality. Therefore, it can be said that the establishment of the Permanent
Lok Adalat is a very useful mechanism for settlement of the public utility services
disputes amicably and quickly.
Every award of the Permanent Lok Adalat under this Act made either on merit or
in terms of a settlement agreement is final and binding. It shall not be called in
question in any original suit, application or execution proceeding. It is deemed to
be a decree of a civil court. It is always by a majority of the persons constituting
the Permanent Lok Adalat. The Permanent Lok Adalat may transmit any award
made by it to a civil court having local jurisdiction and such civil court shall
execute the order as if it were a decree made by that court

Public Interest Litigation

One of the recent modes of getting access to justice is by way of filing a Public
Interest Litigation. The term ‗Public Interest’ means the larger interests of the
public, general welfare and interest of the masses and the word ‗Litigation‘ means
‗a legal action including all proceedings therein, initiated in a court of law
with the purpose of enforcing a right or seeking a remedy.‟ Thus, the
expression „Public Interest Litigation‟ means ‗any litigation conducted for the
benefit of public or for removal of some public grievance.’

Now, any public spirited citizen can move/approach the court for the public cause
(in the interests of the public or public welfare) by filing a petition:

1.In the Supreme Court under Article 32 of the Constitution of India;


2. In the High Court under Article 226 of the Constitution of India;
3. In the Court of Magistrate under Section 133 of the Code of Criminal procedure,
1973
The seeds of the concept of public interest litigation were initially sown in India by
Krishna Iyer J., in 1976 in Mumbai Kamgar Sabha vs. Abdul Thai,and was
initiated in Hussainara Khatoon v. State of Bihar, wherein the PIL was filed by
an advocate on the basis of a news item published in the Indian Express,
highlighting the plight of thousands of undertrial prisoners languishing in various
jails in Bihar. These proceedings led to the release of more than 40, 000 undertrial
prisoners. Right to speedy justice emerged as a basic fundamental right, which had
been denied to these prisoners. The same set pattern was adopted in subsequent
cases.

Krishna lyer J., enunciated the reasons for liberalization of the rule of Locus Standi
in Fertilizer Corporation Kamgar Union v. Union of India. The following were
the reasons:

1. Exercise of State power to eradicate corruption may result in unrelated


interference with individuals‘ rights;
2. Social justice warrants liberal judicial review of administrative action;
3. Restrictive rules of standing are antithesis to a healthy system of
administrative action;
4. Activism is essential for participative public justice.

In 1981 Justice P. N. Bhagwati in S. P. Gupta v. Union of India, articulated the


concept of PIL as follows;
―Where a legal wrong or a legal injury is caused to a person or to a determinate
class of persons by reason of violation of any constitutional or legal right or any
burden is imposed in contravention of any constitutional or legal provision or
without authority of law or any such legal wrong or legal injury or illegal burden is
threatened and such person or determinate class of persons by reasons of poverty,
helplessness or disability or socially or economically disadvantaged position
unable to approach the court for relief, any member of public can maintain an
application for an appropriate direction, order or writ in the High Court under
Article 226 and in case any breach of fundamental rights of such persons or
determinate class of persons, in this court under Article 32 seeking judicial redress
for the legal wrong or legal injury caused to such person or determinate class of
persons.‖

Further the PIL or Social Action Litigation was put on a firm foundation by Justice
Bhagwati in the case of People's Union for the Democratic Rights v. Union of
India, wherein he stated;
―It would not be right or fair to expect a person acting probonus public to incur
expenditure out of his bag for going to a lawyer and preparing a regular Writ
petition. In such a case a letter addressed by him can legitimately be regarded as an
appropriate proceeding‖
The Supreme Court has played a crucial role in formulating several principles in
public interest litigation cases over diverse subjects of law. For instance, the
principle of „Absolute Liability‟ was propounded in the Oleum Gas Leak case,
‗Public Trust Doctrine‟ in Kamalnath Case etc., wherein MC Mehta was the
main proponent of these petitions. Further, the Supreme Court gave variety of
guidelines in various cases of public interest litigation. E.g. Ratlam Municipality
Case, Taj Trapezium Case60, Ganga Pollution Case etc. on the subject of
Environmental Law.
Similarly, the Supreme Court in Bandhua Mukti Morcha v. Union of India,
ordered for the release of bonded labourers; in Murli S. Dogra v. Union of India,
banned smoking in public places; in the landmark judgement of Delhi Domestic
Working Women‟s Forum v. Union of India, issued guidelines for rehabilitation
and compensation for the rape on working women; in Vishaka v. State of
Rajasthan , laid down exhaustive guidelines for preventing sexual harassment of
working women in place of their work and in Minerva Mills Ltd v Union of
India the Court held that the; ‗‗harmony and balance between fundamental
rights and directive principles is an essential feature of the basic structure of
the Constitution.‟‟Since then the judiciary has employed Directive Principles to
derive the contents of various Fundamental Rights. In TN Godavaraman
Thirmulpad v. Union of India, the court examined the issue that where a litigant
filing the PIL lacks bonafide, then the court has to decline its examination at the
behest of a person who, in fact is not a public interest litigant and whose
bonafides and credentials are in doubt. Besides the abovementioned decisions In
a recent judgement of the constitution bench in Lalita Kumari v. Govt. of UP,
Lord the Chief Justice of India, P. Sathasivam, while dealing with a writ petition
regarding FIR and anticipatory bail held as under ;

―The underpinnings of compulsory registration of FIR is not only to ensure


transparency in the criminal justice delivery system by also ensure ‗judicial
oversight‘. Section 157(1) deploys the word ‗forthwith‘. Thus any information
received under Section 154(1) or otherwise has to duly informed in the form of a
report to the magistrate. Thus, the commission of a cognizable offence is not only
brought to the knowledge of the investigating agency but also to the subordinate
judiciary…
While registration of FIR is mandatory, arrest of the accused immediately on
registration of FIR is not at all mandatory. In fact, registration of FIR and arrest of
an accused person are two entirely different concepts under the law, and there are
several safeguards available against arrest. Moreover, it is also pertinent to mention
that an accused person also has a right to apply for ‗anticipatory bail‟ under the
provisions of Section 438 of the Code if the conditions mentioned therein are
satisfied. Thus, in appropriate cases, he can avoid the arrest under that provision by
obtaining an order from the court.‖
Besides the abovementioned decisions, the Supreme Court has evolved certain
principles with respect to Public Interest Litigation, which are summarized
hereunder:
• The court in exercise of powers under Arts. 32 and 226 of the Constitution of
India can entertain a petition filed by any interested person on the welfare of the
people who is in a disadvantaged position and thus, not in a position to knock the
doors of the court. The court is constitutionally bound to protect the fundamental
rights of such disadvantaged people so as to direct the state to fulfill its
constitutional promises.

• Issues of public importance, enforcement of fundamental rights of large number


of public vis a vis the constitutional duties and functions of the state, if raised, the
court treat a letter or a telegram as a public interest litigation upon relaxing
procedural laws as also the law relating to pleadings.
• Whenever injustice is meted out to a large number of people, the court will not
hesitate in stepping in. Articles 14 and 21 of the Constitution of India, as well as
the International Conventions on Human Rights provide for reasonable and fair
trial.

• The common rule of locus standi is relaxed so as to enable the court to look into
the grievances complained on behalf of the poor, depraved, illiterate and the
disabled who cannot vindicate the legal wrong or legal injury caused to them for
any violation of any constitutional or legal right.
• When the court is prima facie satisfied about variation of any constitutional right
of a group of people belonging to the disadvantaged category, it may not allow the
state or the government from raising the question as to the maintainability of the
petition.
• Although procedural laws apply on PIL cases but the question as to whether the
principles of res judicata or principles analogous thereto would apply depending
on the nature of the petition as also the facts and circumstances of the case.
• The dispute between two warring groups purely in the realm of private law would
not be allowed to be agitated as public interest litigation.

• However, in an appropriate case, although the petitioner might have moved a


court in his private interest and for redressal of the personal grievances, the court in
furtherance of the public interest may treat it necessary to enquire into the state of
the subject of litigation in the interest of justice.

• The court in special situations may appoint Commission, or other bodies for the
purpose of investigating into the allegations and finding out the facts. It may also
direct management of public institution taken over by such committee. The court
will not ordinarily transgress into a policy. It shall also take utmost care not to
transgress its jurisdiction while purporting to protect the rights of the people being
involved.
• The court would ordinarily not step out of the known areas of judicial review. The
high courts although may pass an order for doing complete justice to the parties; it
does not have a power akin to Article 142 of the Constitution of India.
• Ordinarily the high court should not entertain a writ petition by way of public
interest litigation questioning the constitutionality or validity of a statute or
statutory rule.

Hence, Judicial Activism of courts in determining PILs have been very vibrant
over the years and have played a significant role in providing access to justice,
while also upholding the rule of law.

Answer to Question No [2018(2)(a)] :

Lok Adalat – Advantages, Drawbacks and Solutions


The Lok Adalat is an innovative Indian contribution to the global jurisprudence. It
is one of the efficient alternative dispute resolution mechanisms that have the
potential to provide amicable settlements of differences. It also provides for an
inclusive justice as envisaged by the Constitution of India. However, currently, this
mechanism has lost public trust due to lack of resources and skilled manpower,
differences among judges and lawyers and inefficiency along with the lack of
consideration of public sensitivity. Thus, it is vital for structurally and culturally
reform this mechanism so that it can achieve its original goal.

What is Lok Adalat?


 Lok Adalat (People‘s Court) is one of India‘s alternate dispute resolution
mechanisms.
 It is where the cases that are pending or at the pre-litigation stage in a court
of law are settled.
 This system, based on Gandhian principles, aims to settle disputes
through arbitration at the grass-root level.
What is the legal basis for Lok Adalat?
 Lok Adalats were given the statutory status under the Legal Services
Authorities Act, 1987
 This Act constitutes legal service authorities to provide free legal services to
the weaker sections of the society according to Article 39 A of the Indian
Constitution.
 It also consists of various provisions for settlements of disputes through Lok
Adalat.
 According to this Act, the decision made by the Lok Adalats is deemed to be
a case of a civil court and is final and binding on all parties and no appeal
against such an award lies before any court of law.
 The strict rule of Civil Procedural Court and evidence is not applicable in
Lok Adalat and the discussions and decisions are informal.
 In 2002, amendments were made in the Legal Services Authorities Act, 1987
to make Lok Adalats permanent body to settle disputes related to public
utility services.
 The Centre and state authorities may establish Permanent Lok Adalats for
determining issues pertaining to public utility services, which include transport
service, postal, telegraph or telephone service, the supply of power, light and
water to the public, public sanitation, insurance services and other services as
notified by the Central and State governments.
How did Lok Adalat come to be?
 The concept of Lok Adalats has deep roots in the Indian legal
history and has close allegiance to the culture and perception of justice in
the Indian ethos.
 However, it was not followed for the last few centuries before independence,
especially during the British colonial era.
 Now it has come back to existence to relieve the heavy burden on
courts with pending cases and to give relief to the litigants who are facing a
delay in the justice.
 It has proven to be one the efficient and important alternative dispute
resolution system and is most suited to the Indian environment, culture and
social interests.
th
 The camps of Lok Adalats were first held on 14 March 1982 at Junagadh,
Gujarat. It has now been extended to all of India.
What is the jurisdiction of Lok Adalat?
 Lok Adalat has jurisdiction on any case pending before or any matter falling
within the jurisdiction of and is not brought before any court for which the Lok
Adalat is organised.
 It can deal with cases like compoundable civil, revenue and criminal cases,
motor accident compensation claims, partition claims, damages cases,
matrimonial and family disputes, land-related cases, bounded labour cases,
bank unpaid loan cases, and so on.
 Despite the wide-ranging jurisdiction, the Lok Adalat is not allowed to deal
with the non-compoundable criminal cases under any law. Thus, serious
crimes are kept outside the ambit of Lok Adalats.
What are the levels of Lok Adalat?
State Authority Level:
 The Member Secretary of the State Legal Services Authority organising the
Lok Adalat would constitute benches at this level.
 Each bench would comprise of sitting or retired High Court Judge or a sitting
or retired judicial officer, a member from the legal profession, and a social
worker
 This social worker should be involved in the upliftment of the weaker areas
and must be interested in the implementation of legal services, plans or
projects.
High Court Level:
 The Secretary of the High Court Legal Services Committee constitutes
benches for Lok Adalat at High Court level.
 Each bench will be consist of a sitting or retired judge of the High Court and
any one or both of either a member from the legal profession.
 The social worker involved in the upliftment of the marginalised is also
included as a part of this bench.
District Level:
 At this level, the Secretary of the District Legal Services Authority will
establish benches of Lok Adalat.
 The bench consists of sitting or retired judicial officer and any one or both of
either a member from the legal profession and/or social worker engaged in the
upliftment of the weaker sections of the society.
 This social worker must also be interested in the implementation of legal
services schemes or programmes or a person involved in para-legal activities
in the area and should preferably be a woman.
Taluk Level:
 The Secretary of the Taluk Legal Services Committee establishes benches of
Lok Adalat.
 Each bench consists of a sitting or retired legal officer and any one or both of
either a member from the legal profession as well as a social worker involved
in the upliftment of the weaker areas.
 The social worker must be interested in the execution of the legal services or
should be involved in para-legal exercises of the area, ideally a woman.
What are the types of Lok Adalat?
National Lok Adalat:
 National Level Lok Adalats are held for at regular interim on a single day
throughout the nation, in every one of the courts, from the Supreme Court till
the Taluk Levels, wherein the cases are disposed of in gigantic amounts.
 They are held every two months across the country to dispose of the pending
cases.
 According to the Law Ministry statistics, more than 50 lakh cases are disposed
of annually on an average by these courts
Permanent Lok Adalat:
 It was established according to Section 22 B of the Legal Service Authorities
Act, 1987.
 These are permanent bodies with a Chairman and two members giving an
obligatory pre-litigation system for conciliation and settlement of cases
pertaining to public utility services.
 In these courts, even if there is a failure in reaching settlement, the Permanent
Lok Adalat has the jurisdiction to decide the matter, provided, the dispute does
not relate to any offence.
 The award given by the Permanent Lok Adalat is last and official for every one
of the parties.
 The jurisdiction of Permanent Lok Adalat is up to Rs.10 Lakhs.
Portable Lok Adalats:
 These are set up in different parts of the country to resolve matters by
encouraging resolution of disputes and easing the burden on the formal
judiciary.
 These dispute settlement bodies would travel from one location to another to
resolve differences in an amicable manner.
What are the powers of Lok Adalat?
 Lok Adalat has the power of a civil court under the Code of Civil Procedure.
 It also has the power to summon and enforce the attendance of any witness and
to examine him/her on oath
 It has the power to enforce the discovery and production of any document and
receive evidence on affidavits
 It also has the power to requisition of any public record or document
 It even has the power to specify its own procedure for the determination of any
dispute coming before it
Why do we need Lok Adalat?
 It is a court that provides free justice. If the case is already filed in the regular
court, the fee paid will be refunded if the disputes are settled via Lok Adalat.
This is in consideration to the economically weaker section of the society.
 It allows for amicable settlement for parties as they can directly interact with
the judge even if they are represented by their lawyer. These parties can
explain their stand in the dispute, which is impossible in the normal courts.
 It reduces unwanted delays by doing away the longwinded legal procedures
and formalities and aims for mutual settlements to reduce the chances for the
further need for appeals.
 The procedure followed by Lok Adalat is simple, flexible, non-technical and
informal. There is no need for strict procedural laws like the Civil Procedure
Code and Evidence Act while determining the claims of the parties.
 The lawyers are not essential during the conciliation process of Lok Adalat.
However, they can assist the proceedings by helping parties understand
contentious issues and available alternatives and persuade them to arrive at a
dispute settlement.
 It disposes of cases via collaborative and participatory efforts of lawyers,
social workers, administrative authorities etc., who are actively involved in the
dispute resolutions.
 It significantly reduces the burden of the formal judiciary so that the latter can
deal with more serious cases.
 This mechanism helps spread awareness at the grass-root level about the
fundamental rights and duties mentioned in the numerous social and welfare
legislation.
 It brings justice to the doorsteps of people by organising at various places like
villages, slums, industrial areas, labour colonies etc.
 There is neither a victor nor a vanquished and both the contestants are gainers
and winners.
 This mechanism promotes local unity and secures substantial equity and social
justice.
What are the problems faced by Lok Adalat?
 Currently, Lok Adalat is considered as one of the best alternative disputes
resolution systems in India.
 However, like every other system, it too suffers from a few limitations.
 Though it is true that ―Justice delayed is justice denied‖, it is also true that
a hurried justice is justice buried.
 Thus, speedy resolutions must not impair the rights of parties.
 In reality, Judges are pressured to quickly dispose of the cases for political
gains, leading to limited consideration to the parties‘ rights and needs.
 Lawyers are sometimes reluctant to refer the matter for settlement in Lok
Adalat.
 There are also instances of parties pressurising their lawyers to stick up to
strict procedures of the court.
 Sometimes there are even the cases where a party‘s attorney is unprepared or
not present, which subsequently prevents parties from reaching a settlement.
 There are even instances of antagonism among the lawyers and judges.
 Another major drawback of this mechanism is that the organisation of the Lok
Adalat is mainly based on compromise or settlement between parties. If
the parties do not arrive at a consensus, the case is either returned to the
court of law or the parties are advised to seek a remedy in the court of law. It
leads to unnecessary delays in the dispensation of justice.
 This issue is addressed by the Permanent Lok Adalat.
 However, Permanent Lok Adalat also is faced with few drawbacks.
 There are concerns that the persons appointed for Permanent Lok Adalat will
not have the necessary legal background.
 Currently, specialised tribunals are appointed with the representatives of social
organisations or experts.
 In the case of mechanisms set up to resolve disputes raised by consumers,
members other than Chairman are persons without a legal background.
 There were also instances of members without a legal background in
administrative tribunals. These members only have administrative
experience.
 While deciding the dispute, the provisions of the Code of Civil Procedure and
the Indian Evidence Act will not be applicable. This means that the
determination or decisions will be in a summary manner.
 A decision is possible only if that cases where there exists an element of the
settlement. In such cases, the Permanent Lok Adalat formulates terms of a
possible settlement and gives such terms to the concerned parties for their
observations.
 These observations will be considered based on the evidence produced by the
parties.
 If they do not consent to the settlement, Permanent Lok Adalat shall decide the
dispute.
 However, the decision or the opinion of the Permanent Lok Adalats as to
whether there exist elements of settlement is also a matter that can be
subjected to judicial review under Article 226 of the Indian Constitution.
What can be the way forward?
Awareness:
 Legal literacy and legal aid programmes should be provided for the poor and
the socially and economically marginalised societies.
 Awareness camps must be conducted at the grass-root levels and mass media
can be utilised for this purpose. This is to encourage people to participate in
the proceedings of Lok Adalat voluntarily.
Bringing lawyers on board:
 The quality of legal aid provided by lawyers must be improved.
 The remunerations offered from legal service authorities to lawyers should be
increased so that they are encouraged to provide effective legal assistance to
the needy.
Expanding jurisdiction:
 The jurisdiction of permanent Lok Adalats can be expanded to include areas
like business disputes or conflicts where the public at large are involved and
matters where the government is involved either directly or indirectly.
Cultural & structural reforms:
 Culturally, there should be a balance between a formal and informal forum so
that people are encouraged to seek redressal from Lok Adalats.
 In doing so, these courts can defeat many social injustices in the rural culture
by ―indigenising‖ some protections of the official judicial system.
 There must be an ideal balance between a national vision of rights based on
equality and justice and a local vision of decentralised judicial administration,
local self-rule and popular justice free from artifice and deceit associated with
formal courts.
 Given India‘s diversity, it is better to incorporate the characteristics and
processes of conciliation specific to each locality.
 In order to achieve these, resources must be mobilised to Lok Adalats to allow
them to be held more regularly. Staff, funding and facilities must be provided
for it to be run more effectively and make it more structurally sound.
 Specialists of the concerned disputes can also be incorporated into this
mechanism.
 The social workers must be provided with free legal training so that they can
help the needy from being exploited by the lawyers.
 In order to regain the public confidence on Lok Adalat, the courts should
encourage mandatory referral to this mechanism so that parties can overcome
their prejudice or lack of understanding of the process.
Conclusion:
The system of Lok Adalat is well appreciated for its capability to dispose of the
cases through amicable settlements. Despite this, it should be noted the overall
functioning of the Lok Adalat appears to be appreciable and not remarkable.
Therefore, this mechanism needs to be strengthened to achieve the constitutional
goal of ―equal and social justice‖ to the fullest extent. In doing so, the public‘s
confidence in the judiciary can be regained.

Method of Legal study and Rules of Interpretation

(Q) Explain any four rules of Interpretation of law. [2019(3)(b)]

(Q)What are commonly recognized methods of interpretation of statutes or


legislations? [2017(2)(e)]

(Q)Briefly explain the Rules of interpretation used by the judiciary in interpreting


statutes in India [2016(3)(c)]

(Q) ―Where, by use of clear unequivocal language capable of only one meaning,
anything is enacted by the legislature. It must be enforced howsoever harsh or
absurd the result may be.‖ In the light of the above, explain the literal rule of
construction of statute. [2018(3)(b)]

A statute is a will of legislature conveyed in the form of text. The Constitution of


India does not use the term ‗Statute‘ but it uses the term ‗law‘. ‗Law‘ includes any
ordinance, order, bye-law, rule, regulation, notification, custom or usage having
the force of law. [Article 13 (3) (a) of the constitution].

Interpretation of statute is the process of ascertaining the true meaning of the words
used in a statute. When the language of the statute is clear, there is no need for the
rules of interpretation. But, in certain cases, more than one meaning may be
derived from the same word or sentence. It is therefore necessary to interpret the
statute to find out the real intention of the statute.
Necessity of interpretation would arise only where the language of a statutory
provision is ambiguous, not clear or where two views are possible or where the
provision gives a different meaning defeating the object of the statute.

If the language is clear and unambiguous, no need of interpretation would arise. In


this regard, a Constitution Bench of five Judges of the Supreme Court in R.S.
Nayak v A.R. Antulay, AIR 1984 SC 684 has held:
―… If the words of the Statute are clear and unambiguous, it is the plainest duty of
the Court to give effect to the natural meaning of the words used in the provision.
The question of construction arises only in the event of an ambiguity or the plain
meaning of the words used in the Statute would be self-defeating.‖

Again Supreme Court in Grasim Industries Ltd. v Collector of Customs,


Bombay, (2002)4 SCC 297 has followed the same principle and observed:
―Where the words are clear and there is no obscurity, and there is no ambiguity and
the intention of the legislature is clearly conveyed, there is no scope for court to
take upon itself the task of amending or altering the statutory provisions.‖

Rules of Interpretation
A Rule is a uniform or established course of things. It is that which is prescribed or
laid down as a guide for conduct or action; a governing direction for a specific
purpose; an authoritative enactment; a regulation; a prescription; a precept; as, the
rules of various societies; the rules governing a school; a rule of etiquette or
propriety etc.

It should be remembered that these Rules are Rules of Practice and not Rules of
Law. Without these rules, it would soon become impossible to not only understand
the law but even just to apply it, as new situations are always coming to light
which Parliament and the courts could not have foreseen when the law was
developed.

Aids of Interpretation

Internal aids mean those materials which are available in the statute itself, though
they may not be part of enactment. These internal aids include, long title,
preamble, headings, marginal notes, illustrations, punctuation, proviso, schedule,
transitory provisions, etc. When internal aids are not adequate, court has to take
recourse to External aids. External Aids may be parliamentary material, historical
background, reports of a committee or a commission, official statement, dictionary
meanings, foreign decisions, etc.
B. Prabhakar Rao and others v State of A.P. and others , AIR 1986 SC 120
O.Chennappa, Reddy J. has observed : ―Where internal aids are not forthcoming,
we can always have recourse to external aids to discover the object of the
legislation. External aids are not ruled out. This is now a well settled principle of
modern statutory construction.‖

RULES OF INTERPRETATION
There are certain general principles of interpretation which have been applied by
Courts from time to time. Over time, various methods of statutory construction
have fallen in and out of favour. Some of the better known rules of interpretation
also referred to as the Primary Rules of Interpretation are discussed hereunder.
1. Rule of Literal Interpretation
In construing Statutes the cardinal rule is to construe its provisions Literally and
grammatically giving the words their ordinary and natural meaning. This rule is
also known as the Plain meaning rule. The first and foremost step in the course of
interpretation is to examine the language and the literal meaning of the statute. The
words in an enactment have their own natural effect and the construction of an act
depends on its wording. There should be no additions or substitution of words in
the construction of statutes and in its interpretation. The primary rule is to interpret
words as they are. It should be taken into note that the rule can be applied only
when the meanings of the words are clear i.e. words should be simple so that the
language is plain and only one meaning can be derived out of the statute.
In Municipal board v State transport authority, Rajasthan, the location of a bus
stand was changed by the Regional Transport Authority. An application could be
moved within 30 days of receipt of order of regional transport authority according
to section 64 A of the Motor vehicles Act, 1939. The application was moved after
30 days on the contention that statute must be read as ―30 days from the knowledge
of the order‖. The Supreme Court held that literal interpretation must be made and
hence rejected the application as invalid.
Lord Atkinson stated, ‗In the construction of statutes their words must be
interpreted in their ordinary grammatical sense unless there be something in the
context or in the object of the statute in which they occur or in the circumstances in
which they are used, to show that they were used in a special sense different from
their ordinary grammatical sense.‘

Meaning
To avoid ambiguity, legislatures often include "definitions" sections within a
statute, which explicitly define the most important terms used in that statute. But
some statutes omit a definitions section entirely, or (more commonly) fail to define
a particular term.
The plain meaning rule attempts to guide courts faced with litigation that turns on
the meaning of a term not defined by the statute, or on that of a word found within
a definition itself.

According to the plain meaning rule, absent a contrary definition within the statute,
words must be given their plain, ordinary and literal meaning. If the words are
clear, they must be applied, even though the intention of the legislator may have
been different or the result is harsh or undesirable. The literal rule is what the law
says instead of what the law means.

―Some laws are meant for all citizens (e.g., criminal statutes) and some are meant
only for specialists (e.g., some sections of the tax code). A text that means one
thing in a legal context might mean something else if it were in a technical manual
or a novel. So the plain meaning of a legal text is something like the meaning that
would be understood by competent speakers of the natural language in which the
text was written who are within the intended readership of the text and who
understand that the text is a legal text of a certain type.‖ (Prof. Larry Solum's
Legal Theory Lexicon).

A literal construction would not be denied only because the consequences to


comply with the same may lead to a penalty. The courts should not be over zealous
in searching for ambiguities or obscurities in words which are plain. (Tata
Consultancy Services V. State of A.P. (2005) 1 SCC 308)

Rationale for this Rule

Proponents of the plain meaning rule claim that it prevents courts from taking sides
in legislative or political issues. They also point out that ordinary people and
lawyers do not have extensive access to secondary sources. In probate law the rule
is also favored because the testator is typically not around to indicate what
interpretation of a will is appropriate. Therefore, it is argued, extrinsic evidence
should not be allowed to vary the words used by the testator or their meaning. It
can help to provide for consistency in interpretation.
Criticism of this rule
Opponents of the plain meaning rule claim that the rule rests on the erroneous
assumption that words have a fixed meaning. In fact, words are imprecise, leading
justices to impose their own prejudices to determine the meaning of a statute.
However, since little else is offered as an alternative discretion-confining theory,
plain meaning survives.
This is the oldest of the rules of construction and is still used today, primarily
because judges may not legislate. As there is always the danger that a particular
interpretation may be the equivalent of making law, some judges prefer to adhere
to the law's literal wording.

2. Golden Rule of Interpretation


The Golden rule, or British rule, is a form of statutory interpretation that allows a
judge to depart from a word's normal meaning in order to avoid an absurd result.

It is a compromise between the plain meaning (or literal) rule and the mischief
rule. Like the plain meaning rule, it gives the words of a statute their plain,
ordinary meaning. However, when this may lead to an irrational result that is
unlikely to be the legislature's intention, the judge can depart from this meaning. In
the case of homographs, where a word can have more than one meaning, the judge
can choose the preferred meaning; if the word only has one meaning, but applying
this would lead to a bad decision, the judge can apply a completely different
meaning.
This rule may be used in two ways. It is applied most frequently in a narrow sense
where there is some ambiguity or absurdity in the words themselves.
For example, imagine there may be a sign saying "Do not use lifts in case of fire."
Under the literal interpretation of this sign, people must never use the lifts, in case
there is a fire. However, this would be an absurd result, as the intention of the
person who made the sign is obviously to prevent people from using the lifts only
if there is currently a fire nearby.
The second use of the golden rule is in a wider sense, to avoid a result that is
obnoxious to principles of public policy, even where words have only one
meaning. Example: The facts of a case are; a son murdered his mother and
committed suicide. The courts were required to rule on who then inherited the
estate, the mother's family, or the son's descendants. There was never a question of
the son profiting from his crime, but as the outcome would have been binding on
lower courts in the future, the court found in favour of the mother's family.

3. The Mischief Rule


The mischief rule is a rule of statutory interpretation that attempts to determine the
legislator's intention. Originating from a 16th century case (Heydon‘s case) in the
United Kingdom, its main aim is to determine the "mischief and defect" that the
statute in question has set out to remedy, and what ruling would effectively
implement this remedy. When the material words are capable of bearing two or
more constructions the most firmly established rule or construction of such words
―of all statutes in general be they penal or beneficial, restrictive or enlarging of the
common law is the rule of Heydon‘s case. The rules laid down in this case are also
known as Purposive Construction or Mischief Rule.
The mischief rule is a certain rule that judges can apply in statutory interpretation
in order to discover Parliament's intention. It essentially asks the question: By
creating an Act of Parliament what was the "mischief" that the previous law did not
cover?
Heydon‟s case
This was set out in Heydon's Case [1584] 3 CO REP 7, where it was stated that
there were four points to be taken into consideration when interpreting a statute:
1. What was the common law before the making of the act?
2. What was the "mischief and defect" for which the common law did not provide?
3. What remedy the parliament hath resolved and appointed to cure the disease of
the commonwealth?
4. What is the true reason of the remedy?
The office of all the judges is always to make such construction as shall suppress
the mischief, and advance the remedy, and to suppress subtle inventions and
evasions for continuance of the mischief, and pro privato commodo, and to add
force and life to the cure and remedy, according to the true intent of the makers of
the Act, pro bono publico.
The application of this rule gives the judge more discretion than the literal and the
golden rule as it allows him to effectively decide on Parliament's intent. It can be
argued that this undermines Parliament's supremacy and is undemocratic as it takes
lawmaking decisions away from the legislature.

Use of this Rule


This rule of construction is of narrower application than the golden rule or the
plain meaning rule, in that it can only be used to interpret a statute and, strictly
speaking, only when the statute was passed to remedy a defect in the common law.
Legislative intent is determined by examining secondary sources, such as
committee reports, treatises, law review articles and corresponding statutes. This
rule has often been used to resolve ambiguities in cases in which the literal rule
cannot be applied.
In the case of Thomson vs. Lord Clan Morris, Lord Lindley M.R. stated that in
interpreting any statutory enactment regard must be had not only to the words
used, but also to the history of the Act and the reasons which lead to its being
passed.
In the case of CIT vs. Sundara devi (1957) (32 ITR 615) (SC), it was held by the
Apex Court that unless there is an ambiguity, it would not be open to the Court to
depart from the normal rule of construction which is that the intention of the
legislature should be primarily to gather from the words which are used. It is only
when the words used are ambiguous that they would stand to be examined and
considered on surrounding circumstances and constitutionally proposed practices.
The Supreme Court in Bengal Immunity Co. V. State of Bihar, (AIR 1995 SC
661) applied the mischief rule in construction of Article 286 of the Constitution of
India. After referring to the state of law prevailing in the province prior to the
constitution as also to the chaos and confusion that was brought about in inter-state
trade and commerce by indiscriminate exercise of taxing powers by the different
Provincial Legislatures founded on the theory of territorial nexus, Chief Justice
S.R.Das, stated ―It was to cure this mischief of multiple taxation and to preserve
the free flow of interstate trade or commerce in the Union of India regarded as one
economic unit without any provincial barrier that the constitution maker adopted
Article 286 in the constitution‖.

In various Supreme Court cases it has been held that, ‗legislation both statutory
and constitutional is enacted, it is true, from experience of evils. But its general
language should not, therefore, necessarily be confined to the form that evil had
taken. Time works changes, brings into existence new conditions and purposes and
new awareness of limitations. A principle to be valued must be capable of wider
application than the mischief which gave it existence. This is particularly true of
the constitutional constructions which are not ephermal enactments designed to
meet passing occasions.
These are designed to approach immortality as nearly as human institutions can
approach it‘. Mischief Rule is applicable where language is capable of more than
one meaning. It is the duty of the Court to make such construction of a statue
which shall suppress the mischief and advance the remedy.
Advantages -
1) The Law Commission sees it as a far more satisfactory way of interpreting acts
as opposed to the Golden or Literal rules.
2) It usually avoids unjust or absurd results in sentencing.
Disadvantages -
1) It is considered to be out of date as it has been in use since the 16th century,
when common law was the primary source of law and parliamentary supremacy
was not established.
2) It gives too much power to the unelected judiciary which is argued to be
undemocratic.
3) In the 16th century, the judiciary would often draft acts on behalf of the king
and were therefore well qualified in what mischief the act was meant to remedy.
4) It can make the law uncertain.

4. Rule of Reasonable Construction


Every statute has a purpose, an objective. If the literal meaning collides with the
reason of enactment of the statute then the intention of the law should be taken up
so that the actual meaning of the statute can be properly understood.
This rule mainly stresses upon the intention of the legislature to bring up the statue
and the sensible and not the prima facie meaning of the statute. This helps us clear
the errors caused due to faulty draftsmanship. However this rule also has its own
limitations. The intent of the statute is in itself a surmise and the rule is usually
avoided to complete the quest for interpretation unless the intent in itself can be
interpreted properly.

5. Rule of Harmonious Construction


When there is a conflict between two or more statues or two or more parts of a
statute then the rule of harmonious construction needs to be adopted. The rule
follows a very simple premise that every statute has a purpose and intent as per law
and should be read as a whole. The interpretation consistent of all the provisions of
the statute should be adopted. In the case in which it shall be impossible to
harmonize both the provisions, the court‘s decision regarding the provision shall
prevail.

The rule of harmonious construction is the thumb rule to interpretation of any


statute. An interpretation which makes the enactment a consistent whole, should be
the aim of the Courts and a construction which avoids inconsistency or repugnancy
between the various sections or parts of the statute should be adopted. The Courts
should avoid ―a head on clash‖, in the words of the Apex Court, between the
different parts of an enactment and conflict between the various provisions should
be sought to be harmonized. The normal presumption should be consistency and it
should not be assumed that what is given with one hand by the legislature is sought
to be taken away by the other. The rule of harmonious construction has been
tersely explained by the Supreme Court thus, ―When there are, in an enactment two
provisions which cannot be reconciled with each other, they should be so
interpreted, that if possible, effect should be given to both‖. A construction which
makes one portion of the enactment a dead letter should be avoided since
harmonization is not equivalent to destruction.

It is a settled rule that an interpretation which results in hardship, injustice,


inconvenience or anomaly should be avoided and that which supports the sense of
justice should be adopted. The Court leans in favour of an interpretation which
conforms to justice and fair play and prevents injustice (Union of India vs. B.S.
Aggarwal) (AIR 1998 S.C. 1537).
When there are two provisions in a statute, which are in apparent conflict with each
other, they should be interpreted such that effect can be given to both and that
construction which renders either of them inoperative and useless should not be
adopted except in the last resort.
This principle is illustrated in the case of Raj Krishna vs Binod AIR 1954. In this
case, two provisions of Representation of People Act, 1951, which were in
apparent conflict, were brought forth. Section 33 (2) says that a Government
Servant can nominate or second a person in election but section 123(8) says that a
Government Servant cannot assist any candidate in election except by casting his
vote. The Supreme Court observed that both these provisions should be
harmoniously interpreted and held that a Government Servant was entitled to
nominate or second a candidate seeking election in State Legislative assembly.
This harmony can only be achieved if Section 123(8) is interpreted as giving the
govt. servant the right to vote as well as to nominate or second a candidate
and forbidding him to assist the candidate in any other manner.
The important aspects of this principle are -
1. The courts must avoid a head on clash of seemingly contradicting provisions and
they must construe the contradictory provisions so as to harmonize them.
2. The provision of one section cannot be used to defeat the provision contained in
another unless the court, despite all its effort, is unable to find a way to reconcile
their differences.
3. When it is impossible to completely reconcile the differences in contradictory
provisions, the courts must interpret them in such as way so that effect is given
to both the provisions as much as possible.
4. Courts must also keep in mind that interpretation that reduces one provision to a
useless number or a dead lumbar is not harmonious construction.
5. To harmonize is not to destroy any statutory provision or to render it loose.

6. Rule of Beneficial Construction


When the literal meaning of the statute defeats the objective of the legislature, the
court may depart from the dictionary and instead give it a meaning which will
advance the remedy and suppress the mischief. This supports the initial and
modern approach that is to effectuate the object and purpose of the act. The main
objective by extending the meaning of the statute is to ensure that its initial
purpose (public safety, maintenance of law and order) is justified. This rule looks
into the reasons as per why the statute was initially enacted and promotes the
remedial effects by suppressing the mischief. Though the rule almost covers the
main grounds of the statute but cannot be applied to Fiscal statutes.

For example, in the case of Alembic Chemical Works vs. Workmen AIR 1961,
an industrial tribunal awarded more number of paid leaves to the workers than
what Section 79(1) of Factories Act recommended. This was challenged by the
appellant. The Supreme Court held that the enactment being a welfare legislation
for the workers, had to be beneficially constructed in the favour of worker and
thus, if the words are capable of two meanings, the one that gives benefit to the
workers must be used.
Similarly, in U.Unichoyi vs. State of Kerala, 1963, the question was whether
setting of a minimum wage through Minimum Wages Act, 1948 is violative of
Article 19(1)(g) of the Constitution because the Act did not define what is
minimum wage and did not take into account the capacity of the employer to pay.
It was held that the Act is a beneficial legislation and it must be construed in
favour of the worker. In an under developed country where unemployment is
rampant, it is possible that workers may become ready to work for extremely low
wages but that should not happen

Beneficial Construction is a tendency and not a rule. The reason is that this
principle is based on human tendency to be fair, accommodating, and just. Instead
of restricting the people from getting the benefit of the statute, Court tends to
include as many classes as it can while remaining faithful to the wordings of the
statute.

7. Rule of Exceptional Construction


The rule of exceptional construction stands for the elimination of statutes and
words in a statute which defeat the real objective of the statute or make no sense. It
also stands for construction of words ‗and‘, „or‟, ‗may‘, ‗shall‘ & ‗must‘. While
‗and‘ is normally considered conjunctive so that both provisions of a statute can be
satisfied, ‗or‘ is used of satisfying the clauses or either of the provisions in a
statute. The word ‗may‘ generally has a directory for but is also has a mandatory
force where subject involves discretion coupled with obligation, where the word
‗may‘ has been used in the statute as a matter of pure conventional courtesy and
also where the word ‗may‘ may defeat the objective of the statute. Similarly
‗shall‟ is considered to have a mandatory force and is used in cases of statutes
providing specific penalty. ‗Must‘ on the other hand had a directory force and is
used for statutes against the government or using a mandatory force may result in
absurd results. While this rule seems simple, the draftsmanship lies in deciding
whether the statute should use a mandatory for or a directory force.

8. Rule of Ejusdem Generis


Ejusdem Generis means "of the same kind and nature‖.
When a list of two or more specific descriptors are followed by more general
descriptors, the otherwise wide meaning of the general descriptors must be
restricted to the same class, if any, of the specific words that precede them. In this
rule a specific word, class or species needs to be mentioned so that the whole
statute revolves around it and the statute will be only meant for these specific
words. However the specific words should not have a wide approach as they would
exhaust the whole statute.
This rule provides that where words of specific meaning are followed by general
words, the general words will be construed as being limited to persons or things of
the same general kind or class as those enumerated by the specific words.

The rule can be illustrated by a reference to the decision of the Kerala High Court
in the case of Kerala Cooperative Consumers' Federation Ltd v CIT (1988)
170 ITR 455 (Ker). In this decision, the court was required to interpret the
meaning of the phrase 'Body of Individuals'. It has said that in construing the
words 'Body of Individuals' occurring in section 2(31) of the Income Tax Act
along-side the words 'Association of Persons', the words 'Body of Individuals'
would have to be understood in the same background, context and meaning given
to the words "Association of Persons'.
The Supreme Court in Siddeshwari Cotton Mills (P) Ltd v UOI, AIR 1989 SC
1019, while interpreting the expression 'any other process' appearing along-with
the words 'bleaching, mercerizing, dyeing, printing, water-proofing, rubberizing,
shrink-proofing, organic processing in section 2(f) of the Central Excise & Salt
Act, 1944 (as it stood prior to its substitution by Central Excise Tariff Act, 1985)
read with Notification No 230 and 231 dated 15th July, 1977 with the aid of the
principle of Ejusdem Generis has said that the foregoing words, which precede the
expression 'or any other process' contemplate process, which import a change of a
lasting nature must share one or the other of these incidents.

9. Noscitur a Sociis
Noscitur a Sociis literally means ―It is known from its associates‖. The rule of
language is used by the courts to help interpret legislation. Under the doctrine of
"noscitur a sociis" the questionable meaning of a word or doubtful words can be
derived from its association with other words within the context of the phrase. This
means that words in a list within a statute have meanings that are related to each
other.
In Foster v Diphwys Casson((1887) 18 QBD 428), the case involved a statute
which stated that explosives taken into a mine must be in a "case or canister". Here
the defendant used a cloth bag. The courts had to consider whether a cloth bag was
within the definition. Under noscitur a sociis, it was held that the bag could not
have been within the statutory definition, because parliament's intention in using
‗case or container‘ was referring to something of the same strength as a canister.

10. Expressio Unius Est Exclusio Alterius


The Expression literally means ―the express mention of one thing excludes all
others‖. Where one or more things are specifically included in some list and others
have been excluded it automatically means that all others have been excluded.
However, sometimes a list in a statute is illustrative, not exclusionary. This is
usually indicated by a word such as "includes" or ―such as‖.
Thus a statute granting certain rights to "police, fire, and sanitation employees"
would be interpreted to exclude other public employees not enumerated from the
legislation. This is based on presumed legislative intent and where for some reason
this intent cannot be reasonably inferred the court is free to draw a different
conclusion.

Source of Law

(Q) (a) write a brief note on Legislation or precedent as a source of law.


[2019(4)(a)]

(Q) Legislation as a source of law. [2018(8)(a)]

(Q)Explain Legislation and precedent as the two main sources of law in any
legal system. [2016(4)]

(Q) ―The great importance attached to judicial precedents is a distinguishing


feature of Indian Legal System‖. Explain. [2018(3)(a)]

(Q)How does a judgment of the court become a judicial precedent? Explain,


what are the factors which increased or decreased authority of a judgment?
[2017(3)]
(Q)―A custom to be legally recognizable and enforceable must fulfill several
requisites such as certainty, reasonableness, historical origin, strict
construction etc.‖ Comment on the statement by exploring the importance of
customs in different personal laws in India. [2015(5)]

(Q)Ratio Decidendi and Obiter Dictum [2015(6)(a)]

Answer to Question No [2019(4)(a)],[2016(4)],[2017(3)]:

Judicial Precedent as Source of Law :

In general use, the term ―precedent means some set pattern guiding the future
conduct. In the judicial field, it means the guidance or authority of past decisions
of the courts for future cases. Only such decisions which lay down some new rule
or principle are called judicial precedents.
Judicial precedents are an important source of law. They have enjoyed high
authority at all times and in all countries. This is particularly so in the case of
England and other countries which have been influenced by English jurisprudence.
The principles of law expressed for the first time in court decisions become
precedents to be followed as law in deciding problems and cases identical with
them in future. The rule that a court decision becomes a precedent to be followed
in similar cases is known as doctrine of stare decisis.

In simple words, judicial precedent refers to previously decided judgments of the


superior courts, such as the High Courts and the Supreme Court, which judges are
bound to follow. This binding character of the previously decided cases is
important, considering the hierarchy of the courts established by the legal systems
of a particular country. In the case of India, this hierarchy has been established by
the Constitution of India. Judicial precedent is an important source of law, but it is
neither as modern as legislation nor is it as old as custom. It is an important feature
of the English legal system as well as of other common law countries which follow
the English legal system.
The reason why a precedent is recognised is that a judicial decision is presumed to
be correct. The practice of following precedents creates confidence in the minds of
litigants. Law becomes certain and known and that in itself is a great advantage.
Administration of justice becomes equitable and fair.

In Union of India Vs. Raghubir Singh (AIR 1989 SC 1933) it has been held
―The doctrine of binding precedent has the merit of promoting a certainty and
consistency in judicial decisions, and enables an organic development of the law,
besides providing assurance to the individual as to the consequence of transactions
forming part of daily affairs. And, therefore, the need for a clear and consistent
enunciation of legal principle in the decisions of a Court.‖

High Courts:

(i) The decisions of High Court are binding on all the subordinate courts and
tribunals within its jurisdiction.

The decisions of one High Court have only a persuasive value in a court which is
within the jurisdiction of another High Court. But if such decision is in conflict
with any decision of the High Court within whose jurisdiction that court is situated,
it has no value and the decision of that High Court is binding on the court.

In case of any conflict between the two decisions of co-equal Benches, generally
the later decision is to be followed.

(ii) In a High Court, a single judge constitutes the smallest Bench. A Bench of two
judges is known as Division Bench. Three or more judges constitute a Full Bench.
A decision of such a Bench is binding on a Smaller Bench.

One Bench of the same High Court cannot take a view contrary to the decision
already given by another coordinate Bench of that High Court. Though decision of
a Division Bench is wrong, it is binding on a single judge of the same High Court.

Thus, a decision by a Bench of the High Court should be followed by other


Benches unless they have reason to differ from it, in which case the proper course
is to refer the question for decision by a Full Bench.

(iii) The High Courts are the Courts of co-ordinate jurisdiction. Therefore, the
decision of one High Court is not binding on the other High Courts and have
persuasive value only.
Pre-constitution (1950) Privy Council decisions are binding on the High Courts
unless overruled by the Supreme Court.

(iv) The Supreme Court is the highest Court and its decisions are binding on all
courts and other judicial tribunals of the country. Article 141 of the Constitution
makes it clear that the law declared by the Supreme

Court shall be binding on all courts within the territory of India. The words ―law
declared‖ includes an obiter dictum provided it is upon a point raised and argued
(Bimladevi v. Chaturvedi, AIR 1953 All. 613).

However, it does not mean that every statement in a judgement of the Supreme
Court has the binding effect. Only the statement of ratio of the judgement is having
the binding force.

Supreme Court

The expression ‗all courts‗ used in Article 141 refers only to courts other than the
Supreme Court. Thus, the Supreme Court is not bound by its own decisions.
However, in practice, the Supreme Court has observed that the earlier decisions of
the Court cannot be departed from unless there are extraordinary or special reasons
to do so (AIR 1976 SC 410). If the earlier decision is found erroneous and is thus
detrimental to the general welfare of the public, the Supreme Court will not
hesitate in departing from it.

English decisions have only persuasive value in India. The Supreme Court is not
bound by the decisions of Privy Council or Federal Court. Thus, the doctrine of
precedent as it operates in India lays down the principle that decisions of higher
courts must be followed by the courts subordinate to them. However, higher courts
are not bound by their own decisions (as is the case in England).

Precedents may be classified as:

(i) Declaratory and Original Precedents: According to Salmond, a declaratory


precedent is one which is merely the application of an already existing rule of law.
An original precedent is one which creates and applies a new rule of law. In the
case of a declaratory precedent, the rule is applied because it is already a law. In
the case of an original precedent, it is law for the future because it is now applied.
In the case of advanced countries, declaratory precedents are more numerous. The
number of original precedents is small but their importance is very great. They
alone develop the law of the country. They serve as good evidence of law for the
future. A declaratory precedent is as good a source of law as an original precedent.
The legal authority of both is exactly the same.

(ii) Persuasive Precedents: A persuasive precedent is one which the judges are
not obliged to follow but which they will take into consideration and to which they
will attach great weight as it seems to them to deserve. A persuasive precedent,
therefore, is not a legal source of law; but is regarded as a historical source of law.
Thus, in India, the decisions of one High Court are only persuasive precedents in
the other High Courts. The rulings of the English and American Courts are
persuasive precedents only. Obiter dicta also have only persuasive value.

(iii) Absolutely Authoritative Precedents: An authoritative precedent is one


which judges must follow whether they approve of it or not. Its binding force is
absolute and the judge‗s discretion is altogether excluded as he must follow it.
Such a decision has a legal claim to implicit obedience, even if the judge considers
it wrong. Unlike a persuasive precedent which is merely historical, an authoritative
precedent is a legal source of law.

Absolutely authoritative precedents in India: Every court in India is absolutely


bound by the decisions of courts superior to itself. The subordinate courts are
bound to follow the decisions of the High Court to which they are subordinate. A
single judge of a High Court is bound by the decision of a bench of two or more
judges. All courts are absolutely bound by decisions of the Supreme Court.

In England decisions of the House of Lords are absolutely binding not only upon
all inferior courts but even upon itself. Likewise, the decisions of the Court of
Appeal are absolutely binding upon itself.

(iv) Conditionally Authoritative Precedents : A conditionally authoritative


precedent is one which, though ordinarily binding on the court before which it is
cited, is liable to be disregarded in certain circumstances. The court is entitled to
disregard a decision if it is a wrong one, i.e., contrary to law and reason. In India,
for instance, the decision of a single Judge of the High Court is absolutely
authoritative so far as subordinate judiciary is concerned, but it is only
conditionally authoritative when cited before a Division Bench of the same High
Court.

Doctrine of Stare Decisis :

The doctrine of stare decisis means ―adhere to the decision and do not unsettle
things which are established‖. It is a useful doctrine intended to bring about
certainty and uniformity in the law. Under the stare decisis doctrine, a principle of
law which has become settled by a series of decisions generally is binding on the
courts and should be followed in similar cases. In simple words, the principle
means that like cases should be decided alike. This rule is based on public policy.
Although doctrine should be strictly adhered to by the courts, it is not universally
applicable. The doctrine should not be regarded as a rigid and inevitable doctrine
which must be applied at the cost of justice.

In the case of Commissioner of Income Tax vsM/s Sun Engineering Works


Private Limited AIR 1993, SC 43, the Hon'ble Apex Court held that, ―while
applying the decision to a later cases, the court must carefully try to ascertain the
true principle laid down by the decision of the Supreme Court and not to pick out
words or sentences from the Judgment divorced from the context of question under
consideration by the court to support their reasoning.‖
It is very clear that, only those statements in an earlier decision which may be said
to constitute the ratio decidendi of that case are binding. Statements which are not
essential or necessary for deciding the later cases, such non authoritative
statements are called as obiter dicta.

Answer to Question No [2015(6)(a)]

Ratio Decidendi :

The underlying principle of a judicial decision, which is only authoritative, is


termed as ratio decidendi. The proposition of law which is necessary for the
decision or could be extracted from the decision constitutes the ratio. The concrete
decision is binding between the parties to it. The abstract ratio decidendi alone has
the force of law as regards the world at large. In other words, the authority of a
decision as a precedent lies in its ratio decidendi.
Prof. Goodhart says that ratio decidendi is nothing more than the decision based
on the material facts of the case.

Where an issue requires to be answered on principles, the principles which are


deduced by way of abstraction of the material facts of the case eliminating the
immaterial elements is known as ratio decidendi and such principle is not only
applicable to that case but to other cases also which are of similar nature.

It is the ratio decidendi or the general principle which has the binding effect as a
precedent, and not the obiter dictum. However, the determination or separation of
ratio decidendi from obiter dictum is not so easy. It is for the judge to determine
the ratio decidendi and to apply it on case to be decided.

Kumar v. UOI and others , 1990 4 SSC 207 :

It has been observed the hon'ble supreme court that:

―In other words, the enunciation of the reason or principle upon which a question
before a court has been decided is alone binding as a precedent. The ratio
decidendi is the underlying principle, namely, the general reasons or the general
grounds upon which the decision is based on the test or abstract from the specific
peculiarities of the particular case which gives rise to the decision. The ratio
decidendi has to be ascertained by an analysis of the facts of the case and the
process of reasoning involving the major premise consisting of a pre-existing rule
of law, either statutory or judge-made, and a minor premise consisting of the
material facts of the case under immediate consideration. If it is not clear, it is not
the duty of the court to spell it out with difficulty in order to be bound by it.‖

Orbiter Dicta :

The literal meaning of this Latin expression is ―said by the way. The expression
is used especially to denote those judicial utterances in the course of delivering a
judgement which taken by themselves, were not strictly necessary for the decision
of the particular issue raised. These statements thus go beyond the requirement of a
particular case and have the force of persuasive precedents only. The judges are
not bound to follow them although they can take advantage of them. They some
times help the cause of the reform of law.

Obiter Dicta are of different kinds and of varying degree of weight. Some obiter
dicta are deliberate expressions of opinion given after consideration on a point
clearly brought and argued before the court. It is quite often too difficult for
lawyers and courts to see whether an expression is the ratio of judgement or just a
causal opinion by the judge. It is open, no doubt, to other judges to give a decision
contrary to such obiter dicta.

Mohandas Issardas and others Vs. A.N. Sattanathan & Others, A.I.R. 1995
(Bom.) 113 :

―the court in India should accept as an authoritative pronouncement on the


particular aspect of law and treat that pronouncement as binding. The Supreme
court has now taken the place of privy council and we would like to say
unhesitatingly that we must show the same respect for the 'obiter dicta' of the
Supreme Court that we did for those of privy council. The Supreme Court is the
highest judicial tribunal in India today and it is as much necessary as in the interest
of judicial uniformity and judicial discipline that all the High Courts must accept
as binding the 'obiter dicta' of the Supreme Court in the same spirit as the High
Courts accepted the 'obiter dicta' of the privy council.‖

Do Judges make Law?

The Indian Constitution confers power on the legislature to make law, while the
judiciary has the power to examine the constitutionality of the law enacted by the
legislature. The courts also adjudicate upon the rights and duties of citizens, and
further interpret the provisions of the Constitution and other statutes.

Through these processes, the Courts create new rights for the citizens. By this
exercise, the judiciary makes additions to the existing laws of the country. It is
argued that while doing this, judges actually make law. There are two views
regarding this issue. One set of jurists say that judges do not make the law but that
they simply declare the existing law. Another set of jurists say that judges do make
the law.
Jurists like Edward Coke and Mathew Hale are of the opinion that judges do not
make law. According to them judicial decisions are not sources of law but, they are
simply the proof of what the law is. Judges are not law-givers, but they discover
law. At the same time jurists like Dicey, Gray, Salmond are of the opinion that
judges do make law. They hold the view that judges, while interpreting the law
enacted by the legislative bodies, contribute to the existing body of law. A large
part of the English law is judge-made law.
The above arguments seem to be complementary. It can be inferred that judges do
not make the law in the same manner in which, legislative bodies do. Judges work
on a given legal material passed as law by the legislature. While declaring the law,
judges interpret the 'legislation' in question and play a creative role. By this
creative role, judges have contributed significantly to the development of law.

In the Indian context, former judges of the Supreme Court of India like Justice
P.N. Bhagwati and Justice Krishna Iyer enlarged the meaning and scope of
various provisions of the Constitution through their creative interpretation of the
legal text. The Supreme Court, too in its role as through an activist, has created
many new rights such as the: right to privacy, right to live in a pollution free
environment, right to livelihood etc.
The Right to Education has received considerable impetus during the last decade
as a result of the concerted effort of many groups and agencies towards ensuring
that all children in India receive at least minimum education, irrespective of their
socio-conomic status and their ability to pay for education, in a situation of
continuous impoverishment and erosion of basic needs. In a way, the right to
education is the culmination of efforts made possible by judicial interpretations and
a Constitutional amendment.

These new rights were created only by way of interpreting Article 21 (Right to
Life) of the Constitution of India. These rights developed by the courts are not in
any sense lesser than the laws enacted by the legislative bodies. Therefore, it can
be concluded that the judicial precedents are important sources of law in modern
society and judges do play a significant role in law-making.

Answer to Question No [2018(8) (a)]:


Legislation as source of a Law:

In modern times, legislation is considered as the most important source of law. The
term 'legislation' is derived from the Latin word legis which means 'law' and latum
which means "to make" or "set". Therefore, the word 'legislation' means the
'making of law'.
The importance of legislation as a source of law can be measured from the fact that
it is backed by the authority of the sovereign, and it is directly enacted and
recognised by the State. The expression 'legislation' has been used in various
senses. It includes every method of law-making. In the strict sense it means laws
enacted by the sovereign or any other person or institution authorised by him.

According to Salmond: ―Legislation is that source of law which comprises in the


assertion of lawful standards by a competent specialist.‖

According To Austin: ―Legislation is the command of the sovereign or the superior


authority which must be followed by the common masses backed by sanctions‖.

The kinds of legislation can be explained as follows:

(i) Supreme Legislation: When the laws are directly enacted by the sovereign, it
is considered as supreme legislation. One of the features of Supreme legislation is
that, no other authority except the sovereign itself can control or check it. The laws
enacted by the British Parliament fall in this category, as the British Parliament is
considered as sovereign. The law enacted by the Indian Parliament also falls in the
same category. However in India, powers of the Parliament are regulated and
controlled by the Constitution, through the laws enacted by it are not under the
control of any other legislative body.

(ii) Subordinate Legislation: Subordinate legislation is a legislation which is


made by any authority which is subordinate to the supreme or sovereign authority.
It is enacted under the delegated authority of the sovereign.

The origin, validity, existence and continuance of such legislation totally depends
on the will of the sovereign authority. Subordinate legislation further can be
classified into the following types:-
(a) Autonomous Law: When a group of individuals recognized or incorporated
under the law as an autonomous body, is conferred with the power to make rules
and regulation, the laws made by such body fall under autonomous law. For
instance, laws made by the bodies like Universities, incorporated companies etc.
fall in this category of legislation.

(b) Judicial Rules: In some countries, judiciary is conferred with the power to
make rules for their administrative procedures. For instance, under the Constitution
of India, the Supreme Court and High Courts have been conferred with such kinds
of power to regulate procedure and administration.

(c) Local laws: In some countries, local bodies are recognized and conferred with
the law-making powers. They are entitled to make bye-laws in their respective
jurisdictions. In India, local bodies like Panchayats and Municipal Corporations
have been recognized by the Constitution through the 73rd and 74th Constitutional
amendments. The rules and bye-laws enacted by them are examples of local laws.

(d) Colonial Law: Laws made by colonial countries for their colonies or the
countries controlled by them are known as colonial laws. For a long time, India
was governed by the laws passed by the British Parliament. However, as most
countries of the world have gained independence from the colonial powers, this
legislation is losing its importance and may not be recognized as a kind of
legislation.

(e) Laws made by the Executive: Laws are supposed to be enacted by the
sovereign and the sovereignty may be vested in one authority or it may be
distributed among the various organs of the State. In most of the modern States,
sovereignty is generally divided among the three organs of the State. The three
organs of the State namely legislature, executive and judiciary are vested with
three different functions. The prime responsibility of law-making vests with the
legislature, while the executive is vested with the responsibility to implement the
laws enacted by the legislature.

However, the legislature delegates some of its law-making powers to executive


organs which are also termed delegated legislation. Delegated legislation is also a
class of subordinate legislation. In welfare and modern states, the amount of
legislation has increased manifold and it is not possible for legislative bodies to go
through all the details of law. Therefore, it deals with only a fundamental part of
the legislation and wide discretion has been given to the executive to fill the gaps.
This increasing tendency of delegated legislation has been criticized. However,
delegated legislation is resorted to, on account of reasons like paucity of time,
technicalities of law and emergency. Therefore, delegated legislation is sometimes
considered as a necessary evil.

Delegated Legislation

 Delegated(subordinate or subsidiary) Legislation alludes to those laws


made by people or bodies to whom parliament has delegated law-making
powers.
 Where Acts are made by Parliament, a Principal Act may cause
arrangement for Subsidiary Legislation to be made and will to indicate
who can make laws as such under that Act.
 Delegated Legislation can just exist in connection to an empowering
or parent Act.
 Delegated Legislation contains the numerous regulatory subtleties
essential to guarantee that the arrangements of the Act will work
effectively. It might be directed by Government Departments, Local
Councils or Courts.
 Guidelines and Statutory Rules are the most widely recognised types of
Delegated Legislation. They are made by the Executive or a Minister
which apply to the overall public. By-laws, and once in a while
Ordinances are made by a Local Government Authority which also
applies to the general population who live around there. Principle and
Parent Act regularly depict methodology to be followed in Courts if there
is any flaw in a delegated law.

Advantages of Legislation as a Source of Law


Verifiably additionally the legislation has dependably been perceived as a
significant wellspring of law as contrasted and different sources. There are two
apparent explanations behind the legislation is viewed as a standout amongst the
most significant sources of law. Right off the bat, it includes setting down of
legitimate principles by the lawmaking bodies which the State perceives as law.

Besides, it has the power and authority of the State. It is hence said by Dias and
Hughes that conscious law-production by a legitimate power, i.e. the State is
called ‗legislation‟ which gave that sovereign is correctly perceived as the supreme
power by the courts. Relative Merit of Legislation over Precedent and customs
have been discussed below.

Some main advantages of legislation are as follows.

1. Abrogative Power—It can change or annul old law, which control isn‘t
controlled by different sources.
2. Effectiveness—It separates the elements of making law and overseeing it
between the Legislature and the legal executive.
3. Declaration — it gives that principles of law will be known before they
are authorised.
4. Reliance on Accidental Legislation — Legislation is independent and
emerges out of as the authoritative source of law it need not hold up until
the original case of legislation.
5. Unrivalled in Form — It is predominant in structure, brief, clear,
effectively available and understandable as against case law, which is an
increase of sense in a considerable amount of pointless issue.

Precedent and Legislation

1. The legislation has its source in the process of law which is basically
enacted and enforced by the State while the precedent has its origin in
ancient and historic judicial pronouncements.
2. Legislation has an authoritative force on courts by the assembly.
However, precedents are made by the courts themselves.
3. Legislation signifies formal declaration of law by the governing body
though precedents are acknowledgement and use of new standards of law
by courts in the administration of equity, justice and good conscience.
4. Legislation is ordered before a case emerges. However, the precedent
appears simply after the case has developed and taken for the choice of
the court.
5. Legislation is basically of an exhaustive structure while the extent of
legal precedent is restricted to comparable cases as it were.
6. Legislation is commonly and generally forthcoming while precedent
is retrospective in nature.
7. Legislation is announced or distributed before it is brought into power, on
the other hand, precedent comes into power on the double, i.e. when the
choice is articulated.
8. Legislation is finished with the goal of the lawmaking process yet it isn‘t
so on account of the precedent. The precedent which incorporates ratio
decidendi and obiter dicta are expected to settle a particular contest on the
purpose of law once for all.
9. It isn‘t hard for people, in general, to realise the law instituted by
lawmaking body yet the precedent dependent on the case law isn‘t
effectively known to the general population. Now and again, the attorneys
who manage law are themselves oblivious about the current case-law.
Therefore it makes a precedent of an ambiguous nature.
10.Legislation includes law-production by deductive strategy while case-law
is made by resorting to an inductive technique.

Answer To Question [2015(5)]:

Custom as Source of Law :

The word ‗custom‘ is derived from an old French word ‗Coustume‟. Some says
that the word ‗custom‘ is based on Latin word ‗Consuetudo‟,some says that the
word ‗Custom‘ is derived from the word ‗Consuetus‘, while others say that it is
the part participate of word ‗Consuescere‘ which means ‗ accustom‘. Some says
that it is derived from two words ‗con‘ means,‗expressing intensive force‘and
‗suescere‘ means ‗become accustomed‘.In Hindi the word ‗custom‘ means
‗reeti‟,‗vyavahar‘,‗rasm‘, or ‗riwaj‘.

The word ‗custom‘ generally means the following:

ace or class or

habitual with an individual.

of the same act; way of acting common to many;

ordinary manner; habitual practice; usage; method of doing or living.

authority on long consent, usage, and prescription.

It means familiar acquaintance or familiarity.

means long established habits or traditions of a society.

Salmond: - According to Salmond, ―custom is the embodiment of those principles


which have commended themselves to the national conscience as principles of
justice and public utility‖.
Austin: - According to Austin, ―custom is a rule of conduct which the governed
observe

VALID CUSTOM Each and every custom cannot be legally enforced. They must
be proved before the courts like any other thing before they can have the force of
law. Custom to be legally recognized by the courts and acquire the binding force of
law evolved some judicial tests. These tests are as follows:

Immemorial Antiquity:

The first test of a valid custom is that it must be immemorial. It must be old or
ancient and must not be of recent origin. Allen, Paton, Salmond and all other
jurists are of the views that before custom can be have the validity in law; it must
be shown that the custom is of immemorial antiquity or origin. In the words of
Allen, ―a mere habit, practice, or fashion which has existed for a number of years
nobody supposes to be Ipso factoan obligatory custom; antiquity is the only
reliable proof of resistance to the changing conditions of different ages.‖ But Dr.
G. William has objected to it as the statement is unconvincing.According to
Blackstone, ―A custom in order that it may be legal and binding, must have been
used as long that the memory of man runneth not to the contrary. In ancient Hindu
law also, the antiquity was one of the essentials for the recognition of custom.
Manu said, ―Immemorial custom is transcendental law‖. The idea of immemorial
custom was derived by the law of England from the Canon law, and by the Cannon
law from the Civil law. Time immemorial means in the Civil law and Cannon law
and in the systems derived there from and originally meant in England also time so
remote that no living man can remember it or give evidence concerning it. In
England a custom must be of reign ofRichard I King of England‖. That is in
England the time of legal memory is 1189 for a custom to be regarded as valid.
The year 1189, was the first year of the reign of Richard I. But the English rule of
‗immemorial origin‟ is not strictly to be followed in India.

The Allahabad High Court as early as 1895 laid down in KaurSenv.Mamman,that


it would be inexpedient to apply the English ‗rule of 1189‘ in India as it would
destroy many customary rights of modern growth in villages and in other places.
The Calcutta High Court, on the other hand, in Ambalika Dasi v. AparnaDasi
was of the view that either 1773 A.D. or 1793 A.D. is the date for treating a custom
which has been in existence as immemorial. The Bombay High Court was of the
view that if within last 20years, instances have occurred in which an alleged
custom has been recognized, the presumption is that it is of immemorial origin.
Likewise the Andhra Pradesh High Court following Bombay High Court observed
in Venkata SubbaRao v. Bhujangarrya that a custom being in existence for 40
years is an enforceable custom. The Supreme Court, however, finally in Gokul
Chand v. Parvin Kumari,decided the mater once for all by laying down that the
English rules of custom in order to be valid must have been used so long that the
memory of man runneth not to the contrary should not be strictly applied to the
Indian customs. In India it has been said that a custom must be of old nature, but
there is no such fixed period for which it must have been in existence as it is in the
English law.

REASONABLENESS

The second important judicial test of a valid custom is that it must be reasonable. It
must not be unreasonable. It must be useful and convenient to the society. If any
party challenges a custom, it must satisfy the court that the custom is unreasonable.
That is the burden of proof lies upon the person who challenges the custom. To
ascertain the reasonableness of a custom it must be traced back to the time of its
origin. The unreasonableness of a custom must be so great that its enforcement
results in greater harm than if there were no custom at all. According to prof.
Allen, the unreasonableness of the custom must be proved and not its
reasonableness. Therefore, a custom shall not be valid if it is apparently repugnant
to right and reason and it is likely to do more mischief than good if enforced. The
authority of a prevailing custom is never absolute, but it is authoritative provided it
conforms to the norms of justice and public utility.

Sir Edward Coke pointed out that a custom is contrary to reason if it is opposed to
the principles of justice, equity and good conscience. Salmond has rightly
suggested that before a custom is denied legal recognition, it must be found out
that the mischief resulting from its enforcement outweighs the harm that would
result from the multiplication of the natural expectation of the people. The judicial
committee of the Privy Council, delivering its judgment through Sir James
Colville in Raja Varma v. Ravi Varma observed that a custom which is not
reasonable is invalid in law and not binding. In this case it was held that a custom
permitting the sale of trusteeship of a religious endowment for pecuniary
advantage of the trustee as unreasonable custom. In Walstanton Ltd.
v.Newcastle-under-Lyme Corporation, it was held that Courts will not enforce
unreasonable customs, for law will not allow what is unreasonable and
inequitable.In case of Lutchmeeput v. sadaulla, a plaintiff, a zamindar sued to
restrain defendants from fishing in certain bhils (ponds) which formed part of his
zamindari and where the defendants contended that they had a prescriptive right to
fish under a custom according to which all the inhabitants of the zamindari have
the right of fishing in the bhils, it was held that the alleged custom was
unreasonable the defendants may take away the entire of fishing in the bhils
leaving nothing for the plaintiff who was admittedly the owner of them.

MORALITY

Third test of a valid custom is that a custom, to be valid, must not be immoral. It is
a well-recognized rule that a custom should not be opposed to decency and
morality. A custom must not be opposed to public policy or justice, equity or good
conscience.

InMathura Naikin v. Esu Naikin, the Bombay High Court has held that the
custom of adoption of girls for immoral purposes, like dancing is illegal as it was
designed to perpetuate this profession.

In case of Balusami v. BalaKishna, the custom permitting marriage with


daughter‘s daughter has also been held immoral. In Gopi v. jaggo, the Privy
Council allowed a custom which recognized and sanctioned re marriage of a
woman who had been abandoned and deserted by her husband.

The Bombay High Court, in Narayan v. Laving held a custom permitting a


woman to desert her husband at her pleasure and marries again without his consent
to be immoral.

Similarly in KeshavHargovan v. BaiGundi, the same Court held as custom by


which the marriage tie could be dissolved by either husband or wife against the
wish of the divorced party on payment of a sum of money, to be immoral.
CONTINUANCE
The fourth test for a valid custom is that it must have been continuously observed
without any interruption. General rule is that if a custom has not been followed
continuously and uninterruptedly for a long time, the presumption is that it never
existed at all. It must have been in existence and recognized by the community
without any intervening break, for such duration as may, under the circumstances
of the case, be recognized as reasonably long.

In case of Muhammad Hussainforki v. Syed Mian Saheb,it was held that unless
there is continuity there is no custom. A custom may be abrogatory and if it
abrogates another custom, such other custom ceases to exist. Blackstone has drawn
a distinction between the interruption of the ―right‖ and the interruption of the
mere ‗possession‖. It is the discontinuance of the ‗right‘, for howsoever small a
time that ends the custom. It means that if possession for some time is disturbed,
but the claim to enjoy the custom is not abandoned, the custom continues. The
discontinuance of the right even for a day shall put the custom to an end.

PEACEABLE ENJOYMENT

The next important test is that custom must have been enjoyed peaceably. If a
custom has been in dispute for a long time in a court of law or otherwise, it will
negate the presumption that it did originate by consent as most of the custom
naturally did. Therefore, for the enforceability of a custom; it is necessary to show
that the custom has been enjoyed without any disturbance or contest. A custom is
based on consent or habit, and unless there was an undisturbed existence of the
custom, we cannot say that it was based on the general consent of the people.

CONSISTENCY The test for a valid custom is that it must be in conformity with
the statute law. It should not be contrary to the statutory law. A custom should
necessary yield where it conflicts with a statutory law. This rule is observed as a
positive principle of law in England and countries like India which follow English
law. The Roman law and various continental systems, however, do not adhere to
this rule. Justinian in his corpus juris mentions several statutes which have fallen
into disuse by a posterior contrary custom.That is to say, the latter rule prevails
over the earlier, regardless of their origins and legislation has no inherent
superiority in this respect over custom. If an enacted law comes first it can be
repealed or modified by a later custom and vice versa. Commenting on this aspect
Savigny pointed out that customs and statutes are put on the same level with
respect to their legal efficiency and customary law may complete, modify or repeal
a statute, it may create a new rule and substitute it for a statutory rule which it has
abolished. In Scotland and ancient Grace also a statute may fall into disuse by the
posterior contrary custom.

But in India the position is clear that custom must not be opposed to statute law, as
the same thing has been held by the Indian Supreme Court in Darshansing v.
NaimumNisaBibi.Obviously custom cannot abrogate a newly enacted legislation.
For instance, among the Hindus all the customary forms of marriage, adoption,
succession, orproperty have been abrogated by the newly enacted legislation
concerning such problems. Hence an old inconvenient and unjust custom cannot be
set up against statutory law. For instance, the custom of child marriage and usage
of dowry has no legal force in modern India. According to Coke, ―No custom or
prescription can take away the force of an Act of parliament. ― A state can
abrogate a custom and not vice versa. However, it is to be observed that there
are writers who hold different views on this point. According to them, legislation
has no inherent superiority over custom. If the enacted law comes first, it can be
repealed or modified by a later custom. If the customary law is the earlier, it can
similarly be dealt with by later enacted law. According to Savigny, ―if we
consider customs and statutes with respect to their legal efficacy, we must put them
on the same level. Customary law may complete, modify or repeal a statute; it may
create a new rule and substitute it for the statutory rule which it has abolished.‖
According to Windshield, ―the power of customary law is equal to that of statutory
law. It may, therefore, not merely supplement but also derogate from the existing
law.‖

CERTAINTY Certainty is an indispensable condition of a valid custom. A


custom, however, ancient must not be indefinite and uncertain. In Wilson v.
Willes, it was held that a custom must be certain and not vague. A custom which is
vague or indefinite cannot be recognized. It is more a rule of evidence than
anything else. The court must be satisfied by a clear proof that custom exists as a
matter of fact, or as a legal presumption of fact. In one case, the plaintiff claimed a
customary right of easement for the shadow falling from the branches of trees
hanging from the neighbour‘s field.Mr Justice Pandalai of the High court of
Madras ruled that there cannot be a custom relating to shadow of tress because it is
so uncertain, ambiguous and transitory that it cannot give rise to any customary
right.

COMPULSORY OBSERVANCE A custom to be legally recognized as a valid


custom must be observed as a right. It means that custom must have been followed
by all concerned without recourse to force and without the necessity of permission
of those who are adversely affected by it. It must be regarded by those affected by
it not merely as an optional rule but as an obligatory or binding rule of conduct. If
a practice is left to individual choice; it cannot be treated as a customary law.These
requisites are expressed in the form of the rule that the user must be nec vi nec
clam necprecario- not by force, nor by stealth, nor at will. In Hamperton v.
Hono, it was held that if the observance of a custom is suspended for a long time,
it would be assumed that such a custom was never in existence.

JURIDICAL NATURE A custom must be of a juridical nature. A custom must


refer to legal relations. A mere voluntary practice not conceived of as being based
on any rule of right or obligation does not amount to a legal custom.
10. PUBLIC POLICY Another test for the validity of a custom is that it should
not be opposed to public policy. This test may be included in the test of
reasonableness, as it is very wide term and it may include public policy as well. In
case of Budanso v. Faturr, a custom which would enable a woman to marry again
during the life time of her husband without any defined rules by which the
marriage with the first husband is dissolved before the second marriage is
contracted, was held to be contrary to public policy. In nut shell a custom is valid if
it not contrary to justice, equity or good conscience or opposed to public policy.

NOT BY ANALOGY Custom cannot be extended by analogy. It must be


established inductively, not deductively and it cannot be established by priori
methods. It cannot be a matter of theory but must be always be a matter of fact.
Likewise one custom cannot be deduced from other. Custom also cannot be set up
against fundamental rights.
Doctrine of Separation of Power

(Q) Explain the doctrine of Separation of powers and discuss its applicability in
US, UK and India. [2019(2)(b)]

(Q)Explain the importance of the theory of ‗separation of powers‟. [2017(2)(d)]

(Q) What do you understand by the Doctrine of separation of Power? Is the


doctrine followed in India? [2016(2)(a)]

The doctrine of Separation of Powers deals with the mutual relations among the
three organs of the Government namely legislature, executive and judiciary. The
origin of this principle goes back to the period of Plato and Aristotle.

It was Aristotle who for the first time classified the functions of the Government
into three categories viz., deliberative, magisterial and judicial.

In 1689 the English political theorist John Locke also envisaged a threefold
classification of powers in the book The Second Treatise of Government as:
―May be too great a temptation to human frailty…for the same person to have the
power of making laws, to have also in their hands the power to execute them,
whereby they may exempt themselves from obedience to laws they make, and suit
the law both in its making and execution, to make their own private advantage.‖

The French Jurist Montesquieu in his book L. Esprit Des Lois (Spirit of Laws)
published in 1748, for the first time enunciated the principle of separation of
powers. That‘s why he is known as modern exponent of this theory.
Montesquieu‟s doctrine, in essence, signifies the fact that one person or body of
persons should not exercise all the three powers of the Government viz. legislative,
executive and judiciary. In other words each organ should restrict itself to its own
sphere and restrain from transgressing the province of the other.
In the view of Montesquieu:
―When the legislative and executive powers are united in the same person, or In the
same body or Magistrate, there can be no liberty. Again, there is no liberty if the
judicial power is not separated from the Legislative and Executive power. Where it
joined with the legislative power, the life and liberty of the subject would be
exposed to arbitrary control, for the judge would then be the legislator. Where it
joined with the executive power, the judge might behave with violence and
oppression. There would be ends of everything were the same man or the same
body to exercise these three powers...
Montesquieu‟s ―Separation‖ took the form, not of impassable barriers and
unalterable frontiers, but of mutual restraints, or of what afterwards came to be
known as ―checks and balances‖. The three organs much act in concert, not that
their respective functions should not ever touch one another. If this limitation is
respected and preserved, ―it is impossible for that situation to arise which Locke
and Monstequieu regarded as the eclipse of liberty- the monopoly, or
disproportionate accumulation of power in one sphere.‖
The man behind the principles is to protect the people again capricious tyrannical
and whimsical powers of the State.

The theory of separation of powers signifies mainly three formulations of


Governmental powers;

i. The same person should not form part of more than one of the three organs
of the state.
ii. One organ should not interfere with any other organ of the state.
iii. One organ should not exercise the functions assigned to any other organ.

United Kingdom: The famous English Jurist Blackstone supported the


doctrine of Montesquieu. According to him, ―wherever the right of making
and enforcing the Law is vested in the same man or in the same body of
men there can be no liberty‖. During the 17th century in England
Parliament exercised legislative powers. The King exercised executive
powers, and the Courts exercised judicial powers, but with the emergence of
cabinet system of Government i.e. Parliamentary form of Government, the
doctrine remains no good. The renowned constitutional Bagehot observed.
―The cabinet is a hyphen which joins, buckle which fastens the legislative
part of the State to the executive part of the State.‖
According to Wade and Phillips the doctrine of separation of powers
implies:
(i) The same person should not form more than one organ of the
Government.
(ii) One organ of the Government should not exercise the function of
other organs of the Government.
(iii) One organ of the Government should not encroach with the function
of the other two organs of the Government.
Now the question in subject is whether this doctrine finds a place in
England? In England the King being the executive head s also an integral
part of the legislature. His ministers are also members of one or other
Houses of Parliament. This concept goes against the idea that same person
should not form part of more than one organ of the Government.
In England House of Commons control the executive. So far as judiciary is
concerned, in theory House of Lords is the highest Court of the country but
in practice judicial functions are discharged by persons who are appointed
specially for this purpose, they are known as Law Lords and other persons
who held judicial post. Thus we can say that doctrine of separation of
powers is not an essential feature of British Constitution.
There are too many examples of overlap between three functions of the
government. We can see the several examples as follows:
· Law Lords sit on the appellate committee of the House of Lords and the
judicial committee of the Privy Council as well as in the House of Lords as
a legislative body;
· Parliament exercises a legislative function and to lesser extent a judicial
function is that it is responsible for the regulation of its own internal affairs;
· Government ministers are member of the executive who exercise a
legislative function not only in Parliament but also in delegate legislation; ·
In addition to exercise the judicial function, courts legislate in the sense that
they develop principles of the common law;
· Magistrates exercise administrative as well as judicial functions in that
they grant licenses.

U.S.A.: Usually it is said that the principle of separation of powers finds a


good mention in the Constitution of United States; while the Federal
Constitution of the United States of America does not expressly provide for
the principle of separation of powers. Having reliance on the doctrine of
Montesquieu, Madison, the Federalist observed; ―The accumulation of all
powers legislative, executive and judicial, in the same hands whether of
one, a few or many and whether hereditary, self-appointed or elective, may
justly be pronounced the very definition of tyranny.‖ The same ideas were
expressed by Hamilton in 1788.
In American Constitution we find that legislative, executive and judicial
powers are vested in separate entities.
Section 1 of Article 1 declares: ―All legislative powers herein granted
shall be vested in a Congress of the United State‖.
Section 1 of Article II says: ―The executive power shall be vested in a
President of the United States of America.‖
Section 1 of Article III reads: ―The judicial power of the United States,
shall be vested in one Supreme Court and in such inferior courts as the
Congress may from time to time ordain and establish.....‖

Let us see the actual position prevailing in America. It is clear from the
above mentioned provisions that the President is the head of the executive in
U.S.A. Besides he (President) possesses the power to vote bill passed by the
Congress and such bills cannot become law unless they are subsequently passed
again by each House, with a two thirds, majority. The character of veto power
vested in the President is purely legislative.

The President also exercises legislative power in making of treaties regarding


foreign affairs. As John Marshall said in his great argument of March 7, 1800, in
the House of Representative, ―The President is the sole organ of the nation in its
external relations, and its sole representative with foreign nations‖. The President
in his legislative capacity gives information of the state of the Union to the
Congress.

Likewise the Congress interferes with the powers of President by casting vote on
Budget. The Budget and Accounting Act, 1921 established the principle and
practice of the executive budget, under which the President is responsible for
formulating and presenting to Congress a complete and detailed expenditure plan
for the following fiscal year. Congress also plays important role in ratification of
treaties, as well as in appointments through its senators. Congress has also
judicial powers. Each house may expel its members by a two thirds, votes or
punish them for disorderly behaviour. Congress is the sole judge of the reason for
expulsion. Not only this, American judges may be removed from the office only by
impeachment proceedings instituted before the Congress.
So far as judicial organ is concerned the Courts have supervisory control over both
the Congress and the President, by way of judicial review. It is true that legislature
enacts the Law, but it is also true that in dealing with the new problems, where
Law is silent, the Courts have to create the Law. The Chief Justice Hughes‟s
remarks are most pertinent in this connection, as he candidly said- ―The
Constitution is what the judges say it is‖. The amendments which have been
incorporated in American Constitution, all are not by Congress itself, but most of
the amendments have been incorporated in Constitution by American Supreme
Court. In this way it can be said that in U.S.A. there is also not any possibility to
have a rigid personal separation of powers.

Position In India:
Constitutional Provisions: There are no separate provisions regarding the
Doctrine of Separation of Powers has been given in our Constitution. But there are
some directive principles are given in the constitution as in Part-IV and Part-V and
Articale-50 of our constitution is separating the judiciary from executive as, ―the
state shall take steps to separate judiciary from the executive in the public services
of the state,‖ and except this there is no formal and dogmatic division of powers.

Judiciary: Under Article-142 and Article-145 of our constitution, the SC has the
power to declare void the laws passed by legislature and actions taken by the
executive if they violate any provision of the constitution or the law passed by the
legislature in case of executive actions. Even the power to amend the constitution
by Parliament is subject to the scrutiny of the Court. The Court can declare any
amendment void if it changes the basic structure of the constitution. In many cases
courts have issued directions for the Parliament to make policies.
Executive: The President of India who is the supreme executive authority in India
exercise law making power in the form of ordinance making power under Article-
123, also the Judicial powers under Article-103(1) (disqualification of members of
either house of parliament) and Article-217(3) (appointment of judges in H.C), he
has the consulting power to the SC of India under Article-143 and also the
pardoning power in Article-72 of the Constitution. The executive also affecting
functioning of the judiciary by making appointments to the office of Chief Justice
of India and other judges.
Legislature: The Council of Minister is selected from the legislature and this
Council is responsible for the legislature. The legislature exercising judicial powers
in cases of breach of its privileges, impeachment of the President under Article-61
and removal of judges. The legislative body has the punitive powers under Article-
105(3). In words of Gledhill, ―constitution of India has not ceremoniously wedded
with Doctrine of Separation of Powers; however, it is whenever possible followed
the doctrine of separation of powers.‖

In Indian Constitution there is express provision that ―Executive power of the


Union shall be vested in the President, and the executive power of the State shall
be vested in Governor..‖ (Article 154(1) of Indian Constitution). But there is no
express provision that legislative and judicial powers shall be vested in any person
or organ.
Now we have to see what is the real position in India regarding the separation of
powers?
President being the executive head is also empowered to exercise legislative
powers. In his legislative capacity he may promulgate Ordinances in order to meet
the situation as Article 123(1) says ―If at any time, except when both Houses of
Parliament are in Session, President is satisfied that circumstances exist which
render it necessary for him to take immediate action, he may promulgate such
Ordinance as the circumstances appear to him to require‖.
When Proclamation of emergency has been declared by the President due to failure
of Constitutional machinery the President has been given legislative power under
Article 357 of our Constitution to make any Law in order to meet the situations. A
power has also been conferred on the President of India under Article 372 and 372-
A to adapt any Law in country by making such adaptations and modifications,
whether by way of repeal or amendment as may be necessary or expedient for the
purpose or bringing the provisions of such Law into accord with the provisions of
the Constitution.
The President of India also exercises judicial function. Article 103(1) of the
Constitution is notable in this connection. According to this Article ―If any
question arises as to whether a member or either of House of Parliament has
become subject to disqualification mentioned in clause (1) of Article 102, the
questions hall be referred for the decision of the President and his decision shall
be final‖. Article 50 lays emphasis to separate judiciary from executive. But in
practice we find that the executive also exercises the powers of judiciary as in
appointment of judges. (Articles 124, 126 & Article 127). The legislative (either
House of Parliament) also exercises Judicial function in removal of President
(Article 56) in the prescribed manner.Judiciary also exercises legislative power,
High Court and Supreme Court are empowered to make certain rules legislative in
character. Whenever High Court or the Supreme Court finds a certain provision of
law against the Constitution or public policy it declares the same null and void, and
then amendments may be incorporated in the Legal System. Some time High Court
and Supreme Court formulate the principles on the point where law is silent. This
power is also legislative in character.

Separation of powers and Judicial opinion:

In Re Delhi Law Act,AIR 1951 SC 382, case Hon‟ble Chief Justice Kania
observed:
―Although in the Constitution of India there is no express separation of powers, it
is clear that a legislature is created by the Constitution and detailed provisions are
made for making that legislature pass laws. It is then too much to say that under the
Constitution the duty to make laws, the duty to exercise its own wisdom, judgment
and patriotism in making law is primarily cast on the legislature? Does it not imply
that unless it can be gathered from other provisions of the Constitution, other
bodies executive or judicial are not intended to discharge legislative functions?‖

In Rai Sahib Ram Jawaya v. State of Punjab reported in AIR 1955 S.C. 549

Hon‟ble Chief Justice B.K. Mukherjea observed:

Although in the Constitution of India there is no express separation of powers, it is


clear that a legislature is created by the Constitution and detailed provisions are
made for making that legislature pass laws. Is it then too much to say that under the
Constitution the duty to make laws, the duty to exercise its own wisdom, judgment
and patriotism in making law is primarily cast on the legislature? Does it not imply
that unless it can be gathered from other provisions of the Constitution, other
bodies executive or judicial are not intended to discharge legislative functions?
―The Indian Constitution has not indeed recognized the doctrine of separation of
powers in the absolute rigidity but the functions of the different parts or branches
of the Government have been sufficiently differentiated and consequently it can
very well be said that our Constitution does not contemplate assumption by one
organ or part of the State of the functions that essentially belong to another.‖

The functions of a modern State unlike the police States of old are not confined to
mere collection of taxes or maintenance of laws and protection of the realm from
external or internal enemies. A modern State is certainly expected to engage in all
activities necessary for the promotion of the social and economic welfare of the
community.

In Smt. Indira Nehru Gandhi v. Raj Narain, AIR 1975 SC 2299,

Hon‟ble Justice Chandrachud observed: ―The American Constitution provides for


a rigid separation of governmental powers into three basic divisions the executive,
legislative and judicial. It is essential principle of that Constitution that powers
entrusted to one department should not be exercised by any other department. The
Australian Constitution follows the same pattern of distribution of powers. Unlike
these Constitutions, the Indian Constitution does not expressly vest the three kinds
of power in three different organs of the State. But the principle of separation of
powers is not a magic formula for keeping the three organs of the State within the
strict confines of their functions.‖

S.C. Advocates-on-Record Case, AIR 1994 :

The State in the present day has become the major litigant and the superior courts,
particularly the Supreme Court, have become centers for turbulent controversies
some of which with a flavour of political repercussions and the courts have to face
tempest and storm because their vitality is a national imperative. In such
circumstances, therefore, can the Government, namely, the major litigant be
justified in enjoying absolute authority in nominating and appointing its
arbitrators? The answer would be in the negative. If such a process is allowed to
continue, the independence of judiciary in the long run will sink without any trace.
In Chandra Mohan v. State of U.P., AIR 1966 SC 1987
Supreme Court held: ―The Indian Constitution, though it does not accept the strict
doctrine of separation of powers, provides for an independent judiciary in the
States....... But at the time the direct control of the executive. Indeed it is common
knowledge that in pre-independence India there was a strong agitation that the
judiciary should be separated from the executive and that the agitation that the
judiciary should be separated from the executive and that the agitation was based
upon the assumption that unless they were separated, the independence of the
judiciary at the power levels would be a mockery.‖

In Asif Hameed v. State of Jammu and Kashmir reported in AIR 1989 S.C.
1899 the Supreme Court observed:
―Although the doctrine of separation of powers has not been recognized under the
Constitution in its absolute rigidity but the Constitution makers have meticulously
defined the functions of various organs of the State. Legislature, executive and
judiciary have to function within their own spheres demarcated under the
Constitution. No organ can usurp the functions assigned to another.
The Constitution trusts to the judgment of these organs to function and exercise
their discretion by strictly following the procedure prescribed therein. The
functioning of democracy depends upon the strength and independence of each of
its organs.‖

In Kesavananda Bharti v. State of Kerala, AIR 1973 SC 1461


Hon‟ble Chief Justice Sikri observed:
―Separation of powers between the legislature, the executive and the judiciary is a
part of the basic structure of the Constitution; this structure cannot be destroyed by
any form of amendment.‖

In Hari Shankar Nagla v. State of M.P. It was observed:


―The Legislature cannot delegate its function of laying down legislative policy in
respect of a measure and its formulation as a rule of conduct. The Legislature must
declare the policy of the law and the legal principles which are to control any given
cases and must provide a standard to guide the officials or the body in power to
execute the law. The essential legislature function consists in the determination of
the choice of the legislative policy and of formally enacting that policy into a
binding rule of conduct.‖
Virtually, absolute separation of powers is not possible in any form of
Government. In view of the variety of situations, the legislature cannot fore-see or
anticipates all the circumstances to which a legislative measure should be extended
and applied. Therefore, legislature is empowered to delegate some of its functions
to administrative authority (executive). But one thing is notable that legislature
cannot delegate its essential legislative power.

Nowadays in response to public interest litigation writs, the courts have begun to
direct the Government on everything from clearing garbage off the streets to
cleansing the polity of political sleaze.
With the widening of the horizons of ―Judicial Activism‖ criticism emanated from
a few percent of the people that the judiciary is overstepping its bounds and taking
over the Government functions, but this is not a justifiable thought.

The Supreme Court and the High Court‘s act as a watch-dogs to keep Executive
and Legislature within the bounds of law. Today millions of the people are
suffering in the country. It is the judiciary which is holding out hope for them.

Advantages: There are various advantages with the acceptance of this doctrine in
the system;

1. The efficiency of the organs of state increased due to separation of works hence
time consumption decreases.

2. Since the experts will handle the matters of their parts so the degree of purity
and correctness increases.

3. There is the division of work and hence division of skill and labour occurs.

4. Due to division of work there is no overlapping remains in the system and hence
nobody interfere with others working area.

5. Since the overlapping removed then there is no possibility of the competition in


between different organs.
Disadvantages: As there are advantages attached to this doctrine, there are some
disadvantages can also occur due to this doctrine;
1. As I have said there will be increased efficiency but reverse effect can also be
seen because of the overlapping between rights of the organs if we are not
following the doctrine in its strict sense because organs may fight for the
supremacy over each other.

2. There is also a possibility of competition between organs again for proving ones
supremacy over the other organ.

3. There is also possibility of delay of process because there will not be any
supervisor over other hence the actions of the organs can become arbitrary.

Evaluation of the Doctrine:


In strict sense the principle of separation of powers cannot be applied in any
modern Government either may be U.K., U.S.A., France, India or Australia. But it
does not mean that the principle has no relevance now days. Government is an
organic unity. It cannot be divided into water tight compartments.
History proves this fact. If there is a complete separation of powers the government
cannot run smoothly and effectively. Smooth running of government is possible
only by co-operation and mutual adjustment of all the three organs of the
government. Prof. Garner has rightly said, ―the doctrine is impracticable as a
working principle of Government.‖ It is not possible to categorize the functions of
all three branches of Government on mathematical basis. The observation of
Frankfurter is notable in this connection.

According to him ―Enforcement of a rigid conception of separation of powers


would make Government impossible. It is my opinion that the doctrine of
Montesquieu is not merely a myth it also carries a truth, but in the sense that
each organ of the Government should exercise its power on the principle of
―Checks and Balances‖ signifying the fact that none of the organs of Government
should usurp the essential functions of the other organs. Professor Laski has aptly
remarked: ―It is necessary to have a separation of functions which need not
imply a separation of personnel.‖

Rule of Law
(Q) Is Rules of Law a myth or reality in Indian Legal System of today? What had
been the judicial approach to ‗Rule of Law‟ in India ? [2018(2)(b)]

(Q)Discuss any landmark cases decided by our supreme court regarding the
implementation of ‗Rule of Law‟. [2017(2)(c)]

(Q) How does Prof. Dicey explain the Doctrine of Rule of Law? Is it followed
by the judiciary in India? [2016(2)(b)]

(Q) Critically explain Dicey‟s Rule of Law and discuss its applicability in India.
[2019(2)(a)]

Rule of Law

The rule of law is a concept that describes the supreme authority of the law over
governmental action and individual behaviour. It corresponds to a situation where
both the government and individuals are bound by the law and comply with it. It is
the antithesis of tyrannical or arbitrary rule.

Rule of Law collates the rules which are based on the principles of freedom,
equality, non-discrimination, fraternity, accountability and non-arbitrariness and is
certain, regular and predictable. ―The concept shares the common English
inheritance and apart from the statement of generalities, it embraces a body of
specific detail.‖ It is this detail that furnishes the foundation for a pragmatic system
of governance.

In fact, it could be regarded as a modern name of Natural Law. Jurisprudentially,


Romans called it 'Jus Naturale', mediaevalists called it the 'Law of God‟,
Hobbes, Locke and Roussueau called it 'Social Contract' or 'Natural Law' and
the modern jurists call it 'Rule of Law'. The idea has been developed from the
French phrase 'la principle de legalite', i.e. a Government based on the principles
of law and not of men. However, it was Edward Coke who is theoretically the
originator when he said that the King must be under God and Law and thus
vindicated the supremacy of law over the pretensions of the executives. In India
too, the concept of Rule of Law can be traced to Upanishads where it provides
that Law is Kings of Kings. Indeed, from the legendary days of Adam and of
Kautilya's Arthasastra -- the rule of law has had this stamp of natural justice,
which makes it social justice.5 Even Plato believed that if ordinary men were
allowed to rule by will alone, the interests of the community would be scarified to
those of the ruler.

Doctrine of Rule of Law by prof. Dicey

According to Prof. Dicey, rule of law has three means or we can say three
principles which are must be followed so that there will be supremacy of rule of
law. The three principles are:

1. Supremacy of law;
2. Equality before Law; and
3. Predominance of Legal Spirit.

1. Supremacy of Law

According to the first principle, A. V Dicey states that rule of law means there
should be lacking of arbitrariness or wide discretionary power. In other words
every act will be controlled by law. According to Dicey the English men were
ruled by the law and law alone. In the words of Dicey, ―Wherever there is
discretion, there is room for arbitrariness and that in a republic no less than under a
monarchy discretionary authority on the part of the Government must mean
insecurity for legal freedom on the part of its subjects.‖

As Wade says the rule of law requires that the Government should be subject to the
law, rather than the law subject to the Government. The rule of law requires both
citizens and governments to be subject to known and standing laws. The
supremacy of law also requires generality in the law. This principle is a further
development of the principle of equality before the law. Laws should not be made
in respect of particular persons. As Dicey postulated, the rule of law presupposes
the absence of wide discretionary authority in the rulers, so that they cannot make
their own laws but must govern according to the established laws. Those laws
ought not to be too easily changeable. Stable laws are a prerequisite of the certainty
and confidence which form an essential part of individual freedom and security.
Therefore, laws ought to be rooted in moral principles, which cannot be achieved if
they are framed in too detailed a manner.
2. Equality before Law

The attribute of ―Rule of Law‖ Dicey stated was ―equality before the law and equal
subjection of all classes to the ordinary law of the land administered by the
ordinary law courts. ―The second principle emphasizes everyone, including the
government, irrespective of rank, shall be subject to the same law and courts. This
element is interpreted to be misguided and facing bundle of criticisms. In fact, by
reason of maintaining the law and order in the society, there are actually
exceptions such as the Crown, police, Members of Parliament. The Crown may
exercise prerogative powers which may defeat the rights of individuals. The police
have powers over and above the citizen. Members of Parliament have immunity
from the law of defamation. Prof. Dicey states that, there must be equality before
the law or equal subjection of all classes to the ordinary law of the land. He
criticised the French legal system of droit Administrative in which there were
separate administrative tribunals for deciding the cases of State Officials and
citizens separately.

3. Predominance of Legal Spirit

The Third meaning of the rule of law is that the general principles of the
constitution are the result of juridical decisions determining file rights of private
persons in particular cases brought before the Court. Dicey states that many
constitutions of the states (countries) guarantee their citizens certain rights
(fundamental or human or basic rights) such as right to personal liberty, freedom
from arrest etc. According to him documentary guarantee of such rights is not
enough. Such rights can be made available to the citizens only when they are
properly enforceable in the Courts of law, For Instance, in England there is no
written constitution and such rights are the result judicial decision.

The view of Dicey as to the meaning of the Rule of Law has been subject of
much criticism. The whole criticism may be summed up as follows. Dicey has
opposed the system of providing the discretionary power to the administration. In
his opinion providing the discretionary power means creating the room for
arbitrariness, which may create as serious threat to individual freedom. Now days
it has been clear that providing the discretion to the administration is inevitable.
The opinion of the Dicey, thus, appears to be outdated as it restricts the
Government action and fails to take note of the changed conception of the
Government of the State. Dicey has failed to distinguish discretionary powers from
the arbitrary powers. Arbitrary power may be taken as against the concept of Rule
of Law. In modern times in all the countries including England, America and India,
the discretionary powers are conferred on the Government. The present trend is
that discretionary power is given to the Government or administrative authorities,
but the statute which provides it to the Government or the administrative officers
lays down some guidelines or principles according to which the discretionary
power is to be exercised.

The administrative law is much concerned with the control of the discretionary
power of the administration. It is engaged in finding out the new ways and means
of the control of the administrative discretion. According to Dicey the rule of law
requires that every person should be subject to the ordinary courts of the country.
Dicey has claimed that there is no separate law and separate court for the trial of
the Government servants in England. He critcised the system of droit
administrative which is prevailing in France. In France there are two types of
courts Administrative Court and Ordinary Civil Courts. The disputes between the
citizens and the Administration are decided by the Administrative courts while the
other cases, (i.e. the disputes between the citizens) are decided by the Civil Court.
Dicey was very critical to the separation for deciding the disputes between the
administration and the citizens According to Dicey the Rule of Law requires equal
subjection of all persons to the ordinary law of the country and absence of special
privileges for person including the administrative authority. This proportion of
Dicey does not appear to be correct even in England. Several persons enjoy some
privileges and immunities. For example, Judges enjoy immunities from suit in
respect of their acts done in discharge of their official function. Besides, Public
Authorities Protection Act, 1893, has provided special protection to the official.
Foreign diplomats enjoy immunity before the Court. Further, the rules of „public
interest privilege may afford officials some protection against orders for discovery
of documents in litigation.‟ Thus, the meaning of rule of law taken by Dicey
cannot be taken to be completely satisfactory. Third meaning given to the rule of
law by Dicey that the constitution is the result of judicial decisions determining the
rights of private persons in particular cases brought before the Courts is based on
the peculiar character of the Constitution of Great Britain. In spite of the above
shortcomings in the definition of rule of law by Dicey, he must be praised for
drawing the attention of the scholars and authorities towards the need of
controlling the discretionary powers of the administration.

Modern concept of Rule of Law

As the object of rule of law is to protect individual liberty then to fulfill this object
it can be taken many meanings. K.C Davis gives seven principles or meanings of
the term rule of law.

1. Law and order

2. Fixed rule

3. Due process of law or fairness

4. Observance of principle of natural justice

5. Elimination of discretionary power

6. Preference for Judges and Ordinary courts

7. Judicial Review of administrative actions.

RULE OF LAW IN INDIAN CONSTITUTION AND JUDICIAL APPROACH

Rule of Law vis a vis Constitution

In Indian Constitution, Rule of Law has been adopted under the Preamble where
the ideals of justice, liberty and equality are enshrined. The Constitution has been
made the supreme law of the country and other laws are required to be in
conformity with the Constitution. Nonetheless, the courts have the onus to declare
any law invalid, which is found in violation of any provision of the Constitution.
Part III of the Constitution of India guarantees the Fundamental Rights. Article
13(l) of the Constitution makes it clear that all laws in force in the territory of India
immediately before the commencement of the Constitution, in so far as they are
inconsistent with the provisions of Part III dealing with the Fundamental
Rights, shall, to the extent of such inconsistency, be void. Article 13(2) provides
that the State should not make any law, which takes away or abridges the
fundamental rights and any law made in contravention of this clause shall, to the
extent of the contravention, be void. The Article 14 of the Constitution guarantees
equality before law and equal protection of laws. Article 21 guarantees right to
life and personal liberty. It provides that no person shall be deprived of his life or
personal liberty except according to the procedure established by law.

Article 19 guarantees six Fundamental Freedoms to the citizens of India --


freedom of speech and expression, freedom of assembly, freedom to form
associations or unions, freedom to live in any part of the territory of India and
freedom of profession, occupation, trade or business. The right to these freedoms is
not absolute, but subject to the reasonable restrictions which may be imposed by
the State. Article 20(1) provides that no person shall he convicted of any offence
except for violation of a law in force at the time of the commission of the act
charged as an offence not be subject to a penalty greater than that which might
have been inflicted under the law in force at the time of the commission of the
offence.

According to Article 20(2), no person shall be prosecuted and punished for the
same offence more than once. Article 20(3) makes it clear that no person accused
of the offence shall be compelled to be witness against himself. In India, the
Constitution is supreme and the three organs of the Government viz. Legislature,
Executive and Judiciary are subordinate to it. It provides though for encroachment
of one organ (eg-Legislature) upon other (eg-Judiciary) if its action is malafide,
and the citizen (individual) can challenge under Article 32 of the constitution if the
action of the executive or legislature violates the fundamental rights of citizens
before the judiciary.

In India, the meaning of rule of law has been much expanded and applied
differently in different cases by the judiciary. It is regarded as a basic structure of
the constitution and therefore, it cannot be abrogated or destroyed even by
parliament.6 The principle of natural justice is also considered as the basic corollary
of rule of law. The Supreme Court of India has held that in order to satisfy a
challenge under Article 14, the impugned State act (enactment in the form of law
passed by parliament) must not only be nondiscriminatory, but also be immune
from arbitrariness7, unreasonableness or unfairness (substantively or
procedurally)and also consonant with public interest.

In A.D.M Jabalpur v Shivakant Shukla,

The question before the apex court was, whether there was any rule of law in India
apart from Article 21 of the Indian Constitution. The court by majority held that
there is no rule of law other than the constitutional rule of law. However, Justice
Khanna did not agree with the above view. He rightly said, ―Even in the absence
of Article 21 of the constitution, the State has no power to deprive a person of
his life or liberty without the authority of law.‖

In K.T. Plantation Pvt. Ltd. v. State of Karnataka,held as follows;

―The rule of law as a principle contains no explicit substantive component like


eminent domain but has many shades and colours. Violation of principle of natural
justice may undermine the rule of law resulting in arbitrariness, unreasonableness,
etc. but such violations may not undermine the rule of law of law so as to
invalidate a statue. Violation must be of such a serious nature which undermines
the very basic structure of the constitution and the democratic principles of India.
But once the court finds, a statue undermines the rule of law which has the status
of a constitutional principle like the basic structure, the said grounds are also
available and not vice versa. Any law which in the opinion of the court is not just,
fair and reasonable is not a ground to strike down a statute because such an
approach would always be subjective not the will of the people because there is
always a presumption of constitutionality for a statue.‖

Role of Courts vis a vis Rule of Law

Indian Courts are bestowed upon with the duty to protect, deliberate and
acknowledge the individual rights of the people with the continuing effort of
upholding the constitutional beliefs of a democratic country. Further, the authority
is also responsible to see that the powers are not abused and that those armed with
such powers exercise them in accordance with the laws enacted for the required
purpose. According to the scheme of our Constitution, such command is exercised
by the courts. The purpose of the courts as arbiter of disputes between the State
and the citizen highlights the importance of the independence of the courts as an
extremely powerful constitutional body, which carries a heavy onus to provide
proper checks and balances in the system of governance.

As held in Narmada Bachao Andolan v. Union of India and Ors. that:

"It is now well settled that the Courts, in the exercise of their jurisdiction, will not
transgress into the field of policy decision. Whether to have an infrastructural
project or not and what is the type of project to be undertaken and how it has to be
executed, are part of policy-making process and the Courts are ill-equipped to
adjudicate on a policy decision so undertaken. The Court, no doubt, has a duty to
see that in the undertaking of a decision, no law is violated and people's
fundamental rights are not transgressed upon except to the extent permissible under
the Constitution..."

Judicial Activism and Rule of Law

Judicial Activism is also alleged to have taken a form of judicial legislation. But it
is through this tool, the judiciary has also taken up the responsibility to fill up the
legislative vacuum in order to uphold the rule of law. The silence of the
Constitution and the abeyances left to be filled by the growth of conventions within
the meaning of the enacted provisions. This exercise has been performed by the
Supreme Court of India in consonance with the constitutional scheme.

The progress of the Society is dependent upon proper application of law to its
needs and the judiciary has to mould and shape the law to deal with such rights and
obligations. Initially the court followed a policy of adhering to a narrow principle
and tended to shy away from progress of the law. However, the mere existence of a
particular piece of beneficial legislation cannot solve the problems of the society at
large unless the judges interpret and apply the law to ensure its benefit to the
benefactors.

Responding to the changing times and aspirations of the people, the judiciary, with
a view to see that the fundamental rights embodied in the Constitution of India
have a meaning for the down-trodden and the under-privileged classes, pronounced
In Madhav Haskot‟s case that providing free legal service to the poor and needy
was an essential element of the ‗reasonable, fair and just procedure‟.
Again, in Hussainara Khatoon‟s case while considering the plight of the
undertrials in jail, speedy trial was held to be an integral and essential part of the
right to life and liberty contained in Article 21 of the Constitution.

In Nandini Satpathy v. D.L. Dani, the Supreme Court held that an accused has
the right to consult a lawyer during interrogation and that the right not to make
self-incriminatory statements should be widely interpreted to cover the pre-trial
stage also.

Again, in Sheela Barse v. State of Maharashtra, the Supreme Court laid down
certain safeguards for arrested persons.

In Bandhua Mukti Morcha‟s case, the Supreme Court held that right to life
guaranteed by Article 21 included the right to live with human dignity, free from
exploitation. The courts have, thus, been making judicial intervention in cases
concerning violation of Human Rights as an ongoing judicial process. Decisions on
such matters as the right to protection against solitary confinement as in Sunil
Batra v. Delhi Administration, the right not to be held in fetters as in Charles
Sohbraj v. Superintendent, Central Jail, the right against handcuffing as in T.V.
Vantheeswaran v. State of Tamil Nadu , the right against custodial violence as in
Nilabati Behera v. State of Orissa, or the rights of the arrestee as in D.K. Basu v.
State of West Bengal , the right of the female employees against sexual
harassment at the place of work as in the case of Vishakha v. State of Rajasthan ,
and Apparel Export Promotion Council v. A.K. Chopra are just a few pointers
in that directions and can be referred to by the members themselves. When there
was no existence of any compensatory jurisprudence, it was the Supreme Court
who ushered new hope by introducing right of compensation in case of ‗torture’
including mental torture inflicted by the State or its agencies. Using this weapon,
many tortured victims were provided their rightful compensations in cases like:
Rudal Shah v. State of Bihar , Bhim Singh v. State of Jammu and Kashmir ,
Saheli v. Comissioner of Police.

Assimilation of Rule of Law in Judicial Process

To witness the actual assimilation of Rule of law in judicial process, one has to
understand the machinery through which the Rule of Law could be furthered. First
from Dicey‘s viewpoint, it is the doctrine of separation of powers. In other words,
Constitution is supreme and the three organs of the Government viz. Legislature,
Executive and Judiciary are subordinate to it. The idea of separation of powers in
the strict sense may not be feasible for the functioning of a modern state, however,
the general object and Montesquieu‘s great point is that if the total power of
Government is divided among autonomous organs, one will act as a check upon
the other and in the check liberty can survive.

In Re: The Delhi Laws Act, 1912, the Supreme Court noted the absence of
specific provisions in the Constitutional document exclusively vesting legislative
powers in the legislature and judicial powers in the judiciary. Did the constitution
then, incorporate doctrine of separation of powers at all? The majority opinion,
however, imported the ‗essence’ of the doctrine of separation of powers and the
doctrine of constitutional limitation and trust implicit in the constitutional scheme.

Again, in the famous case of Indira Gandhi v. Raj Narain, the doctrine of
separation of powers was elevated to the position of a basic feature. It was
observed: ―The exercise by the legislature of what is purely and indubitably a
judicial function is impossible to sustain in the context even of our co-operative
federalism which contains no rigid distribution of powers but which provides a
system of salutary checks and balances. It is contrary to the basic tenents of our
Constitution to hold that the Amending Body is an amalgam of all powers-
Legislative, executive and judicial. ‗Whatever pleases the emperor has the force of
law‘ is not an article of democratic faith.‘‘

Our Constitution allows encroachment of one organ (Judiciary) upon another


(Legislature) in case of mala fide action. Therefore, there has been an expansion
of meaning of rule of law in this process. Rule of law is now regarded as a part of
the basic structure of the Constitution hence, its abrogation or destruction is not
even allowed by the Parliament.

As upheld in Kesavanda Bharti vs. State of Kerala, „Rule of Law‟ and


‗Democracy‘ were amongst the ‗Basic Structures‟ of the Indian Constitution not
amenable to the amending process under article 368 of the Constitution.

In Chief Settlement Commissioner Punjab v. Om Prakash, the Supreme Court


reiterated; ―In our Constitutional system, the central and most characteristic feature
is the concept of the Rule of Law which means, in the present context, the
authority of the law courts to test all administrative action by the standard of
legality.‖

The popular habeas corpus case, ADM Jabalpur v. Shivakant Shukla is one of
the most important cases when it comes to rule of law. In this case, the question
before the court was „whether there was any rule of law in India apart from
Article 21‟. This was in context of suspension of enforcement of Articles 14, 21
and 22 during the proclamation of an emergency. The majority of the bench (Ray,
C.J., Beg, Chandrachud and Bhagwati, JJ.) answered the issue in the negative
and observed:

―The constitution is the mandate. The constitution is the Rule of Law... There
cannot be any rule of law other than the constitutional rule of law. There cannot be
any pre Constitution or post Constitution Rule of Law which can run counter to the
rule of law embodied in the Constitution, nor there any invocation to any rule of
law to nullify the constitutional provisions during the times of emergency... Article
21 is our Rule of Law regarding life and liberty. No other rule of law can have
separate existence as a distinct right... The rule of law is not a mere catchword or
incantation. Rule of law is not a law of nature consistent and invariable at all times
and in all circumstances... There cannot be a brooding and omnipotent rule of law
drowning in its effervescence the emergency provisions of the Constitution.‖

Justice H.R. Khanna, however, did not agree with the majority view. In a powerful
dissent, His Lordship observed:

―Rule of law is the antithesis of arbitrariness. [It is accepted] in all civilised


societies. [It] has come to be regarded as the mark of a free society. It seeks to
maintain a balance between the opposite notions of individual liberty and public
order. The principle that no one shall be deprived of the life and liberty without the
authority of law was not the gift of the Constitution. It was necessary corollary of
the concept relating to the sanctity of life and liberty; it existed and was in force
before the coming into force of the constitution. Even in the absence of Article 21
in the Constitution, the State has got no power to deprive a person of his life or
liberty without the authority of law. This is the essential postulate and basic
assumption of the Rule of Law and not of men in all civilized nations.‖
The secondary meaning of rule of law is that the government should be conducted
within a framework of recognized rules and principles which restrict discretionary
powers. The Supreme Court observed in Som Raj v. State of Haryana that the
absence of arbitrary power is the primary postulate of Rule of Law upon which the
whole constitutional edifice is dependant.

The third meaning of rule of law highlights the independence of the judiciary and
the supremacy of courts. It is rightly reiterated by the Supreme Court in Union of
India v. Raghubir Singh ,that it is not a matter of doubt that a considerable degree
that governs the lives of the people and regulates the State functions flows from the
decision of the superior courts.

In the case of Sukhdev v. Bhagatram, Mathew J. declared that

It is indeed unthinkable that in a democracy governed by the rule of law the


executive Government or any of its officers should possess arbitrary power over
the interests of the individual. Every action of the executive Government must be
informed with reason and should be free from arbitrariness. That is the very
essence of the rule of law and its bare minimal requirement. And to the application
of this principle it makes no difference whether the exercise of the power involves
affection of some right or denial of some privilege.

Simultaneously in Amlan Jyoti Borooah vs. State of Assam and Ors., the
judiciary itself has introduced certain restrictions while maintaining a balance
between upholding Rule of Law and being mere benevolent. It held:

―Equity must not be equated with compassion. Equitable principles must emanate
from facts which by themselves are unusual and peculiar. A balance has to be
struck and the Court must be cautious to ensure that its endeavour to do equity
does not amount to judicial benevolence or acquiescence of established violation of
fundamental rights and the principles of Rule of law.‖

In the case of Bachan Singh v. State of Punjab, Justice Bhagwati has emphasized
that rule of law excludes arbitrariness and unreasonableness. To ensure this, he has
suggested that it is necessary to have a democratic legislature to make laws, but its
power should not be unfettered, and that there should be an independent judiciary
to protect the citizens against the excesses of executive and legislative power.
In addition to this in P. Sambamurthy v. State of Andhra Pradesh, the Supreme
Court has declared a provision authorizing the executive to interfere with tribunal
justice as unconstitutional characterizing it as „violative of the rule of law which
is clearly a basic and essential feature of the constitution.’

Rules of Law a myth or reality

The framers of our constitution had embodied the Rule of Law in the conscience
of our constitution. The three wings of the government work in coordination with
each other for the establishment of Rule of Law through the system of checks and
balances. The judiciary has worked efficiently towards the establishment of Rule
of Law and has been equally supported by civilians and government by obeying
the laws as laid down by the parliament and interpreted by the judiciary. Though
there also have been several instances when the public has resorted to violence
against an Act of Parliament or any Judicial pronouncement or doing acts contrary
to law which according to their perception is not contrary to Law and Justice,
resulting in situations where the principle of Rule of Law has just become a de
jure concept while in de facto the Rule of Men has prevailed.

The malicious practice of honor killing is prevalent in the Indian society


particularly in northern parts of the country. This practice includes the murder of a
member of a family, due to the belief of the perpetrators that the victim has
brought shame or dishonour upon the family, by violating the principles of a
community or a religion, usually for reasons such as divorcing or separating from
their spouse or for engaging in inter-caste marriage. The decision in this regard is
taken by an extra-constitutional body by the nomenclature of Khap Panchayat
which engage in feudalistic activities have no compunction to commit such crimes
which are offences under the Indian Penal Code, 1860. No heed is paid to the basic
human right of ―Right to life and liberty‖ as evident by the actions of the
panchayat. Choice of woman in choosing her partner in life is a legitimate
constitutional right. It is founded on individual choice that is recognised in the
Constitution under Article 19, and such a right is not expected to succumb to the
concept of ―class honour‖ or ―group thinking‖. It is because the sense of class
honour has no legitimacy even if it is practised by the collective under some kind
of a notion.
Several guidelines have been laid down by the court to curb the practice but still
there have been numerous instances of honour killing reported and the masses have
largely turned a deaf ear to the decision of the Apex Court.
The Sabarimala case verdict throws light upon the discretion of men in abiding
by the directions given by the Supreme Court only to the point if they are
analogous to the belief they hold. The court had allowed women of menstruating
age i.e. between 10-50 years to enter the temple premises for worship. The Lord
Ayappa temple has traditionally barred all women of menstruating age from
entering into the shrine. After the verdict massive protest were carried out, there
were also occurrences of violence against women who tried to enter the temple.
The women were denied their constitutional right to worship and the principles of
equality were violated even after the practice was declared as unconstitutional by
the Apex Court.
Another evil practice pertinent in the society is that of mob lynching. Lynching, a
form of violence in which a mob, under the pretext of administering justice without
trial, executes a presumed offender, often after inflicting torture and corporal
mutilation. The term lynch law refers to a self-constituted court that imposes
sentence on a person without due process of law. Supreme Court described it as
horrendous acts of mobocracy and stated that “the law, is the mightiest sovereign
in a civilized society” The majesty of law cannot be sullied simply because an
individual or a group generate the attitude that they have been empowered by the
principles set out in law to take its enforcement into their own hands and gradually
become law unto themselves and punish the violator on their own assumption and
in the manner in which they deem fit. Usually people belonging to religious and
caste-based minorities become the victim of this wicked practice. This practice is a
present-day example of a lawless society where there is denial of basic human
rights as well as fundamental rights.
Besides these, there is a plethora of other instances that are indicative of the
adulteration of the indigenous principle.

Conclusion
Thus, on the basis of the aforementioned information it can be undeniably deduced
that since its inception, the principle of Rule of Law owing to the dynamism
inherent in the very concept itself has evolved at a brisk pace. This development
can be accredited to the several laws laid down by the parliament and also through
the numerous judicial pronouncements.
However, despite all the development that the concept has undergone, Rule of Law
when analysed in the context of India does exist but it cannot be said to be
followed in stricto sensu. Instances often come to light when following a
particular law becomes subject to public convenience and they subscribe to such
law only if it is in line with their perception of right and wrong and is in
consonance with the ideologies that they uphold.

Legal Profession- Advocate Act, 1961

(Q) Explain main feature of the Advocates Act, 1961. [2019(5)(a)]

(Q) Discuss Law Commission of India‘s 26th Report (March 2017) on the
Amendments of Advocates act, 1961. [2017(8)(a)]

(Q)Comment on the objectives and performance of the advocate Act.1961.


[2015(3)]

Answer to Question [2019(5) (a)], [2015(3)]:

In order to implement the recommendations of All India Bar Committee (endorsed


by fourteenth Report of the Law Commission in 1955), the Advocates Act, 1961
came into being. The Act extends to the whole of India, except the State of Jammu
and Kashmir. The Act aims at amending and consolidating the laws relating to
legal practitioners and to provide for the establishment of State Bar Councils and
an All India Bar Council.

The objective of the Act is to integrate and constitute one class of legal
practitioners called ‗Advocates‘.

Secondly, it is aimed at prescribing a uniform qualification for the Bar.


It also aimed at creating an All India Bar Council and State Bar Councils.
SALIENT FEATURES OF ADVOCATES ACT

Following are some of the important salient features.

1. The Act has consolidated all the existing law on legal profession.

2. The Act has made provision for the creation of Bar Council of India at the
Central level and State Bar Councils in each state.

3. The Act has made the provision for the preparation of common roll of
Advocates throughout India.

4. It empowers Advocates whose name is in the common roll to practice in all the
courts in India.

5. The difference between the Advocates and Vakil is abolished and all those who
practice law is called as Advocates.

6. Provisions are made to confer the status as Senior Advocate for those Advocates
who have extraordinary knowledge in the field of law.

7. It has conferred autonomous status to the Bar Councils.

8. Establishment of an All India Bar Council and a number of State Bar Councils –
a federal structure for legal profession. An advocate is initially enrolled with a
State Bar Council and a common roll of all the advocates in the country is
maintained by All India Bar Council. An advocate on common roll has a right to
practice in any court of the country including the Supreme Court. The Advocates
Act was passed by the Parliament in the year 1961.

9. Integration of the bar into a single class of legal practitioners known as


advocates.

10. A uniform qualification for the admission as advocates viz, degree in law.

11. Division of advocates into senior advocates and other advocates based on
merit.

12. No advocate can get himself enrolled with more than one State Bar Council,
though he can get himself transferred from one State Bar Council to another
(i)Every State Bar Council consists of-
(i) the Advocate-General as the ex-officio member (for the Bar Council
of Delhi, since there is no Advocate-General for the Union Territory of
Delhi, the Additional Solicitor General was made the ex-officio member,
and when the office was made the ex-officio member, and when the
office was abolished, the Solicitor-General of India was made exofficio
members, and
(ii) in the case of the Bar Councils of Assam and Nagaland, Orissa, Delhi
and Himachal Pradesh fifteen other members and in the case of all the
other State Bar Councils twenty other members elected in accordance
with the system of proportional representation by means of the single
transferable vote from amongst the advocates on the electorate of the
State Bar Council concerned.
The Act as originally passed prescribed a term of six years for an elected
member of a State Bar Council subject to the principle of rotation but this
provision has since been amended and the term as now fixed is four years
from the date of the publication of the result of the election.
Even State Bar Council has
(i) one or more disciplinary Committee,
(ii) an executive Committee, consisting of five members,
(iii) an enrolment Committee, consisting of those members and
(iv) each other Committees as may be deemed necessary.
Every disciplinary Committee consists of two persons elected by the
Council from
amongst its members and one other person co-opted by the council who
have practiced for at least ten years, Each Bar Council has a Chairman
and a Vice-Chairman elected in such manner as may be prescribed and
has to appoint Secretary, and an accountant, if necessary
The functions of a State Bar Council are to admit persons as advocates on
its roll, to prepare and maintain such roll, to entertain and determine
cases of misconduct against advocates on its roll; to safeguard the rights
privileges and interests of advocates on its roll, to promote and support
law reform; to provide for the Election of its members; to perform all
other functions assigned to it by or under the Act and to do all other
things necessary for discharging the aforesaid functions.
(ii) The Bar Council of India: Membership and Function:
The Act provides that there shall be for the territories to which the Act
extends a Bar
Council known as the Bar Council of India, which consists of the
Attorney General of India, and the Solicitor-General of India as ex-
officio members, and one member elected by each State Bar Council
from amongst its members. There shall be a Chairman and a Vice
Chairman of the Bar Council of India elected as prescribed by rules and a
Secretary, and an Accountant, if any, under Section 7.
The functions of the Bar Council of India are to prepare and maintain a
common roll of advocates, to lay down standards of professional conduct
and etiquette for advocates, to lay down the procedure to be followed by
its disciplinary committee of each State Bar Council; to safeguard the
rights; privileges and interests, of advocates; to promote and support law
reform; to deal with and dispose of any matter arising under Act, which
may be referred to it by a State Bar Council to exercise general
supervision and control over State Bar Councils, to promote legal
education and to lay down standards of such education, the consultation
with the Universities whose degree in law shall be a qualification for
enrolment of an advocate and for the purpose visit and inspect
Universities; to manage and invest the funds of the Bar Council, to
provide for the election of its members; to perform all other functions
conferred on it by or under the Act and to all other thing necessary for
discharging the aforesaid functions.
The Bar Council of India is to have one or more disciplinary Committee,
a Legal Education Committee, an executive Committee, and such other
Committees as may be deemed necessary. Every disciplinary committee
is to consist of three members, two persons elected by it from amongst
advocates as have practised for at least ten years. The senior most
advocates from amongst its members is to be chairman of the committee.
The legal education committee consists of ten members of whom five are
persons elected by the Council from amongst its members and the other
five are those who are not members of the Council. The executive
committee consists or the members elected by the Council from amongst
its members.
The main source of income of the Bar Council of India is the contribution
of 40 per cent out of the fee of Rs. 250 paid by each applicant for
enrolment to the State Bar Council.
(iii) Senior Advocates:
Persons who were "senior advocates immediately before the appointed
day, i.e. 1-12-1961 have been deemed to be senior advocates for the
purpose of the Act. Besides, power has been conferred under Section 16
of the Act to the Supreme Court and the High Courts to designate
any advocate as senior advocate if in its opinion by virtue of his ability,
experience and standing at the Bar, he is deserving of such distinction.
Senior advocates are governed by the rules of the Supreme Court
applicable to them, and are also subject to the restrictions laid down by
the Bar Council of India in the interest of the legal profession.
(iv) Qualifications, for admission as an advocate:
Section 24 of the Act lays down the qualification for admission as an
advocate. Broadly speaking, the following are entitled to be admitted as
advocates, if they fulfill also certain other requirements laid down under
the Section:
(i) A person who has obtained a degree in law from any University in
India or from any University outside the territory of India, recognised by
the Bar Council of India for the purposes of the Act,
(ii) A barrister;
(iii) A vakil or a pleader who is a law graduate if his application for
enrolment was made within two years from the appointed day viz. 1-12-
1961;
(iv). A person who has for at least three years been a vakil or a pleader or
a mukhtar or was at any time entitled to be enrolled under any law for the
time being in force as an advocate of a High Court or of a Court of a
Judicial Commissioner or in any Union Territory.
An applicant for admission has to be a citizen of India and should have
completed the age of 21 years. He should pay a fee of Rs. 250/- to the
State Bar Council and should fulfill the other requirement as laid down
by the rules of the State Bar Councils, which generally include an
application for admission on the production of the certificate granted by
the University in respect of the degree in law granted by the University,
and character. However, a national of any other country may be admitted
as an advocate in a State roll, if he is a citizen of India duly qualified and
permitted to practise law in the country.
If the application is on the basis of the degree in law obtained from a
University in India or outside the territory of India, Section 24 (1) (d)
requires that the applicant should have undergone a course of training in
law and passed an examination both of which prescribed by the State Bar
Council. Under the rules of the State Bar Councils, the Course of training
which was originally for a period of one year, has subsequently been
reduced for an advocate to a period of six months. It has to complete
apprenticeship under an advocate of certain years standing, attendance at
Courts and at Chambers, maintenance of diaries, attendance at lectures,
and the passing of an examination on some subjects.
With regard to a barrister also, the Bar Council of India has specified, the
same requirement as to a degree in law. A provision to this sub-section as
amended provides that the clauses as to training and examination shall
not apply to:
(1) Any person who have obtained a degree in law from any University
in India on the results of an examination held before the 31st day of
March, 1964, or such other later date as may be prescribed, or a barrister
who, was the Bar before such date, or a barrister who, having qualified
later that date, has received such practical training in law as may be
recognised in this behalf by the Bar Council of India.
(2) Any person who has for at least two years held a judicial office in any
area which was comprised before the 15th day of August, 1947 within
India as specified in the Government of India Act, 1935, or has been an
advocate of any High Court in any such area:
(3) Any person who has practised before any High Court and who had
discontinued practice by reason of his taking up employment under the
government, or local authority or any other person: and
(4) Any other class of persons by reason of their legal training or
experience are declared by the Bar Council of India to be exempted from
the provision of this clause.
The last date mentioned in the proviso had owing to agitations and
representations, frequently been extended and sub-section was amended
as above in 1964. Even the amendment of 1964 could not be final on the
subject. The Government of India's latest notification relevant on the
matter has narrowed down the scope of the requirement as to training and
examination practically making it a nullity. Training and examination
will not be necessary according to the notifications if the applicant has
taken the degree of law (after undergoing a course of instruction the three
years (laid down by the rules of the Bar Council of India) or has been
called to the Bar on or before 31.12.1968, or if he had taken the master's
degree in law.
(v) Legal Qualifications:
The degree in law mentioned as one of the qualifications for admission,
may be degree in law:
(i) of a university in India, or
(ii) of a university contains the territory of India.
(iii) A degree in law of University in India;
The standards laid down by the Bar Council of India on legal education
and recognition of degree of law of the University prescribes inter alia
the following conditions:
A person to be eligible to join the course of study in law should be a
graduate of University or hold such academic qualification which is
considered by the Bar Council of India equivalent to a graduate degree of
a University. The duration of the course of instruction shall be three
years. This is with reference to those who join the course of instruction in
or after the first term of the academic year 1967-68. In the case of
fourteen universities, however, for reasons considered sufficient by the
Bar Council of India, it was the first term or the academic years 1968-69
instead of 1967-68. The course of instruction comprises ten compulsory
subject and six others as optional as mentioned in the rules.
During the last year of the course, the instruction and practical training
should also be imparted for a period of six months in the rules of courts
and in drafting and pleadings and documents.
According to the resolution of the Bar Council of India, practical training
would include pleading and conveyancing, moot courts and conducting
civil and criminal proceedings, attending courts; maintaining a record of
the above three requirements and arrangements of at least six lectures of
professional ethics, attendance at four of which shall be compulsory. The
scope and nature of training as set out in this resolution is not to be
deemed as exhaustive. It should thus be open to the Universities and law
colleges to add to the same and work with Bar Council of India at the
earliest. Practical training should be made a part of the examination
system for which marks should be assigned and awarded.
The rules on standards of legal education were made, as required under
the Act, in consultation with the Universities imparting legal education
and the State Bar Councils. In accordance with the above rules, the Bar
Council of India in exercise of the powers conferred under section 24 (i)
(c) (iii) of the Advocates Act, has recognised degrees in law obtained
from any of the Universities which fulfills the requirements of rule II of
its rules and obtained by those who had commenced the three-year -
course of instruction as provided for, in the first terms or the academic
year 1967-68 by actual attendances at colleges. Under the powers vested
in it, it has predicted the postponement of the three-year course to the
academic year 1968-69 in the case of the Universities of Kanpur, Madras,
Meerut, Bangalore, Aligarh, Mysore, Bhagalpur, Patna, Agra, Ranchi,
Behrampur and Jiwaji, Sardar Patel and Vikram Universities, one of the
functions of the Bar Council of India is as mentioned earlier, to visit and
inspect Universities, imparting legal education.
(i) Conditions for admissions to roll:
Section 28 of the Act empowers the State Bar Councils to make rules,
subject to the approval of the Bar Council of India. The rules can be
made, inter alia, on the conditions subject to which a person may be
admitted as an advocate on the roll of Bar Council. The rules accordingly
made by State Bar Councils generally prohibit the enrolment of a person
who though he may be otherwise qualified,, is in full or part time service
or employment or is engaged in any trade or profession. The prohibition
against enrolment does not apply to certain categories of persons
specifically referred to in the proviso to the rule such as 'law officers'
fulfilling certain conditions, articled clerk or an attorney, a person in part-
time service as a professor or Lecturer, teacher in law of a person who by
virtue of his being a member of a Hindu Joint Family has an interest in a
Joint management thereof or other classes of persons specifically
exempted after approval by the Bar Council of India.
The compulsory subjects are: Indian Legal and Constitutional History,
Contracts, Torts, Family Law including Hindu and Mohammedan Law;
Evidence, Legal Theory (Jurisprudence and Comparative Law): Civil
Procedure, Limitation and Arbitration.
(ii) Application for admission to roll:
An application for admission as an advocate has to be made in the
prescribed form to the State Bar Council within whose jurisdiction the
applicant proposes to practice. Applications for enrolment have to be
referred to and disposed of by the enrolment committee of the State
Bar Council. But if the enrolment committee proposed to refuse such
applications, it has to refer the application with the statement of the
grounds in support of the refusal to the Bar Council of India, and has to
dispose of the application finally in conformity with such opinion.
(iii) Right to practise
Chapter IV of the Advocates Act regulates the right of Advocate to
practice. One of the objects of the Act is to have in course of time only
one class of legal practitioners. Section 29 provides that subject to the
provisions of the Act as from the appointed day 1.6.1969, there shall be
only one class of persons entitled to practice throughout the territories
through which the Act extends in all courts including the Supreme Court,
before any tribunal or person legally authorised to take evidence of
before any other authority or person before whom such advocate is
entitled to practise. Further under Section 33 advocates alone are entitled
to practise in any Court.
(iv) Practice by persons not entitled to practice:
Persons illegally practising in Courts or before other authorities when
they are not entitled to practise under the provisions of the Act are liable
for punishment with imprisonment for a term which may extend to six
months.
(v) Some important powers of the Bar Council or India:
Apart from the other powers already enumerated the Bar Council of India
has been specifically conferred certain special powers.
1. Power to remove name from the rolls.
The Bar Council of India is empowered, either on a reference made to it
or otherwise, if it is satisfied that any person has got his name entered in
the roll of the Advocates by misrepresentation, to remove such person
from the roll of advocates after giving him an opportunity of being heard.
The Bar Council of India or any State Bar Council may also remove from
the common roll or the State roll as the case may be, the name of any
advocate, who is dead or from whom a request has been received to that
effect. Besides, the name of advocate may be removed from the roll as
punishment for misconduct in disciplinary proceedings.
2. Revision
Apart from the power vested in it to remove the name of an advocate in
certain cases, and the power vested in its disciplinary committees to hear
and dispose of the disciplinary matters whether by way of original
hearing or on appeal, the Bar Council of India has the power at any time
to call for the record of any proceedings under the Act, which has been
disposed of by a State Bar Council or a Committee thereof, and from
which no appeal lies, for satisfying itself as to the legality or propriety of
such a disposal and may pass such orders thereon as it deemed fit. No
order which prejudicially affects any person can be passed without giving
him a reasonable opportunity of being heard.
3. Directives:
Section 48 B empowers the Bar Council India for the proper and efficient
discharge of the functions of a State Bar Council or any committee
thereof, to give such directions to the State Bar Council or its committees
as may appear it to be necessary, and the State Bar Council or the
Committee has to comply with the directions. Where a State Bar Council
is able to perform its functions for any reason whatsoever, the Bar
Council of India may give such directions to the ex-officio member
thereof as may appear to it to be necessary, and such directions shall have
effect, notwithstanding anything contained in the rules made by the State
Bar Council.
4. Rules 'to make and approval'
Section 15 enumerates the powers of the State Bar Councils and the Bar
Council of India to make rules on the matters dealt with Chapter II of the
Act relating to the Bar Councils. Section 28 gives power to the State Bar
Councils to make rules on some matters connected with the preparation
of rolls, training and examinations for admission of advocates, form of
application for enrolment and conditions for enrolment. Any rule made
by State Bar Council, whether under Section 15 or 28 shall have effect
only if it has been approved by the Bar Council of India. It may be noted
in this connection that under the Indian Bar Councils Act, 1926, the rules
made by the Bar Councils required the previous sanction of the High
Court.
Section 49 confers on the Bar Council of India a general power to make
rules for discharging its functions under the Act. Besides the rules
referred to earlier in this article they include rules on standards of
professional conduct and etiquettes by Council advocates , principles for
guidance of the State Bar Councils and the manner in which directions
issued or orders made by the Bar council of India may be enforced, and
‗any other matter which may be prescribed‘. The Bar Council of India
has made its rules on all these matters.
Supreme Court Rules to regulate the legal profession
Prior to the coming into force of the Constitution, the Federal Court was
empowered under Section 214 of the Government of India Act, 1935 to
make rules, with the approval of the Governor General, for regulating the
practice and procedure of the Court including rules as to the persons
practicing before it. The Federal Court rules, 1942 prescribed the
qualifications of persons to be enrolled before it. The rules permitted the
persons of ten year standing to be enrolled as senior advocates. No
person could appear as an advocate before the Federal Court unless
instructed by an agent.
Article 145(1) of the Constitution empowers the Supreme Court to make
rules, with the approval of the President inter alia as to persons practicing
before it. The Advocates Act, 1961 confers on all advocates whose
names are on the rolls maintained under that Act to practice, inter alia, in
the Court. The Supreme Court Rules, 1950 made in exercise of powers
under Article 145(1) provides for the enrolment of persons as advocates
who are entitled to practice before it. The rules laid down that the role of
Supreme Court shall be in two parts, seniors and others. Persons who had
more than ten years standing could be enrolled as senior advocates.
Order IV of the Supreme Court Rules deals with the Advocates. Rule
2(b) places some restrictions on the senior advocates. Rule 3 provides
that every advocate appearing before the court shall wear robes and
costumes as may from time to time be directed by the Court. In
pursuance of this rule the Chief Justice of India directed that the costume
that be worn by the advocates appearing before the Supreme Court
should be black coat, robe and band worn by the barristers appearing
before the High Courts. The rules also provide for registration of
advocates on record and advocate who is not a senior advocate. Rule 5
lays down that an advocate shall not be qualified to be registered as an
advocate on record unless he has undergone training for one year with an
advocate on record approved by the Court and has thereafter passed such
tests as may be held by the Court for advocates who apply to be
registered as advocates on record. The particulars of the test are to be
notified in the official gazette from time to time. The rules however,
exempt an attorney from such training and test.
An advocate on record is required to have an office in Delhi within a
radius of ten miles from the Court house and give an undertaking to
employ within one month of his being registered as an advocate on
record, a registered clerk. He is also required to pay a registration fee of
twenty five rupees.
The Supreme Court Advocate (Practice in High Court) Act, 1951,
conferred the right on every advocate of the Supreme Court, to practice
in any High Court whether or not he is an advocate of that High Court. In
Aswani Kumar Ghosh v. Arabindha Ghosh, the Supreme Court held
by a majority that an advocate of the Supreme Court became entitled as
of right to appear and plead as well as act in all the High Courts in India
and that he is also entitled to appear on the original side of the West
Bengal and the Bombay High Courts without being instructed by an
attorney and free from the restrictions in this regard to the Rules of
the High Courts of West Bengal and Bombay.
In view of the provision of the Advocates Act, 1961, there has been no
further admission of persons as advocates of the Supreme Court after
1.12.1961, when Chapter III of the Advocates Act came into force.
Under Section 50 (3) (d) of the Advocates Act, 1961- the Supreme Court
Advocates (Practice in High Court) Act, 1951 shall stand repealed on the
date when Chapter IV of the Advocate Act comes into force and as noted
earlier, Section 35 in Chapter IV of the Act has not yet been brought into
force.

Answer to Question [2017(8)(a)] :


The Law Commission, headed by chairman and former Supreme Court
judge Justice BS Chauhan, has released its draft Advocates Act
(Amendment) Bill 2017, including proposals by the Bar Council of India
(BCI) and other stakeholders after the Commission was tasked by the
Supreme Court to look at the BCI‘s failure to do its statutory duty of
regulating the profession.

(1) Foreign Lawyer Entry

The Law Commission has proposed several changes, including creating provisions
that would enable the BCI to allow the entry of foreign law firms,
notably amending the definition of ‗advocate‘ to include ―a foreign lawyer
registered under any law in a country outside India and recognised by the Bar
Council of India‖.

(2)Strikes not good, The Law Commission would like the BCI:

to make rules to deal with strikes, boycotts or abstentions from courts by the
Advocates, provide for suitable measures in this regard and to provide for
punishments including the punishment of disqualification from contesting any
election of Bar Councils or of Bar Association for a period of six years;
(3)Those suffering loss from lawyer strike, should get compensation

If any person suffers loss due to the misconduct of the advocate or for his
participation in strike or otherwise, then, such person may make a claim for
compensation against the advocate in the appropriate forum established under any
law for the time being in force.
The non-payment of fees, either in full or part, by a person to his advocate shall not
be a defence available for the advocate against whom such claim for compensation
is made.
(4)State bar council terms capped at two consecutive terms

An advocate who has been elected as a member of the State Bar Council
consecutively for two terms shall not be eligible to contest elections under clause
(b) of sub-section(2), for the ensuing next term of the State Bar Council.

(5)Misconduct defined

For the avoidance of doubt, in case there was any, the Commission has inserted a
definition for misconduct into the Act:

‗misconduct‘ includes-an act of an advocate whose conduct is found to be in


breach of or non-observance of the standard of professional conduct or etiquette
required to be observed by the advocate; or forbidden act; or an unlawful
behaviour;or disgraceful and dishonourable conduct; or neglect; or not working
diligently and criminal breach of trust; or any of his conduct incurring
disqualification under section 24A
(6)Not much reform of legal education, BCI to remain in charge

according to Law Commission

Legal education in India should be structured in a manner where the BCI, along
with legal academics may endeavour to innovate, experiment and compete
globally. A balance should be maintained in order to change the entire fabric of
legal education system in India, keeping in mind the necessity of globalisation.

(7)Continuing legal education administered by BCI and ‘pre-enrolment’

training

Under the draft amendment, the Law Commission has proposed that the BCI
would be responsible: to provide for continuing legal education for advocates;
Also, the BCI is: to provide for pre-enrolment training and apprenticeship to a
person who has obtained from a recognised institution degree in law for a period
not exceeding one year;

(8)Continuing legal education administered by BCI and „pre-enrolment‟


training
Under the draft amendment, the Law Commission has proposed that the BCI
would be responsible: to provide for continuing legal education for advocates;
Also, the BCI is: to provide for pre-enrolment training and apprenticeship to a
person who has obtained from a recognised institution degree in law for a period
not exceeding one year;

(9)A CLAT under BCI? The Law Commission also recommended that the

BCI:

Provide for Entrance Test for admission in the Institutions imparting legal
education in the country and to provide for measures for improvement of legal
education and to make provision for on-line teachings for all the law students of
the country either directly or through some charitable Institution
(10)And bar association membership apparently to be compulsory for

litigators

An advocate enrolled with the State Bar Council engaged in or intend to practice
before a court of law, tribunal or before any authority or person shall get himself
registered as a member of the Bar Association where he ordinarily practices or
intends to practice law.

Utilization of Library in Legal Studies

(Q) Effective utilization of library in legal studies [2015(1) (c)]

Law libraries have been recognized as the ―heart‖ of every law faculty in all
universities. It is the pivot on which the faculties revolve for attainment of their
educational excellence, goals and objectives. The assemblage of books and non-
book information purveyors in the required quantity and quality depends on good
collection development activities. Library is central to and it is in fact the
laboratory for the legal profession.
HOW TO FIND BOOKS, LEGAL ACADEMIC JOURNALS AND LAW REVIEWS

Books are arranged in a classified manner based on subjects with the stack of the
Library. Generally, Universal Decimal Classification System especially designed
for arranging research library books and other reading materials is used in the law
library. Library housekeeping software is used to maintained bibliographical
information of books. Books on around almost all universe of law subjects may be
found with the help of Online Public Access Catalogue maintained through
computers within the Library. Books may be searched through title, author,
subject, publisher, combination search etc. There are several publishers‘ websites
which provide online access of contents of books published by them. E-Hart,
Questa are few examples of online book databases.
HOW TO FIND LEGISLATION

In case of any ambiguity while interpreting the provision of any statute, judges
have to follow the “legislation‖ of the legislature for enacting a particular subject
matter. The legislation of any provision can be ascertained with the help of the
following tools:

 Objects and Reasons of the Act (published in the bill) Parliamentary debates

 Law Commission Reports (if the bill has been introduced on the recommendation
of the Law Commission)

 Standing Committee/ Joint/Select Committee Reports

 Reports of the Committee appointed by the ministries for enacting/reviewing


any existing enactments.
HOW TO FIND LAW ARTICLE PUBLISHED IN LAW JOURNALS

Law libraries generally maintain article indexing & abstracting system. It is a pre
requisite of a law library to maintain an indexing system containing bibliographical
details of articles published in each academic journal received within the library. A
researcher can browse these articles by title, author, name of the journal, subject
and combination search through online public access catalogue. Beside, in-house
database, the library may also subscribe index to periodicals as published by
several publishers and libraries e.g. in India, ―Index to Indian Legal Periodicals‖
ILI, New Delhi, 2008 is published by the library of the Indian Law Institute. Index
to Legal Periodicals as published by H.W. Wilson, may be browsed for academic
articles published in foreign journals. Online databases have changed the way of
legal research. The following databases are used for searching legal articles.

1. Westlaw International

2. JSTOR

3. Social Science Research Network

4. HeinOnline

5. Global Legal Information Network

6. LexisNexis Online
HOW TO FIND CASE LAW

Law libraries must maintain a sound collection of Reporting Journals at


international and national level. Library specific to law must subscribe at least one
reporting journal from each state.
Foreign Law Reports

• All England law Reports

• Australian Law Reports

• Canadian Supreme Court Reports

• Commonwealth Law Report

• Dominion Law Reports

• Federal Law Report


• English Reports

•Law Reports Reprint Rainbow Series 1874 onwards


• US Supreme Court Reports

• Weekly Law Reports


Indian Law Reports

• Supreme Court Reports

• Supreme Court Cases (SCC)

• All India Reporters (AIR)

• Scale • Judgments Today

• Indian Law Reports

• Law Reports of all States

Digests and Commentaries are the starting tools for finding cases on particular
subject based on nominal, index, subject index and court index. All India Reporter
Manual, Supreme Court Digest, Labour Law Digest, Criminal Law Digest and All
Indian High Court Case Manual are some example of leading digests.

In digital era, Case law on a particular subject or party name or citation, may be
searched with the help of various online databases. Westlaw International,
LexisNexis may be used to search foreign caselaw. SCC Online, AIR SC& HC,
Criminal Law Journal, Manupatra, Indlaw, Law Premium may be used to search
Indian case law.
TO FIND CONSTITUTIONS OF DIFFERENT COUNTRIES

A constitution is the system of fundamental principles by which a political body


(state or nation) governs itself. Law libraries have a separate section maintaining
constitutional laws of all countries in the world. Constitution of any country of the
world may also be downloaded from various websites i.e. http://confinder.
richmond.edu/
WHERE TO FIND COMMISSION & COMMITTEE REPORTS

In India, various commissions and committee are in existence like Women


Commission, Commission for SC/ST National Human Right Commission etc.
Reports of such commissions are maintained within the law library Collection.
Parliamentary Committee Reports are also major sources of legal information
which may be referred through website of Parliament of India. Annual Reports
of the Government Departments are also useful for legal research.
HOW TO FIND LEGISLATION

Law library must have a good collection of following International and National
Legislations i.e. Bills and Acts of the concerned Parliaments like: US Supreme
Court Code, General Public Acts (UK), Australia Consolidated Common wealth
Act, AIR Manual, India Code, Act of the Parliament, Civil Court Digest, Gazette
of India, Current Central Legislations, and Current Indian Statutes etc. Acts and
Legislations of all countries are also provided through online services.

PARLIAMENTARY DEBATES AND PARLIAMENTARY COMMIT-TEE A law


library provides a sound collection of Parliamentary Debates of Rajya Sabha and
Lok Sabha. Parliamentary debates may be downloaded from the website of
Parliament of India from XIth Lok Sabha 1996 onwards. All Parliamentary
Committee Reports as published by the Parliament of India are also browsed to
disseminate students within the library through online access. Electronic version
of Command Papers i.e. Debates of House of Lords and House of Commons are
also available on the websites of UK Parliament.
http://www.parliament.uk/parliamentary_publications_and_archives/parliamentary
_archives/archives_electroni c.cfm
TREATIES AND INTERNATIONAL AGREEMENTS

Law Libraries maintain several research tools containing international treaties and
agreements like Encyclopaedia of the United Nations, Consolidated Treaty Series,
and League of the Nations Treaty Series. Treaties as available in digital form in
various online databases are also provided to students and end users in anywhere
in the world.
SEMINAR REPORTS AND THESIS/DISSERTATIONS

Law libraries, especially supporting university system maintain International and


National Seminar Reports conducting inside and outside of the countries. Digital
version of these reports may be preserved within the law library. Thesis and
dissertation submitted by research scholars may also be useful for legal
researchers.
TYPE OF REFERENCE TOOLS

Law libraries maintain a good collection of reference tools like Index to Legal
Periodicals, Legal Encyclopedias, Legal Dictionaries, Professional Legal
Directories,Legal Bibliographies, Biographies etc. Besides a law library must also
maintain subject based search tools like Corpus Juris Secundum, American
Jurisprudence, Halsbury‘s Laws of England, Halsbury‘s Laws of India, Supreme
Court Yearly Digest, Criminal Law Digest, Supreme Court Case Citator, Supreme
Court Case Comparative Tables etc.
ELECTRONIC DATABASES USEFUL FOR LEGAL RESEARCH

A number of electronic databases are maintained by commercial companies as


information source providers. They hire team of subject experts for integrating
into comprehensive database and for enhancing editorial skills and other technical
aspects like searchable documents, advances web technologies and easy to use
interface.

Westlaw India & International Westlaw India is specially designed for South
Asia law researchers facilitate comprehensive search of Indian primary and
secondary sources. Westlaw India provides access to Case Law, Legislation, Law
Reviews, Treatises, and Directories organized by topical and jurisdictional
libraries with editorially enhanced like head notes, citatory and legal update alerts.

LexisNexis India : (www.lexisnexis.com) LexisNexis India and Academic


encompasses news, business and legal topics. It contains more than 6000 sources
from all over the world, drawn from print, broadcast and online media It includes
Butterworths® in the United Kingdom, Canada, the Asia-Pacific region, Les
Editions du Juris Classeur in France, and MartindaleHubbell® and Matthew
Bender® worldwide. . This database provides collection of comprehensive legal
information including legal news, law reviews and journals, Case Laws of major
countries around the world, Statutes and reports of national and international
spheres. The important feature of Lexis India is it has included more than 90 e
books and commentaries on law and legal subjects published by LexisIndia and
Wadhwa Publications, Legal dictionaries and lexicons.
JSTOR:( www.jstore.org)Journal Store i.e. JSTOR facilitates scholars,
researchers, and students discover, use and build upon a wide range of content in
digital achieve. It provides a high quality, interdisciplinary achieve to support
scholarship and teaching. It includes archives of over one thousand leading
academic law journals titles and other materials valuable for academic work.

Hein Online HeinOnline provides American, foreign, and international legal


sources, including law journals, foreign case reports, federal administrative
regulations, federal statutes, U.S. treaties and agreements, historical legal treatises,
among others. HeinOnline is accessible through www.heinonline.org/

Kluwer Arbitration Online


KluwerArbitration.com is the online resource for international arbitration
research. It contains commentary from authors and an extensive collection of
primary source materials, exclusive materials including ICC cases and awards.
KluwerArbitration.com online database is developed and maintained in
conjunction with two partners i.e. International Council for Commercial
Arbitration (ICCA) and Institute for Transnational Arbitration (ITA). The database
can be accessed through its link i.e
http://www.kluwerarbitration.com/default.aspx#

SCC Online SCC Online is a product of Eastern Book Company. It provides


digital content access of Supreme Court Cases Journals published by EBC. Apart
from Supreme Court Judgements, this database also contains Indian legislations,
legal articles, Commissions & Committee Reports, Notifications and Circulars.
Being an IP based access users can registered on the home page of
www.scconline.co.in with simple two fold steps.

Manupatra Manupatra is India‘s most comprehensive online legal and business


policy database. Manupatra contains judgements of Supreme Court of India and
High Courts of Indian states, Commission & Committee Reports, Gazette
Notifications & Circulars, Bare Acts, Rules & Regulations, Ordinance & Pending
Cases and legal material on subject based research. It provides comprehensive
search techniques including Manu Search, Legal Search, Citation Search and Act
Search. Manupatra can be search through www.manupatra.com
CLA Online Corporate Law Advisor is a product of Corporate Law Advisor. It
provides digital contents of Corporate Law Advisor magazine along with Business
Law Supplement. It covers all volumes since its inception of both Corporate Law
Advisor and Business Law Supplement. It provides comprehensive search
techniques like article search, case law search, notification search, circulars
search, acts search, rules search and regulations search. It can be accessed through
http://www.claonline.in/

Indiastat It provides depth of India specific, socio-economic statistical facts and


figures culled from various secondary sources it is a portal of state specific sites
which provide statistical data for all the major socio-economic parameters of the
Indian States. District level data where ever available can also be viewed. Through
this exhaustive compiled data can be accessed and download in MS-Excel/HTML
formats. Can be accessed http://www.indiastat.com

Taxman-Online Taxman online is leading publishers on Taxes and Corporate


Law`s, Accounting and Auditing, Banking, Finance and Management. Can be
access through http://www.taxmann.com/

Economic & Political Weekly First published in 1949 as the Economic Weekly
and since 1966 as the Economic and Political Weekly, EPW, as the journal is
popularly known, occupies a special place in the intellectual history of
independent India. For more than five decades EPW has remained a unique forum
that week after week has brought together academics, researchers, policy makers,
independent thinkers, members of non-governmental organisations and political
activists for debates straddling economics, politics, sociology, culture, the
environment and numerous other disciplines. Along with its archival and current
issues, EPW can be accessed through http://www.epw.in/
ADVANTAGES OF DIGITAL LIBRARIES
i.) No physical boundary: The user of a digital library need not go to the library
physically, as long as an internet connection is available.
ii.) Round the clock availability: A major advantage of digital libraries is that
people can gain access to the information at any time night or day.
iii.) Multiple accesses: The same resources can be used simultaneously by a
number of institutions and patrons.
iv.) Information retrieval: The user is able to use terms (word, phrase, title, name
and subject) to search the entire collection.
v.) Preservation and conservation: Digitalization is a long term preservation
solution for physical collections.
vi) Space: Whereas traditional libraries are limited in storage space, digital library
have the potential to store much more information simply because digital
information requires very little physical space to contain them and media storage
technologies are more affordable than ever before.

Added value: Certain characteristics of objects primarily the quality of images may
be improved Digitalization can enhance legibility and remove visible flaws such as
stains and discoloration.
DISADVANTAGES AND PROBLEMS
Lack of constant power supply as experienced in India is a serious drawback. The
backbone of a virtual library is availability of power supply at all times to cool the
air conditioners, the machines and make the hard wares and software work.

The main object of a virtual library is to reduce operational cost. But instead a lot
of cost is incurred because of high import tariffs on hard wares, soft wares and
spares. Installation, maintenance and processing costs are generally high.
In India the ICT infrastructure is poorly developed. The ICT skills are very
inadequate. It will therefore, not be easy to manage the virtual libraries.

You might also like